Download Document

Document related concepts

United Arab Emirates Legal Process wikipedia , lookup

Insanity defense wikipedia , lookup

Criminalization wikipedia , lookup

Right to silence wikipedia , lookup

Transcript
CRIMINAL PROCEDURE
Prepared by Andrew Carvajal
Instructor: Prof. Alana Klein
Fall 2008
Notes:
1. Most of the material summarized corresponds to the casebook “Learning Canadian Criminal Procedure – 9th
Edition” by Stuart et al.
2. Feminine pronouns are generally used as neutral pronouns throughout the text, unless the facts of the case require
otherwise.
3. Many thanks to the law clerks with the tedious task of putting case headnotes together. Their work doesn’t go
unappreciated.
1
Index
Jurisdiction .................................................................................................................................... 12
Legislative division of powers .................................................................................................. 12
s. 91 - Constitution Act, 1867 ............................................................................................... 12
s. 92 - Constitution Act, 1867 ............................................................................................... 12
R. v. Trimarchi (1987) .......................................................................................................... 13
A.G. Canada v. C.N. Transportation (1983)......................................................................... 13
Classification of offences .......................................................................................................... 13
Indictable offences .................................................................................................................... 13
A note on jurisdiction................................................................................................................ 14
s. 468 – Criminal Code ......................................................................................................... 14
s. 469 – Criminal Code ......................................................................................................... 14
s. 552 – Criminal Code ......................................................................................................... 15
s. 553 – Criminal Code ......................................................................................................... 16
Summary conviction offences................................................................................................... 17
Crown elections offences (dual, hybrid) ................................................................................... 17
s. 34(1) - Interpretation Act ................................................................................................. 18
Time limitation in Canadian law............................................................................................... 18
s. 786(2)– Criminal Code ...................................................................................................... 18
Territorial limitations ................................................................................................................ 18
s. 6(2)– Criminal Code .......................................................................................................... 18
s. 24 Canadian Charter of Rights and Freedoms................................................................... 20
R. v. Mills (1986) .................................................................................................................. 21
R. v. Big M Drug Mart .......................................................................................................... 21
s. 52(1) Constitutional Act, 1982 .......................................................................................... 21
Police Stopping Powers ................................................................................................................ 22
s. 9 – Canadian Charter of Rights and Freedoms.................................................................. 22
s. 10 – Canadian Charter of Rights and Freedoms................................................................ 22
R. v. Deadman (1985), 46 CR (3d) 193 ................................................................................ 23
Doctrine of ancillary police powers .......................................................................................... 24
Stop Powers and Racial Profiling ............................................................................................. 24
1. Vehicle Stops ........................................................................................................................ 24
2. Investigative Detention ......................................................................................................... 25
R. v. Simpson (1993), 20 CR (4th) 1 ..................................................................................... 25
2
R. v. Brown (2003), 9 CR (6th) 240 ...................................................................................... 26
R. v. Mann, [2004] 3 SCR 59 ................................................................................................ 27
3. Roadblock Stops ................................................................................................................... 29
Search and Seizures ...................................................................................................................... 31
s. 8 – Canadian Charter of Rights and Freedoms.................................................................. 31
Before the Charter .................................................................................................................... 31
Search and seizure under the Criminal Code............................................................................ 31
s. 487 – Criminal Code ......................................................................................................... 32
s. 487.1(1) – Criminal Code .................................................................................................. 32
s. 487.11 – Criminal Code .................................................................................................... 32
s. 488 – Criminal Code ......................................................................................................... 33
s. 489 – Criminal Code ......................................................................................................... 33
s. 29 – Criminal Code ........................................................................................................... 33
Laporte v. Laganière (1972), 18 CRNS 357 ........................................................................ 34
Two special warrant powers to search the person .................................................................... 35
Three main requirements of search warrants ............................................................................ 35
FORM 1 ................................................................................................................................ 36
Search and seizure under the common law ............................................................................... 36
Three types of search powers recognized at common law ....................................................... 36
A) Search incident to Arrest ..................................................................................................... 37
R. v. Brezack, [1949] OR 888 ............................................................................................... 37
Laporte v. Laganière supra (1972), 18 CRNS 357 .............................................................. 38
R. v. Tomaso (1989), 70 CR (3d) 152 ................................................................................... 38
R. v. Stillman (1989), 70 CR (3d) 152 .................................................................................. 39
R. v. Caslake, [1998] 1 SCR 51 ............................................................................................ 42
B) Consent to search ................................................................................................................. 44
Law Reform Commission of Canada, Working Paper 30: Police Powers – Search and
Seizure in Criminal Law Enforcement ................................................................................. 44
R. v. Deadman supra (1985), 46 CR (3d) 193 ...................................................................... 44
Note on R. v. Wills (1992), 70 CCC (3d) 529 ....................................................................... 45
Note on R. v. Lewis (1998), 122 CCC (3d) 481 .................................................................... 45
C) Consent incident to duties of police officers if reasonably necessary (ancillary powers
doctrine) .................................................................................................................................... 46
Colet v. R., [1981] 1 SCR 2 .................................................................................................. 46
R. v. Deadman supra (1985), 46 CR (3d) 193 ...................................................................... 47
3
Doctrine of ancillary police powers .......................................................................................... 48
Other federal statutes ............................................................................................................ 49
R. v. Noble (1984), 48 OR (2d) 643 ...................................................................................... 49
Provincial statutes ..................................................................................................................... 50
Constitutional minimum standards ........................................................................................... 50
s. 8 – Canadian Charter of Rights and Freedoms.................................................................. 50
R. v. Rao (1984), 46 OR (2d) 80 ........................................................................................... 50
Hunter et al. v. Southam Inc., [1984] 2 SCR 145 ................................................................. 52
R. v. Collins, [1987] 1 SCR 265............................................................................................ 54
R. v. Edwards, [1996] 1 SCR 128 ......................................................................................... 55
R. v. Tessling, [2004] 3 SCR 432 .......................................................................................... 57
R. v. Kang-Brown, 2008 SCC 18 .......................................................................................... 58
R. v. A.M., 2008 SCC 19 ....................................................................................................... 61
R. v. Wong, [1990] 3 SCR 36 ................................................................................................ 63
Lecture by Sarah Henningsson - Provincial prosecutor ............................................................ 64
Arrest............................................................................................................................................. 65
R. v. Whitfield ........................................................................................................................ 65
Citizen’s Power of Arrest.......................................................................................................... 65
s. 494 – Criminal Code ......................................................................................................... 65
s. 31 – Criminal Code ........................................................................................................... 66
s. 495 – Criminal Code ......................................................................................................... 66
R. v. Storrey, [1990] 1 SCR 241 ........................................................................................... 67
R. v. Biron, [1976] 2 SCR 56 ................................................................................................ 68
Roberge v. R .(1983) ............................................................................................................. 70
Constitutional Minimum Standards (Charter ss. 9 and 10(a)) .................................................. 70
R. v. Duguay (1985), 45 CR (3d) 140 ................................................................................... 71
Reasons for Arrest..................................................................................................................... 72
Christie v. Leachinsky (1947) ............................................................................................... 72
Gamracy v. R. (1974) ............................................................................................................ 72
R. v. Evans, [1991] 1 SCR 869 ............................................................................................. 72
Entry into Premises ................................................................................................................... 74
Ouimet Report ....................................................................................................................... 74
R. v. Feeney, [1997] 2 SCR 13.............................................................................................. 74
s. 529 – Criminal Code ......................................................................................................... 77
4
s. 529.1 – Criminal Code ...................................................................................................... 77
s. 529.2 – Criminal Code ...................................................................................................... 78
s. 529.3 – Criminal Code ...................................................................................................... 78
R. v. Godoy, [1999] 1 SCR 311 ............................................................................................ 78
Meaning of Arrest ..................................................................................................................... 80
R. v. Latimer, [1997] 1 SCR 217 .......................................................................................... 80
Appearance Notice .................................................................................................................... 82
s. 496 – Criminal Code ......................................................................................................... 82
s. 497 – Criminal Code ......................................................................................................... 82
s. 498 – Criminal Code ......................................................................................................... 83
s. 499 – Criminal Code ......................................................................................................... 84
Interrogation – Right to Counsel and Arrest Informational duties ............................................... 85
Right to counsel (Charter s. 10(b)) ........................................................................................... 85
s. 2(c) – Canadian Bill of Rights ........................................................................................... 85
s. 10 – Canadian Charter of Rights and Freedoms................................................................ 86
1. Triggering mechanisms ......................................................................................................... 86
Brownridge v. R. (1972)........................................................................................................ 86
Chromiak v. R (1980 ............................................................................................................. 86
R. v. Therens, [1985] 1 SCR 613 .......................................................................................... 86
Thomsen v. R, [1988] 1 SCR 640.......................................................................................... 88
R. v. Moran (1987), 36 CCC (3d) 225 .................................................................................. 89
R. v. Mickey (1988), 46 CCC (3d) 278 ................................................................................. 90
R. v. Hawkins (1992), 14 CR (4th) 286.................................................................................. 91
R. v. Feeney supra, [1997] 2 SCR 13 ................................................................................... 92
R. v. Bartle, [1994] 3 SCR 173 ............................................................................................. 93
2. Informational duties .......................................................................................................... 93
R. v. Brydges, [1990] 1 SCR 190 .......................................................................................... 94
R. v. Bartle (1994), 33 CR (4th) 1.......................................................................................... 95
2. Implementation duties ........................................................................................................... 96
Jumaga v. R (1977) ............................................................................................................... 97
R. v. Playford (1987) ............................................................................................................. 97
R. v. Manninen, [1987] 1 SCR 1233 ..................................................................................... 97
Baig v. R., [1987] 2 SCR 537................................................................................................ 98
Leclair and Ross v. R., [1989] 1 SCR 3 ................................................................................ 99
5
R. v. Burlingham, [1995] 2 SCR 206 .................................................................................. 101
R. v. Prosper (1994) ............................................................................................................ 102
3. Waiver and duty to be reasonably diligent in exercise of right .......................................... 103
R. v. Clarkson, [1986] 1 SCR 383 ...................................................................................... 103
R. v. Smith, [1989] 2 SCR 368 ............................................................................................ 104
Right to Silence ....................................................................................................................... 107
Confession rule ....................................................................................................................... 107
What is the Crown requested to show under the voluntary confession rule? ......................... 107
s. 7 – Canadian Charter of Rights and Freedoms................................................................ 108
R. v. Hebert, [1990] 2 SCR 151 .......................................................................................... 108
R. v. Singh, [2007] SCC 48 ................................................................................................. 112
Bail Hearings .............................................................................................................................. 114
The rational for the Bail Reform Act of 1961 ......................................................................... 115
Intake Procedures ................................................................................................................ 115
s. 504 – Criminal Code ....................................................................................................... 116
s. 505 – Criminal Code ....................................................................................................... 116
s. 506 – Criminal Code ....................................................................................................... 117
s. 507 – Criminal Code ....................................................................................................... 117
s. 507.1 – Criminal Code .................................................................................................... 118
s. 508 – Criminal Code ....................................................................................................... 119
s. 515 – Criminal Code ....................................................................................................... 120
s. 516 – Criminal Code ....................................................................................................... 125
s. 517 – Criminal Code ....................................................................................................... 126
s. 518 – Criminal Code ....................................................................................................... 126
s. 519 – Criminal Code ....................................................................................................... 127
s. 520 – Criminal Code ....................................................................................................... 128
s. 521 – Criminal Code ....................................................................................................... 129
s. 522 – Criminal Code ....................................................................................................... 131
s. 795 – Criminal Code ....................................................................................................... 132
Show cause hearings ............................................................................................................... 132
R. v. Thompson (1972), 18 CRNS 102 (BC SC)................................................................. 132
Grounds for Detention ............................................................................................................ 133
Re Powers and R. (1973), 20 CRNS 23 (Ont. HC)............................................................. 133
R. v. Graham, Ont. Dist. Ct., August 2 1990 ...................................................................... 133
6
R. v. Lamothe (1990), 77 CR (3d) 236 ................................................................................ 134
s. 11(e) – Canadian Charter of Rights and Freedoms ......................................................... 135
R. v. Morales, [1992] 3 SCR 711 ........................................................................................ 135
R. v. Hall (2002), 4 CR (6th) 197 ........................................................................................ 138
3. The Judge Must Decide ...................................................................................................... 140
R. v. Major (1990), 76 CR (3d) 104 (Ont. Dist. Ct.)........................................................... 141
4. Reasonable Conditions on Release ..................................................................................... 141
Re Keenan and R., 1979 ...................................................................................................... 141
R. v. Anoussis (2008) .......................................................................................................... 141
5. Charter Standards ................................................................................................................ 142
R. v. Pearson, [1992] 3 SCR 665 ........................................................................................ 142
R. v. Cantave, Court o f Quebec Criminal Chamber – September 9 2008 ......................... 143
Disclosure and Discovery ........................................................................................................... 143
Distinction between disclosure and production ...................................................................... 144
s. 625.1 – Criminal Code .................................................................................................... 144
Royal Commission on the Donald Marshall Jr. Prosecution .............................................. 144
R. v. Stinchcombe, [1991] 3 SCR 326................................................................................. 145
Production of Third Party Records ......................................................................................... 146
L.L.A. v. A.B ........................................................................................................................ 147
R. v. O’Connor, [1995] 4 SCR 411..................................................................................... 147
Lost evidence cases ................................................................................................................. 148
R.v. Carosella (1997) .......................................................................................................... 148
R. v. La; R. v. Vu ................................................................................................................. 148
Bill C-46.................................................................................................................................. 148
R. v. Mills, [1999] 3 SCR 668 ............................................................................................. 148
Preliminary Inquiry ..................................................................................................................... 151
s. 535 – Criminal Code ....................................................................................................... 151
Preferring the Indictment ........................................................................................................ 151
s. 566 – Criminal Code ....................................................................................................... 152
s. 574 – Criminal Code ....................................................................................................... 152
s. 577 – Criminal Code ....................................................................................................... 153
Preliminary inquiry as discovery vehicle ................................................................................ 153
Skogman v. R, [1984] 2 SCR 93 ......................................................................................... 153
Test for committal: sufficiency of evidence ............................................................................... 154
7
s. 577 – Criminal Code ....................................................................................................... 154
R. v. Nelles (1982), 16 CCC (3d) 97 (Ont. Prov. Ct) .......................................................... 155
R. v. Arcuri, [2001] 2 SCR 828........................................................................................... 155
United States v. Ferras, [2006] 2 SCR 77 .......................................................................... 156
R. v. Hynes .......................................................................................................................... 158
Constitutional Remedies ............................................................................................................. 158
s. 24 – Canadian Charter of Rights and Freedoms.............................................................. 159
R. v. Big M Drug Mart Ltd. ................................................................................................. 159
s. 52(1) Constitutional Act, 1982 ........................................................................................ 159
R. v. Therens ....................................................................................................................... 159
R. v. Buhay .......................................................................................................................... 160
Exclusion of evidence ............................................................................................................. 160
A.G. Que. V. Begin .............................................................................................................. 160
R. v. Collins supra, [1987] 1 SCR 265 ............................................................................... 160
R. v. Stillman supra, [1997] 1 SCR 607.............................................................................. 163
R. v. Feeney supra, [1997] 2 SCR 13 ................................................................................. 166
R. v. Grant ........................................................................................................................... 168
R. v. Buhay .......................................................................................................................... 168
Judicial stay as abuse of process ............................................................................................. 168
Rourke v. R., [1978] 1 SCR 1021........................................................................................ 168
R. v. Jewitt, [1985] 2 SCR 128............................................................................................ 169
R. v. Keyowski, [1988] 1 SCR 657 ...................................................................................... 171
R. v. O’Connor, [1995] 4 SCR 411..................................................................................... 172
R. v. L.(W.K), [1991] 1 SCR 1091 ...................................................................................... 174
R. v. Neil (2002) .................................................................................................................. 175
Pleas and Plea Bargaining ........................................................................................................... 175
s. 606 – Criminal Code ....................................................................................................... 175
R. v. Lessard (1976), 33 CRNS 16 (Ont. CA) .................................................................... 176
Adgey v. R. (1973), 23 CRNS 298 (SCC) ........................................................................... 177
Plea Bargaining ....................................................................................................................... 180
Klein, Let’s Make a Deal .................................................................................................... 180
Ericson, Making Crime: A Study of Detective Work ......................................................... 181
Ferguson and Roberts, Plea Bargaining: Directions for Canadian Reform ........................ 181
Ministry of Attorney General for Ontario Crown Policy Manual (2005) ........................... 181
8
Kipnis, Criminal Justice and the Negotiated Plea ............................................................... 182
R. v. D.(E.) (1990), 73 OR (2d) 758 ................................................................................... 182
Law Reform Commission of Canada. (1975). Working paper no. 15: Criminal procedure:
Control of the process ......................................................................................................... 183
The Trial Process: Preliminary Considerations .......................................................................... 185
Right to counsel at trial (Charter s. 7) .................................................................................... 185
Barrette v. R., [1977] 2 SCR 121 ........................................................................................ 186
Deutsch v. Law Society of Upper Canada (1986), 47 CR (3d) 166.................................... 187
R. v. Peterman (2004), 19 CR (6th) 258 .............................................................................. 188
R. v. B.(G.D) ....................................................................................................................... 190
R. v. Tran (2001) and R. v. Moghaddam (2006) ................................................................. 191
Adjournments .......................................................................................................................... 191
R. v. G. (J.C) (2004), 189 CCC (3d) 1 ................................................................................ 191
Trial within a reasonable time (Charter s. 11(b)) .................................................................... 193
s. 11(b) Canadian Charter of Rights and Freedoms ............................................................ 193
R. v. Akov, [1990] 2 SCR 1199 ........................................................................................... 194
R. v. Morin, [1992] 2 SCR 771 ........................................................................................... 196
Formal attacks on the information or indictment .................................................................... 198
s. 535 – Criminal Code ....................................................................................................... 198
s. 566 – Criminal Code ....................................................................................................... 199
Three reasons to challenge the indictment .............................................................................. 199
A) Grounds.............................................................................................................................. 200
1. Insufficiency ....................................................................................................................... 200
s. 581 – Criminal Code ....................................................................................................... 200
s. 583 – Criminal Code ....................................................................................................... 201
s.795 – Criminal Code ........................................................................................................ 201
s. 601(1) – Criminal Code ................................................................................................... 201
Brodie v. R., [1936] SCR 188 ............................................................................................. 201
R. v. McKenzie, [1972] 2 SCR 409 ..................................................................................... 202
R. v. Coté, [1978] 1 SCR 8.................................................................................................. 202
R. v. Wis Development Corporation Ltd. et al, [1984] 1 SCR 485..................................... 203
R. v. B. (G.), [1990] 2 SCR 30 ............................................................................................ 204
2. Duplicity ............................................................................................................................. 205
s. 581(1) – Criminal Code ................................................................................................... 205
s. 590(1) – Criminal Code ................................................................................................... 205
9
s. 789(1) – Criminal Code ................................................................................................... 205
s. 581 – Criminal Code ....................................................................................................... 206
R. v.Sault Ste. Marie , [1978] 2 SCR 1299 ......................................................................... 206
3. Improper joinders of Counts ............................................................................................... 206
s. 590(3) – Criminal Code ................................................................................................... 207
s. 591(3) – Criminal Code ................................................................................................... 207
R. v. Racco (No. 1) (1975), 29 CRNS 303 (Ont. Co. Ct) .................................................... 207
B) Saving devices ................................................................................................................... 208
s. 587 – Criminal Code ....................................................................................................... 208
Thatcher v. R. (1984), 42 CR (3d) 259 ............................................................................... 209
s. 601 – Criminal Code ....................................................................................................... 209
R. v. Moore, [1988] 1 SCR 1097 ........................................................................................ 212
R. v. Tremblay, [1993] 2 SCR 932...................................................................................... 212
3. On appeal ............................................................................................................................ 213
The Trial Process: Jury Trials ..................................................................................................... 213
R. v. Bryant (1984), 48 OR (2d) 732................................................................................... 214
s. 11(f) – Canadian Charter of Rights and Freedoms.......................................................... 214
s. 631(5) – Criminal Code ................................................................................................... 215
s. 631(5) – Criminal Code ................................................................................................... 215
R. v. Church of Sientology .................................................................................................. 215
Challenge to array ................................................................................................................... 215
s. 629 – Criminal Code ....................................................................................................... 215
s. 630 – Criminal Code ....................................................................................................... 216
R. v. Born With a Tooth (1993), 22 CR (4th) 232 (Alta. Q.B)............................................. 216
R. v. F.A. (1984) .................................................................................................................. 217
R. v. Born With a Tooth (1993), 22 CR (4th) 232 (Alta. Q.B)............................................. 217
Peremptory challenges ............................................................................................................ 218
s. 632 – Criminal Code ....................................................................................................... 218
s. 633 – Criminal Code ....................................................................................................... 219
s. 634 – Criminal Code ....................................................................................................... 219
s. 635 – Criminal Code ....................................................................................................... 220
Challenges for cause ............................................................................................................... 220
s. 635 – Criminal Code ....................................................................................................... 220
The process of challenges ....................................................................................................... 221
10
R. v. Crosby (1979), 49 CCC (2d) 255 ............................................................................... 221
R. v. Sherratt, [1991] 1 SCR 509 ........................................................................................ 221
R. v. Williams, [1998] 1 SCR 1128 ..................................................................................... 222
The Trial Process – Further Considerations................................................................................ 224
Jerome Frank, Courts on Trial: Myth and Reality in American Justice ............................. 224
Res Judicata ............................................................................................................................ 226
s. 607 – Criminal Code ....................................................................................................... 226
1. Double jeopardy .................................................................................................................. 226
s. 11(h) – Canadian Charter of Rights and Freedoms ......................................................... 227
R. v. Riddle, [1980] 1 SCR 380 ........................................................................................... 227
When are matters considered identical? ................................................................................. 228
s. 609 – Criminal Code ....................................................................................................... 228
R. v. Wigglesworth, [1988] 1 SCR 541 ............................................................................... 229
2. Multiple punishment ........................................................................................................... 230
R. v. Kinnear (2005), 30 CR (6th) 1 (Ont. CA) ................................................................... 230
11
Jurisdiction
Sources of criminal procedure
 Constitution
 Statutes
o Not limited to the Criminal Code; Competition Act, Tax Act
 Common law/judge-made law
o Procedural defences created by the courts
 Rules of Court
o S.486 of the CC gives allows the courts to create their own rules
o They cannot be inconsistent with statutes but they are there
Legislative division of powers
s. 91 - Constitution Act, 1867
It shall be lawful for the Queen, by and with the Advice and Consent of the Senate and House
of Commons, to make Laws for the Peace, Order, and good Government of Canada, in relation
to all Matters not coming within the Classes of Subjects by this Act assigned exclusively to the
Legislatures of the Provinces; and for greater Certainty, but not so as to restrict the Generality
of the foregoing Terms of this Section, it is hereby declared that (notwithstanding anything in
this Act) the exclusive Legislative Authority of the Parliament of Canada extends to all Matters
coming within the Classes of Subjects next hereinafter enumerated; that is to say,
...
27. The Criminal Law, except the Constitution of Courts of Criminal Jurisdiction, but including
the Procedure in Criminal Matters.
28. The Establishment, Maintenance, and Management of Penitentiaries.
s. 92 - Constitution Act, 1867
In each Province the Legislature may exclusively make Laws in relation to Matters coming
within the Classes of Subjects next hereinafter enumerated; that is to say,
...
6. The Establishment, Maintenance, and Management of Public and Reformatory Prisons in and
for the Province.
...
13. Property and Civil Rights in the Province.
14. The Administration of Justice in the Province, including the Constitution, Maintenance, and
Organization of Provincial Courts, both of Civil and of Criminal Jurisdiction, and including
Procedure in Civil Matters in those Courts.
15. The Imposition of Punishment by Fine, Penalty, or Imprisonment for enforcing any Law of
the Province made in relation to any Matter coming within any of the Classes of Subjects
enumerated in this Section.
12
Legislative jurisdiction over criminal law and procedure is given to the Federal Parliament under
s.91 of the Constitution Act
 But provinces can enact penal legislation for enforcement for matters that would
otherwise fall under provincial jurisdiction, provided that they do not infringe on
Parliament’s exclusive jurisdiction
The federal Parliament has enacted the procedure to be followed for the prosecution of all crimes
enacted by the Criminal Code and federal legislation
 Provincial provisions vary, yet they mostly mirror federal ones
The interpretation section of the Criminal Code (s.2) set out for each of the provinces the
meaning of the “court or appeal”, “court of criminal jurisdiction”, “magistrate”, “territorial
division”, and “superior court of criminal jurisdiction”
The prosecution of federal matters will depend on whether the matter concerns the Criminal
Code or other federal statutes
 Look at the interpretation section of the Criminal Code (s.2) in relation to the definitions
of “prosecutor” and “Attorney General”
 The enforcement of the Code will normally be given to the Attorney General of the
provinces, while the enforcement of other general legislation is given to the Attorney
General of Canada
 R. v. Trimarchi (1987) determined that a provincial judge is entitled to try offences
created by the Code and other federal legislation
 In A.G. Canada v. C.N. Transportation (1983), the court held that whether it was criminal
legislation or not, the federal government could authorize who would prosecute it
Classification of offences
At common law a distinction was drawn between indictable offences triable only by judge and
jury and offences triable only summarily by justices of the peace sitting alone
In Canada there are, broadly speaking, four types of offences
1. Indictable conviction
2. Indictable summary conviction
3. Dual (hybrid) offences
4. Contraventions (regulatory) offences under the Contraventions Act
The wording of the criminal offence states the type of offence
Indictable offences
 There are the more serious offences
 Usually the offences, or its penalty section, set out the punishment applicable to the
offence
o Otherwise, s.743 of the Code provides a maximum of 5 years
13


Where the accused is charged with an indictable offence, she must in general be present
at all stages of proceedings (except for the exceptions provided in 650 – 650.02 of the
Code)
There are mainly three types of indictable offences, and their classification affects their
forum of trial and jurisdiction
o The most serious are given to the exclusive jurisdiction of the superior court of
criminal jurisdiction (see ss. 468 and 469)
o The least serious are absolutely within the jurisdiction of a provincial court (s.
553)
o For the great bulk of offences, the accused is entitled to choose her model of trial
(s. 536(2))
 If the accused does not elect a mode of trial, she would have been deemed
to have elected trial by judge and jury, s. 565(1)(c)
A note on jurisdiction
 Every superior court of criminal jurisdiction has jurisdiction to try any indictable offence
as per s. 468 of the Code
o This does not imply that every superior court will do so, however, see s. 469
 Other courts of criminal jurisdiction are allowed to try anything except for
the offences described there
o This refers to the exclusive jurisdiction of the superior court
 Provincial court judges are said to have absolute jurisdiction because they are absolutely
entitled to try s. 553 offences without being dependent on the accused’s choice
 As for the rest of offences, the jurisdiction lies on the superior court except for Quebec
o See s. 552 and the definition of judge
o Here the accused will have the choice of jurisdiction
 Trial by jury, trial by judge alone or trial in provincial jurisdiction
s. 468 – Criminal Code
Superior court of criminal jurisdiction
Every superior court of criminal jurisdiction has jurisdiction to try any indictable offence.
s. 469 – Criminal Code
Court of criminal jurisdiction
Every court of criminal jurisdiction has jurisdiction to try an indictable offence other than
(a) an offence under any of the following sections:
(i) section 47 (treason),
(ii) section 49 (alarming Her Majesty),
(iii) section 51 (intimidating Parliament or a legislature),
(iv) section 53 (inciting to mutiny),
(v) section 61 (seditious offences),
(vi) section 74 (piracy),
(vii) section 75 (piratical acts), or
14
(viii) section 235 (murder);
Accessories
(b) the offence of being an accessory after the fact to high treason or treason or murder;
(c) an offence under section 119 (bribery) by the holder of a judicial office;
Crimes against humanity
(c.1) an offence under any of sections 4 to 7 of the Crimes Against Humanity and War Crimes
Act;
Attempts
(d) the offence of attempting to commit any offence mentioned in subparagraphs (a)(i) to (vii);
or
Conspiracy
(e) the offence of conspiring to commit any offence mentioned in paragraph (a).
PART XIX: INDICTABLE OFFENCES — TRIAL WITHOUT JURY
s. 552 – Criminal Code
In this Part,
"judge"
«juge »
"judge" means,
(a) in the Province of Ontario, a judge of the superior court of criminal jurisdiction of
the Province,
(b) in the Province of Quebec, a judge of the Court of Quebec,
(c) in the Province of Nova Scotia, a judge of the superior court of criminal jurisdiction
of the Province,
(d) in the Province of New Brunswick, a judge of the Court of Queen’s Bench,
(e) in the Province of British Columbia, the Chief Justice or a puisne judge of the
Supreme Court,
(f) in the Provinces of Prince Edward Island and Newfoundland, a judge of the Supreme
Court,
(g) in the Province of Manitoba, the Chief Justice or a puisne judge of the Court of
Queen’s Bench,
(h) in the Provinces of Saskatchewan and Alberta, a judge of the superior court of
criminal jurisdiction of the province,
(i) in Yukon and the Northwest Territories, a judge of the Supreme Court, and
15

(j) in Nunavut, a judge of the Nunavut Court of Justice.
Quebec appears to be the exception, since all other courts mentioned are federal
s. 553 – Criminal Code
Absolute jurisdiction
The jurisdiction of a provincial court judge, or in Nunavut, of a judge of the Nunavut Court of
Justice, to try an accused is absolute and does not depend on the consent of the accused where
the accused is charged in an information
(a) with
(i) theft, other than theft of cattle,
(ii) obtaining money or property by false pretences,
(iii) unlawfully having in his possession any property or thing or any proceeds of
any property or thing knowing that all or a part of the property or thing or of the
proceeds was obtained by or derived directly or indirectly from the commission
in Canada of an offence punishable by indictment or an act or omission
anywhere that, if it had occurred in Canada, would have constituted an offence
punishable by indictment,
(iv) having, by deceit, falsehood or other fraudulent means, defrauded the public
or any person, whether ascertained or not, of any property, money or valuable
security, or
(v) mischief under subsection 430(4),
where the subject-matter of the offence is not a testamentary instrument and the alleged value of
the subject-matter of the offence does not exceed five thousand dollars;
(b) with counselling or with a conspiracy or attempt to commit or with being an
accessory after the fact to the commission of
(i) any offence referred to in paragraph (a) in respect of the subject-matter and
value thereof referred to in that paragraph, or
(ii) any offence referred to in paragraph (c); or
(c) with an offence under
(i) section 201 (keeping gaming or betting house),
(ii) section 202 (betting, pool-selling, book-making, etc.),
(iii) section 203 (placing bets),
(iv) section 206 (lotteries and games of chance),
(v) section 209 (cheating at play),
(vi) section 210 (keeping common bawdy-house),
(vii) [Repealed, 2000, c. 25, s. 4]
(viii) section 393 (fraud in relation to fares),
(viii.1) section 811 (breach of recognizance),
(ix) subsection 733.1(1) (failure to comply with probation order),
(x) paragraph 4(4)(a) of the Controlled Drugs and Substances Act, or
(xi) subsection 5(4) of the Controlled Drugs and Substances Act.
16
Summary conviction offences
 These convictions are set in Part XXVII of the Criminal Code
 Summary offences are time barred by 6 months
 The maximum penalty is 2,000 dollars or six month imprisonment or both (s.787)
o In theory, Parliament could establish a penalty of five years less one day without
violating the right to a jury trial under s. 11(f) of the Charter
 Some summary conviction can have convictions to imprisonment longer than 6-months
o See s.271.1
 Parliament may want to give a higher punishment, yet allow more
flexibility to the prosecution in avoiding a preliminary inquiry
 These can be called super-summaries
o There are some examples of offences involving summary conviction which have
higher fines
 Summary convictions are always tried in Provincial Court or by justice of the peace
(where applicable)
o While superior court judges have the jurisdiction to try these, they rarely exercise
it
 Provincial offences are all summary offences
 What is known as the indictment for indictable offences is called “an information” for
summary convictions
 Accused individuals charged with a summary offence do not normally need to appear in
court, but may instead chose to have a lawyer or some other agent present, unless the
judge requires otherwise (s. 800(2))
 Summary convictions are appealed to a summary conviction appeal court with a judge of
the superior court seating alone
o Indictable offences are appealed at the court of appeal of the province
Crown elections offences (dual, hybrid)
 The prosecutor can decide whether to prosecute as a summary conviction or indictable
offence
o This should be indicated prior to trial
 An example of this types of offences is Fraud (s. 380)
 If the choice is to proceed by indictment, the forum of the trial will depend on the type of
indictable offence involved
 Various considerations are taken by the Crown in the election
o Higher penalty of indictable offences
o Prior convictions of the accused
o The desire to require the accused’s personal presence
o Wishing to get the matter over more quickly
 In 1994, Parliament increased the maximum penalty for several hybrid offences
proceeded against by way of summary conviction to 18 months
 If the Crown chooses summary proceedings, the accused no longer has the choice of a
preliminary inquiry under s. 536
 S. 34(1) of the Interpretation Act (see below) has been applied to equate hybrid offences
with indictable offences in contexts such as arrest and fingerprinting
17
s. 34(1) - Interpretation Act
Where an enactment creates an offence,
(a) the offence is deemed to be an indictable offence if the enactment provides that the
offender may be prosecuted for the offence by indictment;
...
Even in looking at the powers of arrest without warrant we see the importance of the
classification of offence
 S.495
o Officers can arrest without warrant people who have committed indictable
offences, or any person fleeing from any type of offence
 The classification also applies to judicial interim release
Time limitation in Canadian law
Summary convictions are barred by 6-months of the commission of the offence (s. 786(2))
s. 786(2)– Criminal Code
Limitation
No proceedings shall be instituted more than six months after the time when the subject-matter
of the proceedings arose, unless the prosecutor and the defendant so agree.
For the case of indictable offences there is only time limitation, which is a 3-year bar to the
crime of treason (see s. 48(1))
Provincial offences usually specify their own time limitation
Territorial limitations
In the time of jury trial it was natural that criminal trial should be conducted at the place where
the criminal offence occurred
 While the jury system has changed, there is still a right for a community to see firsthand
that justice is made with respect to matters that concern it
The general common principle can be seen today in s. 6(2) of the Code
s. 6(2)– Criminal Code
Offences outside Canada
Subject to this Act or any other Act of Parliament, no person shall be convicted or discharged
under section 730 of an offence committed outside Canada.
18
There are certain statutory exceptions to this principle
 Certain offences on aircrafts (s. 7(1))
 Certain offences against internationally protected protected person pr property (s. 7(3))
 Hostage taking (s. 7(3.1)
 Offences involving nuclear material (s. 7(3.2))
 War crimes (s. 7(3.71))
 Terrorist offences (ss. 7(3.73) – 7(3.75))
 Child sex tourism (s. 7(4.1))
 Treason (s. 46(3))
 Others include forging a passport, using a citizenship certificate fraudulently, priacy,
bigamy, and conspiracy
R. v. Greco (2001), 159 CCC (3d) 146 (Ont. CA)
Facts
 Appeal by the accused from his conviction for breach of probation
 Greco and a female companion named Smith travelled to Cuba for a holiday,during
which he viciously assaulted her
 The Cuban authorities refused to become involved
 The police in Toronto would not charge Greco for the assault
 It was subsequently discovered that, at the time of the holiday, Greco was subject to a
probation order after he was convicted of assault and threatening death
o The order required him to keep the peace and be of good behaviour. Greco tried
to have the charge stayed
 . He relied upon section 6(2) of the Criminal CodeAt issue was whether Greco was. The
second question was whether
Issues:
1. Was the accused obligated to comply with the probation order outside of Canada?
2. Does the Ontario court have jurisdiction to try him for this offence.
Holding: 1. Yes, 2. Yes; appeal dismissed
Moldaver JA
1.
Territorial limitations applied to the ability of the court to enforce its orders
 However, such limitations did not affect the jurisdiction of the court to make
orders that applied outside of Canada.
 Various provisions existed in the Code that asserted such jurisdiction
Probation orders had to apply outside of Canada for policy reasons
 The probation order did not require an express provision for it to apply outside of
Canada
2.
The court had jurisdiction to try and convict Greco
19
There was a real and substantial connection between the offence and this country
 The connection was that Canada had an interest to ensure that its court orders were
complied with
 Section 6(2) did not apply to this situation.
Re Bigelow and R (1982), 37 OR (2d) 304
Facts
 The appellant was charged with unlawful detention of his infant son
 The appellant and his wife were separated, with custody of their infant son given to the
wife with access to the appellant every second weekend
o The wife and son resided in Ontario
 On a weekend allotted to him, the appellant removed the son to Alberta, whereupon the
police were required to intervene to return the son to the wife
 The appellant was committed for trial in Ontario, where he brought a motion for
certiorari to quash for lack of jurisdiction
 The trial judge dismissed the application, whereupon the appellant appealed.
Issue: Could the accused be charged in Ontario for having removed his son to another province?
Holding: Yes; appeal dismissed
The Court
In considering the question of jurisdiction over inter-provincial offences, Ontario courts
would have no jurisdiction if an offence were WHOLLY committed in another province
However, Ontario courts would have jurisdiction over an offence if any element could be
said to have been committed in Ontario
 This required an analysis of the acts of an accused on a flexible basis and included
three categories: first, a continuity of operation extending from Ontario to the other
province; second, the commission of an overt act in Ontario; or, third, the registration of
effects in Ontario from acts committed in other provinces
Ramifications of the offence occurred in Ontario, and the execution of the offence, including
the obtaining of possession of the child and boarding of the plane, took place in Ontario,
together with a refusal to redeliver the child to Ontario
 The offence of detention is a "withholding", which was beyond the mere confinement in
Alberta
Jurisdiction under the Charter
s. 24 Canadian Charter of Rights and Freedoms
(1) Anyone whose rights or freedoms, as guaranteed by this Charter, have been infringed or
denied may apply to a court of competent jurisdiction to obtain such remedy as the court
considers appropriate and just in the circumstances.
(2) Where, in proceedings under subsection (1), a court concludes that evidence was obtained in
20
a manner that infringed or denied any rights or freedoms guaranteed by this Charter, the
evidence shall be excluded if it is established that, having regard to all the circumstances, the
admission of it in the proceedings would bring the administration of justice into disrepute.
Charter remedies under s.24 can be given by a court of competent jurisdiction
 In R. v. Mills (1986) the SCC held that all criminal trial courts as defined by s. 2 of the
Code were courts of competent jurisdiction (this includes provincial and federal courts)
o However, a justice at a preliminary inquiry would not be a court of competent
jurisdiction under s.24 of the Charter
o The SCC also determined that s.24 of the Charter does not confer any
jurisdiction in the form of a any new rights of appeal
 This position is no longer clear
The SCC has not directly pronounced on the issue of standing of complainants at trial to raise
Charter rights, but it has conferred such jurisdiction in some of its rulings
As for s. 52 of the Charter
 In R. v. Big M Drug Mart (1985), the SCC held that s. 52 has a remedial role in
criminal law independent of s. 24
 An accused may defend a charge arguing that the law under which the charge has been
brought was constitutionally invalid
 The Charter normally has no remedy when the conduct of officials in question occurred
outside of Canada
s. 52(1) Constitutional Act, 1982
The Constitution of Canada is the supreme law of Canada, and any law that is inconsistent with
the provisions of the Constitution is, to the extent of the inconsistency, of no force or effect.
Timeline of Criminal Proceedings




Charge – information containing name and offence. If judge is satisfied then complaint
can be laid. It’s the decision of the judge, certainly not of complainant.
o Summons goes out. You’ve been charged, show up at this court at this time, if
you don’t you’re guilty of another offence
o Arrest – not everyone arrested will be charged for the thing they were arrested,
must be brought before judge within 24 hours.
o Once someone is charged, there must be a judicial decision formalizing the charge
of the accused. It’s the judge who decides, not crown
Disclosure
Judicial interim release (bail)
Preliminary Inquiry used to be to test whether there was sufficient evidence to go to trial
and also to give defence chance to test crown witnesses and get testimony on record
o The accused used to be entitled to it, but now you have to ask for it, specify which
witnesses you want to hear and what you want to hear from them (practically this
is never refused but technically it could be)
21






Pre-trial motions (e.g. based on the Charter)
Preferred indictment (can supersede preliminary inquiry)
o I.e. gives crown right to take away preliminary inquiry
o E.g. Paul Bernardo case, allows crown to spare family from testifying twice. It’s
entirely at the discretion of the crown.
Trial
o “Information” is a sworn declaration
o If s.469 offence, or other offence in the middle, then document at trial is called an
indictment
Verdict
Sentence
Appeal
o Part 21 of the Code for indictable offences
Police Stopping Powers
Some of the issues which are salient when looking at police stopping powers
 The rule of law
 Limits if state authority vis-s-vis the Charter
 Treatment of aboriginals by the criminal justice system
The state of affairs before the Charter in relation to stopping powers
 Police were entitled to detain people for the purposes of investigation
 There are some limits, however, on police power
o For long they have not be allowed to randomly stand and ask people for
identification
o Deadman is a good example of the state of the law before the Charter
o Dickson J (dissenting) states what the main state of the law was then
 While the police was assumed to have stopping powers to question an
individual, she would not be obliged to answer question
 If the police were to stop someone because of failure to cooperate they
would need reasonable belief that the person had committed an offence or
was about to commit an offence
 Absent of statutory or clear common law provision, the powers of the
police
s. 9 – Canadian Charter of Rights and Freedoms
Detention or imprisonment
Everyone has the right not to be arbitrarily detained or imprisoned.
s. 10 – Canadian Charter of Rights and Freedoms
22
Arrest or detention
Everyone has the right on arrest or detention
a) to be informed promptly of the reasons therefor;
b) to retain and instruct counsel without delay and to be informed of that right; and
c) to have the validity of the detention determined by way of habeas corpus and to be
released if the detention is not lawful.
R. v. Deadman (1985), 46 CR (3d) 193
Facts
 The accused was randomly stopped as part of a spot check program, whose principal aim
is to detect, deter and reduce impaired driving
 He was accused with failing, without reasonable excuse to provide a breath sample, but
was acquitted
 On appeal, the acquittal was confirmed as it was held that there was neither statutory nor
common law authority to require him to stop his motor vehicle, so he had a reasonable
excuse
 The Ontario CA reversed the acquittal
Issue: Did the police officer possess authority, either statutory or at common law, to require the
appellant to stop his motor vehicle?
Holding: Yes; appeal dismissed
Le Dain J
The test laid down in R. v. Waterfield, while generally invoked in cases where it is at issues
whether a police officer had acted in the execution of her duties, has been recognized as a test for
whether the officer had common law authority to do what she did
 Applying the Waterfield test, the random vehicle stop was a prima facie unlawful
interference with liberty since it was not authorized by statute
The right to circulate in a motor vehicle on the public highway may be described as a liberty;
however, it cannot be regarded as a fundamental liberty like an individual's right of
movement, since it is a licensed activity subject to regulation and control for the protection
of life and property
The random stop falls within the general scope of police duties to prevent crime and to
protect life and property by the control of traffic as these are the very objects of the
program, a measure intended to improve the deterrence and detection of impaired driving
 The stop was not an unjustifiable use of police power because it was both necessary
to the execution of police duty and reasonable, having regard to the nature of the
liberty interfered with and the importance of the public purpose served by the
interference
Dickson CJ (dissenting)
It has always been a fundamental tenet of the rule of law that police, in carrying out their general
duties as law enforcement officers have limited powers and are only entitled to interfere with the
23
liberty or property of the citizen to the extent authorized by law
The fact that a police officer has a general duty to prevent crime and protect life and property
does not mean that he or she can use any or all means for achieving these ends.
In the criminal law, the rules and principles relating to arrest establish justifiable limits upon a
citizen's liberty
 Short of arrest, the police have never possessed legal authority at common law to
detain any one against his or her will for questioning or to pursue an investigation
 These random stops are indistinguishable from detention for questioning or
investigation whether or not they might be committing a criminal offence and,
without validly enacted legislation to support them, are unlawful
 It would be contrary to the long standing protection accorded individual liberty by the
common law and detrimental to the individual's fundamental right to be free from
arbitrary interference to conclude that this action of the police was authorized and lawful
The fact that driving a motor vehicle is a licensed activity subject to regulation and control in the
interests of safety is irrelevant to police power if the conditions for licensing have been met and
are adhered to
Lecture Notes
 This case establishes the new police power of investigative detention
 The Waterfield test is the source of the doctrine of ANCILLARY POLICE POWERS
o It came about, however, in trying to determine whether a police officer was acting
within her legally prescribed authority
 The general duties of police officers, were a common law authority that would justify this
type of stopping
 If the powers of the police officer are prima facie unlawful, you can check whether
the actions fall within the general powers of the police, and whether they are
justifiable
o The stops were justifiable given the importance of highway traffic and the fact
that the activity was a licensed one
 A crucial point about this case, was whether the creation of these powers was a matter of
the legislature or courts could do it as well
 Since then, many provinces have enacted statutory authorizations for random road stops
Stop Powers and Racial Profiling
1. Vehicle Stops
In R. v. Hufsky (1988) and R. v. Ladouceur (1990) the SCC held that detention in the
unfettered discretion of a police officer is necessarily arbitrary and contrary to s.9 of the
Charter
 At issue in both cases, was the constitutionality of the spot check procedure of s.216(1) of
the Ontario Highway Safety Act
24


However, Le Dain J states that the limit on this Charter rights was justified under the
s.1 test by the importance of highway safety, the gravity of motor vehicle accidents, and
the role played by random stopping in helping detect dangerous drivers which may
otherwise not be detected by mere observation of driving
Hufsky dealt with a situation where the random sport check was in a fixed location, while
in Ladouceur the random stop was roving
According to the Report of the Commission on Systemic Racism in the Ontario Criminal Justice
System (1995), black men in the Toronto metropolitan are particularly vulnerable to being
stopped by the police
In the SCC case of R. v. Mellenthin (1992), Cory J decided that while random stops were
justifiable in order to reduce mort vehicle accidents, and as such could include licence,
sobriety and ownership tests, the scope of the check could not be extended beyond these
aims
 A man who was found to be in possession of hash oil through a random vehicle check
was acquitted as the evidence related to this offence was excluded
 Random blocks could not be used to go on fishing expeditions unrelated to driving
In R. v. Orbanski (2005), the SCC held that implied in the operation requirements of drunk
driving legislation were police powers to ask motorists whether they had anything to drink and
administer sobriety tests
2. Investigative Detention
R. v. Simpson (1993), 20 CR (4th) 1
Facts
 Acting on information that a particular house was suspected to be a "crack house", a
police officer patrolled the area and observed a woman leaving the place with the accused
 The police officer, who had no information pertaining to either person, followed them
and stopped the vehicle
 The police officer noticed a bulge in the accused's front pant pocket, he touched it and
felt a hard lump
o At that point the officer did not have reasonable grounds to arrest the accused
o He asked the accused to remove the object, which turned out to be a baggie
containing cocaine
 The accused was charged with possession of cocaine for the purpose of trafficking
 The trial judge rejected the accused's argument that his rights under ss. 9 and 8 of the
Canadian Charter had been infringed, and he was convicted
Issue: Was the accused arbitrarily detained in contravention of his s.9 Charter right?
Holding: Yes; appeal allowed
Doherty JA
The accused was arbitrarily detained contrary to s. 9 of the Charter
 The police officer admitted that his decision to stop the motor vehicle had nothing to
25

do with the enforcement of laws relating to the operation of motor vehicles
o Hence he was not statutorily authorized
Once road safety concerns are removed as a basis for the stop, the powers associated
upon those particular concerns cannot be relied on to legitimize the stop
The detention was also not authorized by the common law
 Where an individual is detained by the police in the course of efforts to determine
whether that individual is involved in criminal activity being investigated by the
police, that detention can only be justified if the detaining officer has some
ARTICULABLE CAUSE for the detention
o There must exist a constellation of objectively discernable facts which give
the detaining officer reasonable cause to suspect that the detainee is
criminally implicated in the activity under investigation
o A hunch based entirely on intuition cannot suffice. However, something less than
the grounds required to support an arrest will suffice.
The presence of an articulable cause does not render any detention for investigative purposes a
justifiable exercise of a police officer's common law powers
 The inquiry into the existence of an articulable cause is only the first step in the
determination of whether the detention is justified
There was no articulable cause in this case justifying the detention
 The police officer had information of unknown origin that another police officer had been
told that the residence was believed to be a crack house
 He had no reason to suspect that the accused or the driver of the car was involved in
criminal activity
Lecture notes
 Using the Waterfield test, while there is a general police duty which is being carried out
here by the detaining Constable, the detention is not justified
 The articulable cause introduced by Doherty JA is the justification for detention
o The standard is lower than reasonable and probable grounds needed for
detention
 The cases of Terrence and Thompson influenced this lower standard of detention
 The case does not talk about the cause needed for search, nor s.10 Charter rights and the
right to counsel
o It also fails to mention whether this lower standard of detention allows for the use
of force
R. v. Brown (2003), 9 CR (6th) 240
This case is the leading pronouncement on racial profiling
Morden JA
Racial profiling involves the targeting of individual members of a particular racial group,
based on the supposed criminal propensity of the group
26
The attitude underlying racial profiling may be consciously or unconsciously held
The test to be applied under s. 9 of the Charter, is whether the police officer who stopped
the motorist had articulable cause for the stop
 Articulable cause exists where the grounds are reasonable and can be clearly expressed
 If a police officer stops a person based on her colour or other discriminatoiry
ground, there is no articulable cause
To succeed, the accused had to prove that it was more probable than not that there was no
articulable cause and that the real reason for the stop was race
 I.e. there is a reverse burden
Comments
 The reverse burden to prove that the cause for arrest was race has been dismissed by the
Ontario CA
Courts tend to reject racial profiling claims where race was only part of the reason for
police intervention
Some lower courts favoured a reverse onus for racial profiling, but Doherty JA for the Ontario
CA in Peart v. Peel Police Services Board (2006), held that while the reality of racial profiling
cannot be denied, it could not accept it as the rule rather than the exception when police detain a
black person
R. v. Mann, [2004] 3 SCR 59
Facts
 As two police officers approached the scene of a reported break and enter, they observed
Mann, who matched the description of the suspect, walking casually along the sidewalk
 They stopped him, he identified himself and complied with a pat-down search of his
person for concealed weapons
 During the search, one officer felt a soft object in his pocket, and as he reached into the
pocket he found a small plastic bag containing marijuana
 The accused was arrested and charged with possession of marijuana for the purpose of
trafficking
 The trial judge found that the search of his pocket contravened s. 8 of the Charter and
evidence was excluded under s. 24(2)
 The Manitoba CA set aside the acquittal and ordered a new trial, finding that the
detention and the pat-down search were authorized by law and were reasonable in the
circumstances
Issue: Was the police officer justified in looking into the accused’s pockets?
Holding: No; appeal allowed
Iacobucci J
The police were entitled to detain the accused for investigative purposes and to conduct a
pat-down search to ensure their safety, but the search of his pockets was unjustified and
the evidence discovered therein must be excluded
27
Although there is no general power of detention for investigative purposes, police may detain
an individual if there are reasonable grounds to suspect that the individual is connected to
a particular crime and that the detention is reasonably necessary on an objective view of
the circumstances
Investigative detentions carried out in accordance with the common law power recognized in this
case will not infringe the detainee's rights under s. 9 of the Charter
Investigative detentions should be brief in duration, so compliance with s. 10(b) will not excuse
prolonging, unduly and artificially, any such detention
 They do not impose an obligation on the detained individual to answer questions posed
by the police
 At a minimum, individuals who are detained for investigative purposes must be advised,
in clear and simple language, of the reasons for the detention
 Where a police officer has reasonable grounds to believe that his safety or the safety of
others is at risk, the officer may engage in a protective pat-down search of the detained
individual
 The investigative detention and protective search power must be distinguished from an
arrest and the incidental power to search on arrest
In this case, the seizure of the marijuana contravened s. 8 of the Charter
 The officers had reasonable grounds to detain the accused and to conduct a protective
search, but no reasonable basis for reaching into his pocket
Deschamps J (dissenting) concurred with the majority’s analysis on the issues of a common law
power to detain, but disagreed that the evidence obtained would bring justice into disrepute, so as
to exclude it pursuant to s.24(2) of the Charter
There is a common law power to detain and search those who the police have an articulable
cause to believe have been or will be involved in the commission of a criminal offence
 In formulating the standard which must be met in order to give rise to the common law
power to detain, the term "articulable cause" is preferable to the term "reasonable
grounds to detain"
o Using the term "reasonable grounds" could lead to the erroneous conclusion that
the same degree of justification is required to detain as to arrest, which would
undermine the very purpose of the common law power to detain
 A search incidental to detention has to be rationally connected to the purpose of the initial
detention and reasonably necessary to either to ensure the security of police officers or
the public, to preserve evidence or to prevent the escape of an offender
o (I.e. she expands the scope of what a lawful search incidental to detention can
encompass, and does not limit it to the security of the officer like the majority)
Comment
 Iacobucci believes that courts can create powers of detention to adjust the common law
to police practices
 While this case does not expand on the scope of investigative detention, it does raise
some constitutional limitations on the power of search
28
3. Roadblock Stops
R. v. Clayton and Farmer (2007), 47 CR (6th) 219 (SCC)
Facts
 A 911 caller indicated that a number of persons were openly displaying handguns in a
strip club's parking lot, and identified four of the cars in the lot
 Within minutes, police stopped the first car leaving from the lot, yet this car was not one
of the cars described by the caller
 When asked to step out of the car, Farmer protested twice, giving rise to concerns for the
officers' safety. Meanwhile, Clayton gave strange and evasive answers
o Clayton then ran away from police, was chased and subdued
o Clayton was searched by police and a loaded prohibited handgun was found in his
pocket
o Farmer was also searched and a loaded prohibited handgun was found under his
jacket
 The Court of Appeal quashed their convictions concluding that the accused's Charter
rights under sections 8 and 9 were violated
o According to the CA, the roadblock was unlawful as there was no individualized
suspicion of criminal activity, and there were no specific statutory powers to
establish roadblocks
Issue: Did the roadblock violate the accused’s Charter rights?
Holding: No; appeal allowed
Abella J
The police constables lawfully exercised their common law powers when they detained and
searched the accused
In determining the boundaries of police powers, caution was required to ensure the proper
balance between preventing excessive intrusions on an individual's liberty and privacy, and
enabling the police to do what was reasonably necessary to perform their duties in
protecting the public
Searches incident to an investigative detention could be justified if the officer believed on
reasonable grounds that his or her own safety, or the safety of others, was at risk
 This determination is made based on the totality of the circumstances: the seriousness
of the offence; the information known to the police about the suspect or the crime; and,
the extent to which the detention was reasonably responsive or tailored to these
circumstances, including its geographic and temporal scope
The initial and continuing detentions of Clayton and Farmer's car were justified based on the
information the police had, the nature of the offence, and the timing and location of the
detention
 Requiring police to stop only those vehicles described in the 911 call imposed an
unrealistic burden on police in this case, which was inconsistent with their duty to
29
respond in a timely manner, at least initially, to the seriousness of the circumstances
Based on their observations, there were reasonable grounds for the police to conclude that the
two occupants of the car they had stopped were implicated in the crime being investigated
 The officers' safety concerns also justified the searches incidental to the lawful
investigative detention
Binnie J (concurring)
On occasion the Crown will argue that a common law which authorizes police conduct that
infringes individual Charter freedoms may nevertheless be justified in the larger interest of
society
 The majority applies the pre-Charter test set out in Waterfield, however, continued use
of the Waterfield approach without modification not only adds to the problematic
elasticity of common law police powers, but sidesteps the real policy debate in which
competing individual and societal interests should be clearly articulated in the
established framework of Charter analysis
The common law Waterfield test should be modified where police claim a common law power
that is challenged on Charter grounds by requiring a court first to determine whether the power
claimed by the police exists at common law, and if so whether the common law would, if valid,
authorize police interference with Charter rights, and finally whether the interference thus
authorized can be justified under s. 1 of the Charter
 If necessary, the court may also have to determine in a particular case whether the power
was exercised reasonably by the police in the totality of the circumstances.
The common law authority of detention by a carefully tailored blockade in response to a 911 gun
call constitutes a reasonable limit under s. 1 of the Charter on the rights of the accused
guaranteed by s. 9 of the Charter to be free of arbitrary detention
 Section 1 is the proper place to weigh the accused’s individual rights against
society’s collective rights
 The protection of society from the flaunting of illegal handguns in a crowded public place
is clearly a pressing and substantial public purpose
 An investigation prompted by a gun call engages fundamental issues of public peace and
public order. Handguns pose a serious and growing societal danger
 The roadblock was a rational response to the 911 gun call
 The blockade in this case minimally impaired the accused’s right to be free from
arbitrary detention. The roadblock was tailored to the information given to the police and
anything less than a full blockade would not have served the purpose of the roadblock
 The salutary effects of the roadblock exceeded its deleterious effects
Lecture notes
 The scope of an investigative detention is a balancing exercise
 Compared to Simpson, there was more specific information with regards to the suspects,
and the presence of guns and the seriousness of the offence made the detention more
justifiable than if it had been a lesser offence
 The reasonably necessary standard in looking at the totality of circumstances comes from
30

the second branch of the Waterfield test
Binnie J in the dissent objects to an absence of Charter analysis and that the second
branch of the Waterfield case seems to not require a Charter analysis
o Better to use Charter analysis than a British case decided 20 years before the
advent of the Charter
o The Oakes test provides a better assessment of the balancing act to be made
between the objectives pursued by the State and individual freedom
Search and Seizures
In Canada, all powers to search and seize are now subject to s.8 of the Charter, which has forced
courts to grapple with the balance between the rights of a citizen to be free from state intrusion
and the need of state intervention for a legitimate purpose
s. 8 – Canadian Charter of Rights and Freedoms
Search or Seizure
Everyone has the right to be secure against unreasonable search or seizure.
Before the Charter, the purpose of a search warrant was to avoid an action in trespass
 The action in trespass or other writs would be used to determine the illegality of police
conduct
 There was no exclusionary requirement
 Because of the lack of remedies and a clear procedure to avoid illegal searches, this area
of the law was largely underdeveloped
 A good example of this is Colet v. R.
Search and seizure under the Criminal Code
The search powers conferred by the Criminal Code are often restricted to “peace officers” as
defined in s.2, which include police, correctional guards custom officers, mayors and certain
members of the Canadian Forces
The powers to search and seize without a search warrant are restricted to certain stipulated
offences
 Possession of weapons, ss.117.02(1) and 117.04(2)
 Common gaming houses, s.199(2)
 Impaired driving, s.254(2) to (4)
 Suspected stolen timber, s.339(3)
 Cockpits, s.447(2)
 Counterfeit money, s.462
 These shows that there are very few statutory grounds for warrantless searches
31
Under ss.487-489 a justice is empowered to issue a search warrant to search in relation to an
offence under the Criminal Code or other federal statute
s. 487 – Criminal Code
Information for search warrant
(1) A justice who is satisfied by information on oath in Form 1 that there are reasonable
grounds to believe that there is in a building, receptacle or place
(a) anything on or in respect of which any offence against this Act or any other Act of
Parliament has been or is suspected to have been committed,
(b) anything that there are reasonable grounds to believe will afford evidence with
respect to the commission of an offence, or will reveal the whereabouts of a person who
is believed to have committed an offence, against this Act or any other Act of
Parliament,
(c) anything that there are reasonable grounds to believe is intended to be used for the
purpose of committing any offence against the person for which a person may be
arrested without warrant, or
(c.1) any offence-related property,
may at any time issue a warrant authorizing a peace officer or a public officer who has been
appointed or designated to administer or enforce a federal or provincial law and whose duties
include the enforcement of this Act or any other Act of Parliament and who is named in the
warrant
(d) to search the building, receptacle or place for any such thing and to seize it, and
(e) subject to any other Act of Parliament, to, as soon as practicable, bring the thing
seized before, or make a report in respect thereof to, the justice or some other justice for
the same territorial division in accordance with section 489.1
...
s. 487.1(1) – Criminal Code
Telewarrants
Where a peace officer believes that an indictable offence has been committed and that it would
be impracticable to appear personally before a justice to make application for a warrant in
accordance with section 256 or 487, the peace officer may submit an information on oath by
telephone or other means of telecommunication to a justice designated for the purpose by the
chief judge of the provincial court having jurisdiction in the matter.
...
s. 487.11 – Criminal Code
Where warrant not necessary
A peace officer, or a public officer who has been appointed or designated to administer or
enforce any federal or provincial law and whose duties include the enforcement of this or any
32
other Act of Parliament, may, in the course of his or her duties, exercise any of the powers
described in subsection 487(1) or 492.1(1) without a warrant if the conditions for obtaining a
warrant exist but by reason of exigent circumstances it would be impracticable to obtain a
warrant.
 Exigent circumstances provision
s. 488 – Criminal Code
Execution of search warrant
A warrant issued under section 487 or 487.1 shall be executed by day, unless
(a) the justice is satisfied that there are reasonable grounds for it to be executed by night;
(b) the reasonable grounds are included in the information; and
(c) the warrant authorizes that it be executed by night.
s. 489 – Criminal Code
Seizure of things not specified
(1) Every person who executes a warrant may seize, in addition to the things mentioned in the
warrant, any thing that the person believes on reasonable grounds
(a) has been obtained by the commission of an offence against this or any other Act of
Parliament;
(b) has been used in the commission of an offence against this or any other Act of
Parliament; or
(c) will afford evidence in respect of an offence against this or any other Act of
Parliament.
Seizure without warrant
(2) Every peace officer, and every public officer who has been appointed or designated to
administer or enforce any federal or provincial law and whose duties include the enforcement of
this or any other Act of Parliament, who is lawfully present in a place pursuant to a warrant or
otherwise in the execution of duties may, without a warrant, seize any thing that the officer
believes on reasonable grounds
(a) has been obtained by the commission of an offence against this or any other Act of
Parliament;
(b) has been used in the commission of an offence against this or any other Act of
Parliament; or
(c) will afford evidence in respect of an offence against this or any other Act of
Parliament.
s. 29 – Criminal Code
Duty of person arresting
(1) It is the duty of every one who executes a process or warrant to have it with him, where it is
feasible to do so, and to produce it when requested to do so.
33
The Code speaks of searching a building, receptacle or place, can one then obtain a warrant to
search person?
Laporte v. Laganière (1972), 18 CRNS 357
Facts
 The police had reason to suspect that the petitioner was involved in a hold-up, which had
taken place about a year and a half before
 There are scars on the petitioner's neck and shoulder which resemble bullet wounds. Xrays reveal the presence in the petitioner's shoulder of a foreign body, a metallic object
corresponding in size and shape to a 38-calibre slug
 To remove the slug it would require more than minor or superficial surgery
o Doctors were however in agreement that the operation, if performed, would
require a general anaesthetic and ipso facto would involve a certain element of
risk to the petitioner
o They also agreed that they would not normally perform such an operation without
the consent of the patient, which they do not have
 The case then deals with is a writ of certiorari for the purpose of quashing and setting
aside a search warrant issued by justice Laganière
Issue: Did justice Laganière have jurisdiction to order a search warrant of the petitioner’s body?
Holding: No; appeal allowed
Hugessen J
All parties conceded that there is no precedent in point that they have been able to find either in
Canadian or British case law
 The issues arose in the US, however, in Re Crowder, where Curran C.J. authorized the
issuance of a search warrant for the surgical removal of what was thought to be a bullet
"lying superficially beneath the skin" of a prisoner's forearm
o This case relied on another US case Schmerber v. California, which concerned the
result of a blood test, and whereby the majority of the US Supreme Court
repeatedly emphasized the minor nature of the "operation", which involved an
almost total lack of risk
o It is to be noted that in Crowder the search warrant was authorized under
provisions of the Federal Criminal Rules of Practice
The question is then, does the right to search the person exist either under the provisions of
the Criminal Code or at common law?
Is it possible to say that a living human body is a "building, receptacle or place" (see s.487) into
which a surgical intrusion may be justified by means of a search warrant?
 Clearly it is not a "building", nor a "receptacle"
o Receptacle cannot be held to include the interior of a living human body
 With regards to “place”, the use of this word throughout the Code is indicative of the
meaning intended by Parliament, which refers to a geographic and not an anatomical
location
34
o It is clear that the word "place" was never intended to extend to or to include the
interior of a human body
The criminal law has always had to strike the precarious balance between the protection of
society on the one hand and the protection of the rights of individuals
 Both rights are equally important, but any conflict between them must wherever
possible be resolved in a manner most compatible with individual human dignity
 The constant preoccupation of our courts with the protection of the citizen against the
state results in the Crown having always to bear the burden in any criminal prosecution
 “The possibility that some guilty persons may escape the net of justice is not too high a
price to pay for the right to live in freedom”
“It is my duty to interfere and prevent what I can only describe as a grotesque perversion of the
machinery of justice and an unwarranted invasion upon the basic inviolability of the human
person”
Lectures Notes
 Parliament responded quickly to this with powers to search the person
 Under s.487(1) police officers don’t even have the power to search individuals
Parliament has recently established two special warrant powers to search the person
 A blood sample where the person is reasonably suspected to have caused bodily harm due
to impaired driving and is unable to consent to taking the sample, s.256
 Obtaining bodily substances for forensic DNA analysis in the investigation of certain
offences, such as murder, sexual assault, kidnapping, robbery, see ss.487.04-487.09
S.11 of the Controlled Substances Act also allows police powers to search a person
Parliament had also authorized post-conviction DNA samples for a national DNA bank (see ss.
487.051-487.091)
 The constitutionality of this was upheld in R. v. Rodgers (2006) by the SCC
The scheme of s. 487(1) has been judicially interpreted to include three main requirements of
search warrants
1. The informant must present the justice with information upon oath that there are
sufficient factual details to confer jurisdiction
2. The judge must act judicially in the independent assessment of the facts and in exercising
discretion as to whether to issues the search warrant
3. The search warrant must obtain sufficient description of the objects of search in relation
to category and offence
The 1985 Criminal Code amendment added the possibility of a new telewarrant procedure for
search warrants (s. 487.1(1)). and a procedure for the seizure of documents in the possession of a
lawyer with whom solicitor-client privilege is maintained
 Nonetheless, in R. v. Lavellee, Rackel & Heintz (2002), the SCC struck down the
provision dealing with solicitor-client privilege warrants as contrary to s.8 of the Charter
35
o The court declared detailed common law rules that would guide this procedure
instead
Despite the procedure laid out in the Code many illegal warrants are issued in practice, as
ultimately the efficacy of the procedure rest on the quality of those who are justices
 S.11(d) of the Charter holds that a person is to be presumed innocent until found guilty
by an independent and impartial tribunal
 The Ontario CA has held that both provincial judges and justices of the peace are
independent and impartial
o See R. v. Valente (2004), which was further confirmed by the SCC
o For Quebec see Universal Spa Ltée v. Valois (1986) and Charest v. Lippé (1990)
Even Form 1 (see below) at the end of the Criminal Code does not include the reasons why the
warrant is being issued in relation to whether there is evidence on a particular crime, there may
be evidence on a crime, or there will be evidence on a crime
FORM 1
(Section 487)
INFORMATION TO OBTAIN A SEARCH WARRANT
Canada,
Province of ................,
(territorial division).
This is the information of A.B., of ................ in the said (territorial division), (occupation), hereinafter
called the informant, taken before me.
The informant says that (describe things to be searched for and offence in respect of which search is to be
made), and that he believes on reasonable grounds that the said things, or some part of them, are in the
(dwelling-house, etc.) of C.D., of ................, in the said (territorial division). (Here add the grounds of
belief, whatever they may be.)
Wherefore the informant prays that a search warrant may be granted to search the said (dwelling-house,
etc.) for the said things
Search and seizure under the common law
The common law is an important source of search powers
 Common law searches are searches without a search warrant, since search warrants can
only be based on valid legislation
 Three types of search powers recognized at common law
1. Search incident to arrest, or investigative detention
2. Consent searches
3. Searched pursuant to the ancillary powers doctrine
36
A) Search incident to Arrest
Unlike the search powers awarded in s.487, search incident to arrest does not require
reasonable and probable grounds
 The arrest itself must be lawful, but once the standard for arrest is met, the standard for
the search is lower and must satisfy mere reasonableness
R. v. Brezack, [1949] OR 888
Facts
 The accused was charged with the assault of a police constable
 The police, having made some investigations, had reason to believe that the accused had
in possession some narcotics which were concealed in his mouth
 Acting on the information they had, they approached the accused and seized him by his
arms while holding his throat to prevent him from swallowing
o The three of them fell to the ground and a struggle ensued there
o The constable persistently tried to insert his finger in the accused’s mouth and
was bit by him
o When he finally succeeded in doing so, he discovered no drugs in his mouth
o He did not have any narcotics on his clothes either
 They then took him to his car where they found two other individuals as well as some
capsules of containing narcotics
 He was then taken into custody and later charged for assaulting the police officer
Issue: Was the search of the accused’s mouth lawful?
Holding: Yes; appeal dismissed
Robertson CJO
The evidence in this case shows that the constable was engaged in the lawful execution of
his duty as a peace officer in making the arrest (given the information he had), and that the
attempt to search the inside of appellant's mouth was a justifiable incident of that arrest
 That the appellant was liable to arrest without a warrant is beyond question, and the
evidence afforded by the capsules containing a narcotic, found in appellant's motor car
strongly supports the reasonableness of the constable's belief in the information he had
It is important to observe that the search that was made is justifiable as an incident of the
arrest
 The constable who makes an arrest has important duties, such as to see that the
prisoner does not escape by reason of being armed, and to see if any evidence of the
offence for which he was arrested is to be found upon him
 A constable may not always find his suspicions to be justified by the result of a search, it
is sufficient if the circumstances are such as to justify the search as a reasonable
precaution
There was an arrest here when the constables seized the person of the appellant
 The evidence would indicate that they did not inform him immediately that he was
37
arrested, and of the cause of the arrest.
Constables have a task of great difficulty in their efforts to check the illegal traffic of drugs
 While it is important that constables should be instructed that there are limits upon their
right of search, including search of the person, they are not to be encumbered by
technicalities in handling the situations with which they often have to deal in narcotic
cases
Lecture Notes
 This is a pre-Charter case
Laporte v. Laganière supra (1972), 18 CRNS 357
Facts
 For full facts see above
Issue: The court refused to extend the power to search incident to arrest to include a surgical
operation
Hugessen J
There is no doubt that there is a common law right to search a prisoner at the time of her
arrest
This common-law right of search is limited to that search which is incidental to the making
of an arrest or the continued detention of the prisoner in safe custody
 The reasons for such right are is to make the arrest effective, to ensure that evidence
does not disappear and to prevent the commission of a further offence
 Examples would be to ascertain whether a suspect has weapons that could harm a police
officer or others, or to recover evidence of the crime which the accused allegedly
committed
Nothing in the jurisprudence justifies a surgical intrusion into the body of a prisoner
months after his arrest, for the purpose of obtaining evidence against him on a different
charge than that for which he is being held
R. v. Tomaso (1989), 70 CR (3d) 152
Facts
 The accused was involved in a car crash which resulted in a fatality
 The police obtained blood from a free-flowing wound while the accused was unconscious
in hospital to determine whether he was impaired while driving
Issue: Was the seizure of the accused's blood while he was unconscious unreasonable?
Holding: Yes; appeal allowed
Howland CJO
At common law a police officer has the power to search a person as incident to a lawful
arrest, and to take from his person any property which the officer reasonably believes is
connected with the offence charged, or which may be used as evidence against the person
arrested, or any weapon or instrument found upon the person arrested
38
In R. v. Debot the Ontario CA went further and held that some searches incident to arrest
can PRECEDE THE ACTUAL ARREST, provided that the police officer had reasonable
grounds, prior to the search, for arresting the respondent under
In this case the seizure of the blood sample took place on July 14, however, the police officer
decided to await the analysis of the blood sample and arrest the accused on July 29
 It has not been established that the police officer had reasonable grounds on July 14 for
the arrest, nor could it be said that the analysis of the blood sample was not part of the
justification for such arrest
 In any case, it would be going too far to attempt to justify the seizure on July 14 on
the basis that it was incident to an arrest more than two weeks later, for the offences
with which the appellant was charged
It should be noted that since the offence the Criminal Code has been amended (s. 254(3))
allowing a peace officer, who believes on reasonable and probable grounds that an offence is or
has been committed as a result of the consumption of alcohol, the right to require that a person
have blood samples taken by or under the direction of a qualified medical practitioner
 This right may be exercised only where there are reasonable and probable grounds to
believe that the taking of breath samples is not possible, or is impracticable, by reason of
any physical condition of the person
 Section 256 similarly makes provision for the issue of a warrant for the taking of blood
samples where there is an accident resulting in the death of or bodily injury to any
person, and the medical practitioner is of the opinion that the person is unable to consent,
but that the taking of the samples would not endanger his life or health
o These amendments did not come into force until after the offence in question
Lecture Notes
 While a search may precede the arrest, the grounds for the arrest had to be present
at the time of the search
R. v. Stillman (1989), 70 CR (3d) 152
Facts
 The accused was arrested in 1991 for the brutal murder of a teenage girl
 He was the last person seen with the victim on the night of the crime
 The victim died from wounds to the head. Semen was found in her vagina and a human
bite mark had been left on her abdomen.
 At the police station, the accused’s lawyers informed the police by letter that the accused
was not consenting to provide any bodily samples, including hair and teeth imprints, or to
give any statements
o Once the lawyers left, police officers took, under threat of force, scalp hair
samples from the accused and he was made to pull some of his own pubic hair;
Plasticine teeth impressions were also taken
o A police officer then interviewed the accused for an hour in an attempt to obtain a
statement
o The accused sobbed throughout the interview and, after being permitted to call his
lawyer, he went to the washroom escorted by an officer where he used a tissue to
39
blow his nose, which was retrieved from the wastebasket for DNA testing
 The accused was subsequently released but was arrested again several months later. At
that time, a dentist took new impressions of the accused’s teeth without his consent in a
procedure lasting two hours. More hair was taken from the accused, as well as a saliva
sample and buccal swabs
 Following a voir dire held to determine the admissibility of certain evidence, the trial
judge found that the hair samples, buccal swabs and teeth impressions had been obtained
in violation of s. 8 of the Charter but concluded that the evidence was nevertheless
admissible
o With respect to the tissue containing mucous, he found that it had not been
obtained in violation of s. 8 and should thus be admitted
 The accused was later convicted by a jury of first degree murder
 The majority of the CA upheld the trial judge’s ruling and affirmed the verdict
Issue: Was the evidence obtained in contravention to the accused’s s. 8 rights?
Holding: Yes; the appeal was allowed and a new trial ordered
Cory J
There are three requirements which must be met if a search is to be found reasonable: (a) it
must be authorized by law; (b) the law itself must be reasonable; and (c) the manner in which the
search was carried out must be reasonable
 At the time the seizure occurred, the Criminal Code only provided a procedure for
obtaining a warrant to search a "building, receptacle or place"; it did not authorize the
search of a person, nor the seizure of parts of the body, and as such the seizure was not
statutorily authorized
Three conditions must be satisfied in order for a search to be validly undertaken pursuant
to the common law power of search incident to a lawful arrest
1. The arrest must be lawful
2. The search must have been conducted as an "incident" to the lawful arrest
3. The manner in which the search is carried out must be reasonable
1. The accused’s arrest was lawful since the police officers, subjectively, were under the
impression that they had reasonable and probable grounds to believe that the accused had
committed the murder
2. The common law power of search incidental to arrest does not extend beyond the
purpose of protecting the arresting officer from armed or dangerous suspects or of
preserving evidence that may go out of existence or be otherwise lost
 The power of search incidental to arrest cannot be so broad as to encompass the seizure
without valid statutory authority of bodily samples in the face of a refusal to provide
them
 These samples are usually in no danger of disappearing
 Parliament has recently amended the Code so as to create a warrant procedure for the
seizure of certain bodily substances for the purposes of DNA testing
o If this type of invasive search and seizure came within the common law power of
search incident to arrest, it would not have been necessary for the government to
40
create a parallel procedure for the police to follow
L’Heureux-Dubé (dissenting)
While the search and seizure at issue was clearly not necessary in order to protect the accused,
the police or the public, they were authorized by law under the common law search power
incidental to arrest
The following guidelines strike a proper balance between society's interest in detecting and
punishing crime and the individual's interests in personal privacy and autonomy over his
own body where the taking of bodily samples or impressions occurs as an incident to arrest
1. The police must exercise their discretion to conduct the search given all of the
circumstances
2. The search must be for a valid objective in pursuit of the ends of criminal justice without
running counter to the general objectives of the proper administration of justice
3. Consideration must be given to the intrusiveness involved in the search: the more
intrusive, the higher the threshold for finding that the taking of bodily samples or
impressions is both justified and conducted in a reasonable manner in given
circumstances
4. The police must have reasonable and probable grounds to conduct the search
5. The search must be predicated on sufficiently important circumstances; those
circumstances will generally be established where: (a) it is impracticable to obtain a
warrant to secure the desired evidence; (b) such evidence cannot be obtained by a less
intrusive means; (c) there is no alternative evidence available; (d) the offence for which
the arrest was made is a serious one; and (e) public policy is served by the type of search
at issue
6. The manner in which the search is cinducted myst not be abusive or unreasonable given
the totality of circusmtances
[As for the discarded tissue, the court divided more narrowly (5:4) held that the seizure violate
s. 8 of the Charter]
Cory J
 Where an accused who is not in custody discards an item offering potentially valuable
DNA evidence, the police may ordinarily collect and test the item, since the accused
abandoned the item and ceased to have a reasonable expectation of privacy in it
 The situation is different, however, when an accused is in custody
 Here, the accused had announced through his lawyers that he would not consent to
the taking of any samples of his bodily fluids
 In these circumstances the seizure was unreasonable
The tissue containing the mucous, however, should not be excluded
 The police did not force, or even request, a mucous sample from the accused
 Where an accused who is not in custody discards an item offering potentially valuable
DNA evidence, the police may ordinarily collect and test the item without any concern
about consent since, in the circumstances, the accused abandoned the item and ceased to
41


have a reasonable expectation of privacy in it.
The violation of the accused’s Charter rights with respect to the tissue was not serious.
The seizure did not interfere with the accused’s bodily integrity, nor cause him any loss
of dignity
In any event, the police could and would have obtained the discarded tissue. It was
discoverable and the administration of justice would not be brought into disrepute if
the evidence obtained from the mucous sample were to be admitted
McLachlin J (dissenting)
 The tissue was not obtained as a result of a search of the appellant nor was it seized from
him; he had discarded it
 To put it another way, the appellant had abandoned any privacy interest in the
tissue that he may have had
Lecture Notes
 The court disagrees with the wide powers of search incidental to arrest awarded in
Cloutier and does not seem to allow a search incidental to arrest that would lead to the
discovery of evidence, beyond its preservation
R. v. Caslake, [1998] 1 SCR 51
Facts
 An RCMP officer, several hours after arresting the accused for possession of narcotics,
conducted an inventory search of the accused's impounded car pursuant to police policy
and found $14,000 in cash and two individual packages of cocaine
 He did not have the accused’s permission or a search warrant
 The accused unsuccessfully appealed his conviction of possession of marijuana for the
purposes of trafficking and of possession of cocaine to the Manitoba CA
 At issue here was whether of Rights and Freedoms which guarantees the right to be
secure against unreasonable search or seizure, and if not consistent, whether the evidence
should have been admitted
Issue: Was the search of the car inconsistent with s. 8 of the Charter?
Holding: Yes (but the evidence should not be excluded); appeal dismissed
Lamer CJ
Since search incident to arrest is a common-law power, there are no readily ascertainable
limits on its scope
 It is therefore the courts' responsibility to set boundaries which allow the state to
pursue its legitimate interests, while vigorously protecting individuals' right to
privacy
There are three main purposes of search incident to arrest
1. Ensuring the safety of the police and public
2. The protection of evidence from destruction at the hands of the arrestee or others
3. The discovery of evidence which can be used at the arrestee's trial
42
The restriction that the search must be "truly incidental" to the arrest means that the
police must be attempting to achieve some valid purpose connected to the arrest
 Whether such an objective exists will depend on what the police were looking for and
why; there are both SUBJECTIVE and OBJECTIVE aspects to this issue
o The police must have ONE of the purposes for a valid search incident to
arrest in mind when the search is conducted (objective purpose)
o Further, the officer's belief that this purpose will be served by the search must
be a reasonable one (subjective purpose)
 It would be contrary to s.8 of the Charter to allow searches incident to arrest which do
not meet these subjective and objective criteria
 There is no need reasonable and probable grounds, the only requirement is that there
be some reasonable basis for doing what the police officer did
Delay and distance do not automatically preclude a search from being incidental to arrest,
but they may cause the court to draw a negative inference
 That inference may be rebutted by a proper explanation
A police search of the car for the purpose of finding evidence which could be used at the trial on
the charge of possessing marijuana for purposes of trafficking would have been well within the
scope of the search incident to arrest power, as there was clearly sufficient circumstantial
evidence to justify a search
 However, the police cannot rely on the fact that, objectively, a legitimate purpose for
the search existed when that is not the purpose for which they searched
 Agents of the state must act in accordance with the rule of law; hence, they must not only
objectively search within the permissible scope but also turn their mind to this scope
before searching, and satisfy themselves that there is a valid purpose for the search
Here, the purpose of the search was to inventory the contents of the vehicle which falls
outside the bounds of the legitimate purposes of search incident to arrest
Bastarache J
The search, given that the arrest was lawful, was incidental to the arrest because it was related to
the arrest, rather than the arrest's being incidental to the search
 Regardless of the police officer's subjective belief in the purpose and justification for
his inventory search, the officer had the right to search the vehicle (as part of the
accused’s immediate surroundings)
 The question of delay was immaterial for the search to qualify as "incidental".
This power draws its authority from the arrest itself
 It is not necessary to establish reasonable and probable grounds independently to conduct
a search incidental to an arrest
The common law power to search incident to an arrest is not unreasonable and does not violate s.
8 of the Charter if it is consistent, in the circumstances, with the proper administration of justice
 The issue must be whether the inventory search was truly incidental to the arrest and
43
reasonably performed, and not whether the Charter was infringed because the police
officer could have obtained a warrant
Lecture Notes
 Caslake seems to narrow the scope of search incidental to arrest by adding the subjective
standard
 Here it is clear (unlike Stillman) that the search power incidental arrest can include the
discovery of new evidence
o The standard is still reasonableness
Caslake was applied to authorize the seizure of a briefcase as incident to an arrest of a person
found in possession of a stolen vehicle (see R. v. Mohamad (2004))
B) Consent to search
Law Reform Commission of Canada, Working Paper 30: Police Powers – Search and
Seizure in Criminal Law Enforcement
 The common law tolerance of search with consent is founded on the principles that one
who has invited to an act being done towards her cannot, when she suffers from it,
complain of it as a wrong
 Few cases of consent search appear to have been litigated in the Canadian context
o The discretionary basis upon which police forces have adopted practices to carry
out consent searches points to wide gaps in Canadian law
R. v. Deadman supra (1985), 46 CR (3d) 193
Facts
 The accused was randomly stopped as part of a spot check program, whose principal aim
is to detect, deter and reduce impaired driving
 He was accused with failing, without reasonable excuse to provide a breath sample, but
was acquitted
 On appeal, the acquittal was confirmed as it was held that there was neither statutory nor
common law authority to require him to stop his motor vehicle, so he had a reasonable
excuse
 The Ontario CA reversed the acquittal
Issue: Was the seizure lawful because of the accused’s consent?
Le Dain J
The accused's compliance with the signal to stop did not alter the legal basis on which it
must be justified
 Police officers, when acting or purporting to act in their official capacity as agents of the
state, only act lawfully if they act in the exercise of authority which is either conferred by
statute or derived as a matter of common law from their duties
 The reason for this is the authoritative and coercive character of police action
A person should not be prevented from invoking a lack of statutory or common law
authority for a police demand or direction by reason of compliance with it in the
44
ABSENCE OF A CLEAR INDICATION from the police officer that the person is free to
refuse to comply
 Because of the intimidating nature of police action and uncertainty as to the extent of
police powers, compliance in such circumstances cannot be regarded as voluntary in any
meaningful sense
A person should not be penalized for compliance with a signal to stop by having it treated as a
waiver or renunciation of rights, or as supplying a want of authority for the stop
Dickson CJ (concurring)
Police officers only act lawfully when they exercise authority conferred upon them by statute or
at common law
 The apparent voluntary compliance by a citizen with a police request to stop a
motor vehicle cannot alter the legal basis which must justify such police action when
it is challenged in later proceedings
Having regard to the authoritative and coercive character of police requests, submission to a
police officer's exercise of apparent authority, such as a demand to stop at a roadblock, cannot
be characterized as voluntary or consensual unless it was clear to the person at the time
that he was free to refuse to comply
Deadman has now been relied upon to hold that the consent to a search must have been real
and voluntary, and that the person searched must have sufficient awareness to have waived the
constitutional right conferred by s.8 of the Charter, and the individual must be aware of the
consequences of giving consent
 See R. v. Nielson (1988), R. v. Wills (1992) and R. v. Borden (1994(
Note on R. v. Wills (1992), 70 CCC (3d) 529
Doherty JA
The application of the waiver doctrine to situations where it is said that a person has
consented to what would otherwise be an unauthorized search or seizure requires that the
Crown establish on the balance of probabilities that:
1. There was a consent, express or implied
2. The giver of the consent had the authority to give the consent in question
3. The consent was voluntary in the sense that it was not the product of police oppression,
coercion or other external conduct which negated the freedom to choose
4. The giver of the consent was aware of the nature of the police conduct to which she was
being asked to consent
5. The giver of the consent was aware of her right to refuse to permit the police to engage in
the conduct requested
6. The giver of the consent was aware of the potential consequences of giving the consent
Note on R. v. Lewis (1998), 122 CCC (3d) 481
45
Doherty JA
The police are not under a "duty" to advise a person of the right to refuse to consent to a search
in the sense that the failure to do so will amount to a violation of s. 8
 Unlike s. 10(b) of the Charter, s. 8 does not contain an informational component
 The failure to advise a person of the right to refuse to consent to a search may,
however, lead to a violation of s. 8 where the police conduct can be justified only on
the basis of an informed consent
 It is well established that a person cannot give an effective consent to a search unless the
person is aware of their right to refuse to consent to that search
 Where the police do not inform a person of the right to refuse to consent to a search, it is
certainly open to a trial judge to conclude that the person was unaware of the right to
refuse and could not, therefore, give an informed consent
C) Consent incident to duties of police officers if reasonably necessary (ancillary
powers doctrine)
Colet v. R., [1981] 1 SCR 2
Facts
 The accused was charged with five counts including two counts of attempted murder and
two counts of intending to cause bodily harm, all of which arose out of his conduct in
defence of his property, which the City of Prince Rupert had instructed to be demolished,
against what he deemed to be a wrongful intrusion of police officers acting under the
purported authority of a warrant to seize firearms
 A verdict of acquittal was entered at a trial held before a judge and a jury in view of the
trial judge's ruling that a warrant to seize did not give police officers a right to search
 The British Columbia CA allowed the appeal and directed a new trial
Issue: Did the power to seize confer ancillary powers to the police to search around the accused’s
property?
Holding: No; appeal allowed
Ritchie J
What is involved here is the longstanding right of a citizen of this country to the control and
enjoyment of his own property, including the right to determine who shall and who shall not be
permitted to invade it
There are occasions when the interest of a private individual in the security of his house
must yield to the public interest, when the public at large has an interest in the process to
be executed
Whatever the occasion may be the police are not justified in making an entry unless they
have first announced their presence and demonstrated their authority by stating a lawful
reason for their entry
 In this case, although the police officers waved the warrant at the appellant from a
distance, it was not a warrant "to search the premises" nor in my view did the
46
authority to seize specified in that warrant carry with it the right to enter and
search.
Any statutory provision authorizing police officers to invade the property of others without
invitation or permission would be an encroachment on the common law rights of the property
owner and in case of any ambiguity would be subject to a strict construction in favour of the
common law rights of the owner
.
Any provision authorizing police officers to search and enter private property must be
phrased in express terms and the provisions of the Interpretation Act are not to be considered
as clothing police officers
 Extensive number of sections of the Criminal Code expressly include the dual authority
"to search" and "to seize"
If Parliament intended to include the power "to search" in the provisions of s. 105(1), the failure
to do so was a clear case of legislative oversight
Lecture Notes
 Ritchie J required strict statutory construction of the power to search within the power to
seize
o The principle of legality requires Parliament to speak clearly when justifying the
intrusion of individual’s liberty and property
 Parliament reacted to this by adding the words “and search” to the statutory provisions
that dealt with seizure
R. v. Deadman supra (1985), 46 CR (3d) 193
Facts
 The accused was randomly stopped as part of a spot check program, whose principal aim
is to detect, deter and reduce impaired driving
 He was accused with failing, without reasonable excuse to provide a breath sample, but
was acquitted
 On appeal, the acquittal was confirmed as it was held that there was neither statutory nor
common law authority to require him to stop his motor vehicle, so he had a reasonable
excuse
 The Ontario CA reversed the acquittal
Issue: Did the police officer possess authority, either statutory or at common law, to require the
appellant to stop his motor vehicle?
Holding: Yes; appeal dismissed
Le Dain J
The test laid down in R. v. Waterfield, while generally invoked in cases where it is at issues
whether a police officer had acted in the execution of her duties, has been recognized as a test for
whether the officer had common law authority to do what she did
 Applying the Waterfield test, the random vehicle stop was a prima facie unlawful
interference with liberty since it was not authorized by statute
47
The right to circulate in a motor vehicle on the public highway may be described as a liberty;
however, it cannot be regarded as a fundamental liberty like an individual's right of
movement, since it is a licensed activity subject to regulation and control for the protection
of life and property
The random stop falls within the general scope of police duties to prevent crime and to
protect life and property by the control of traffic as these are the very objects of the
program, a measure intended to improve the deterrence and detection of impaired driving
 The stop was not an unjustifiable use of police power because it was both necessary
to the execution of police duty and reasonable, having regard to the nature of the
liberty interfered with and the importance of the public purpose served by the
interference
Dickson CJ (dissenting)
It has always been a fundamental tenet of the rule of law that police, in carrying out their general
duties as law enforcement officers have limited powers and are only entitled to interfere with the
liberty or property of the citizen to the extent authorized by law
The fact that a police officer has a general duty to prevent crime and protect life and property
does not mean that he or she can use any or all means for achieving these ends.
In the criminal law, the rules and principles relating to arrest establish justifiable limits upon a
citizen's liberty
 Short of arrest, the police have never possessed legal authority at common law to
detain any one against his or her will for questioning or to pursue an investigation
 These random stops are indistinguishable from detention for questioning or
investigation whether or not they might be committing a criminal offence and,
without validly enacted legislation to support them, are unlawful
 It would be contrary to the long standing protection accorded individual liberty by the
common law and detrimental to the individual's fundamental right to be free from
arbitrary interference to conclude that this action of the police was authorized and lawful
The fact that driving a motor vehicle is a licensed activity subject to regulation and control in the
interests of safety is irrelevant to police power if the conditions for licensing have been met and
are adhered to
Lecture Notes
 This case establishes the new police power of investigative detention
 The Waterfield test is the source of the doctrine of ANCILLARY POLICE POWERS
o It came about, however, in trying to determine whether a police officer was acting
within her legally prescribed authority
 The general duties of police officers, were a common law authority that would justify this
type of stopping
 If the powers of the police officer are prima facie unlawful, you can check whether
the actions fall within the general powers of the police, and whether they are
justifiable
48
o The stops were justifiable given the importance of highway traffic and the fact
that the activity was a licensed one
 A crucial point about this case, was whether the creation of these powers was a matter of
the legislature or courts could do it as well
The SCC accepted the ancillary powers doctrine as given in Godoy
 Here, the court recognized a common law power to enter the premises to investigate a
disconnected 911 call
Other federal statutes
Many other federal statutes contain powers of search without warrant
 An example is the issuance of writs of assistance, conferring the holder a “walking search
warrant” and not subject to judicial control
 In Noble writs of assistance were held to be contrary to s.8 of the Charter
o The Law Reform Commission of Canada had reached this conclusions earlier and
recommended their replacement with the telewarrant procedure
o The Code now provides for this under s. 487.1
R. v. Noble (1984), 48 OR (2d) 643
Martin JA
An understanding of the nature of writs of assistance is essential to a determination of their
constitutional validity
There are at the present time in Canada four statutes which provide for the issuance of writs of
assistance: the Customs Act, the Excise Act, the Narcotic Control Act, the Food and Drugs Act
 There are some variations in the four statutes with respect to the powers conferred on the
holder of a writ of assistance
The writ is not a grant of authority to conduct a particular search of particular premises,
but empowers the writ-holder without limitation as to time or place to exercise the search
and seizure powers conferred by the statute under which the writ is issued
At common law, general warrants, that is, warrants which did not specify the person to be
arrested, the premises to be searched and the things to be seized, are illegal and void
 It is not fit, that the receiving or judging of the information should be left to the discretion
of the officer
 The magistrate ought to judge; and should give certain directions to the officer
Writs of assistance, because of the generality of the power conferred by them, are sometimes
classified as a statutory species of general warrant
 The Law Reform Commission of Canada, however, expresses the view that a writ of
assistance is more properly characterized as a certificate of the legal competence of its
bearer to exercise a statutory power of search and seizure without a
49

The commission also takes the view that since the judge of the Federal Court has no
discretion to refuse to grant an application made in proper form for the issuance of the
writ, in essence the writ is a ministerial or executive authority or commission to the
holder to exercise without warrant statutory powers of search
The fact that the writ is granted by the Federal Court inevitably tends to give the writ an aura of
judicial authorization which is misleading, since the court has no discretion with respect to its
granting.
Care must be taken to insure that the writs do not say anything other than that which Parliament
has directed and does not contain anything that is calculated to mislead the reader into thinking
that the writ is anything other than that which the terms of the legislation require
The writ of assistance is a document issued out of the Federal Court which identifies the holder
as a person entitled to exercise without a warrant the statutory powers of search and seizure
under the relevant statute
 It is like an identification card signifying that the holder is entitled to conduct warrantless
searches and seizures pursuant to the search and seizure powers conferred by the relevant
statute
 Consequently, searches under a writ of assistance are warrantless searches by
designated persons pursuant to statutory powers.
Provincial statutes
There are hundreds of search powers under various provincial statutes
 Wide powers to stop, inspect and seize are found under provincial Highway Traffic and
Liquor acts
Constitutional minimum standards
s. 8 – Canadian Charter of Rights and Freedoms
Search or Seizure
Everyone has the right to be secure against unreasonable search or seizure.
The Canadian protection against unreasonable search and seizure is different from the more
elaborate version of the Fourth Amendment to the US Constitution, which is discussed in Rao
R. v. Rao (1984), 46 OR (2d) 80
Facts
 At the trial of the accused on a charge of possession of cannabis resin for the purpose of
trafficking, the Crown sought to introduce certain narcotics which had been seized from
the accused's office
50

The search of the office had been conducted without warrant, relying on the warrantless
search provisions of s. 10(1)(a) of the Narcotic Control Act
 The accused protested against the warrantless search and asked to call a lawyer
 The trial judge held that the warrantless search was unreasonable and contrary to s.8 of
the Charter and excluded the evidence having regard to all the circumstances
 The accused was acquitted
Issue: Did warrantless searches violate s.8 of the Charter?
Holding: Yes; appeal dismissed
Martin JA
In looking at the Fourth Amendment of the US Constitution, the first clause merely provides that
a search or seizure must meet the standard of reasonableness, while the second clause merely
provides that a warrant must meet the requirements specified in that clause
An examination of the decisions of the US Supreme Court reveals two distinct lines of
approach to the interpretation of the Fourth Amendment
 Some consider that the first clause should be read separately from the warrant
clause, and that the existence of a warrant is only one factor in determining the
reasonableness of a search
 Others have insisted that the two clauses must be read together so that warrantless
searches are per se unreasonable, subject only to a few well-delineated exceptions
where it would be impracticable to obtain a warrant
 There is some indication that the first interpretation is becoming more predominant
In Texas v. Brown, the court listed a number of exceptions to the warrant requirement, two
of which are of particular interest in the present case
 The first of those exceptions is the "automobile" exception, where it is not practicable
to obtain a warrant because the vehicle can be quickly put out of reach of the jurisdiction
of a police officer
 The second exception is the "exigent circumstances" exception
o The Supreme Court of the United States has authorized bodily intrusions to seize
evidence without a warrant where the police have reasonable cause to believe that
the evidence exists and the delay in obtaining a warrant would almost certainly
result in the loss or destruction of the evidence
o A number of appellate courts have held that an entry and search of premises to
prevent the removal or destruction of illicit drugs falls within the "exigent
circumstances" exception
Although the language of s. 8 of the Charter unmistakably shows the influence on the
draftsman of the first clause of the Fourth Amendment, the second clause of the Fourth
Amendment has no counterpart in the Charter
o Consequently, the central issue is whether the particular search or seizure meets the
constitutional requirement of reasonableness unfettered by any constitutional
requirement of a warrant
Whether a search was authorized by a warrant may be an important or even critical factor
51
in assessing the reasonableness of a search in a given case, but the omission from s. 8 of a
warrant provision signals caution in the extent of the use of the American jurisprudence
Although there is no express constitutional warrant requirement under s. 8 of the Charter, it is
manifest that the legal systems derived from the common law generally require a warrant
to enter and search private premises, as distinct from vehicles and vessels which may rapidly
move away
Section 10(1)(a) of the Narcotic Control Act authorizes a warrantless search of a "place" other
than a dwelling-house by a peace officer who has reasonable grounds for believing that the
"place" contains a narcotic
o The word "place" includes places of fixed location as well as vehicles
The search of an office without a warrant where the obtaining of a warrant is not
impracticable, is unreasonable and, to that extent, s. 10(1)(a) is of no force or effect
o The search of an office without a warrant in circumstances where it is not practicable to
obtain a warrant may be entirely reasonable
o A warrantless search of vehicles, vessels or aircraft, which may move quickly away, may
be reasonable where there are reasonable grounds for believing that such contains a
narcotic
Hunter et al. v. Southam Inc., [1984] 2 SCR 145
Facts
 Pursuant to s. 10(1) of the Combines Investigation Act, Lawson A. W. Hunter, Director of
Investigation and Research of the Combines Investigation Branch authorized several
Combines Investigation officers to enter and examine documents and other things at
Southam premises in Edmonton "and elsewhere in Canada"
 The authorization was certified by a member of the Restrictive Trade Practices
Commission pursuant to s. 10(3) of the Act
o The authorization had a great scope
 The Canadian Charter was proclaimed after the authorization was made but before the
actual search had begun
 Southam Inc. unsuccessfully sought an interim injunction pending trial of the question
whether the search was in violation of s. 8 of the Charter
 The Alberta CA ordered all documents taken from the respondent's premises sealed as an
interim measure and proceeded with the appeal
 Hunter appeals the Court's finding that s. 10(3), and, by implication, s. 10(1) of the Act
were inconsistent with the Charter and therefore of no force or effect
Issue: Did the legislation violate the protection from unreasonable search and seizure?
Holding: Yes; the appeal was dismissed
Dickson J
The Charter is a purposive document which must be subjected to a purposive analysis
 Section 8 of the Charter guarantees a broad and general right to be secure from
unreasonable searches and seizures which extends at least so far as to protect the right
of privacy from unjustified state intrusion
52


Its purpose requires that unjustified searches be prevented
It is not enough that a determination be made, after the fact, that the search should not
have been conducted; this can only be accomplished by a requirement of prior
authorization
Accordingly, prior authorization, where FEASIBLE, is a precondition for a valid search
and seizure
 Warrantless searches are PRIMA FACIE UNREASONABLE under s. 8
o The party seeking to justify a warrantless search bears the onus of rebutting
the presumption of unreasonableness
The procedures established by s. 10(3), however, are constitutionally defective in two respects
 First, for the authorization procedure to be meaningful, it is necessary for the person
authorizing the search to be able to assess the conflicting interests of the state and
the individual in an entirely neutral and impartial manner
o He must not be someone charged with investigative or prosecutorial functions
under the relevant statutory scheme
 Second, reasonable and probable grounds, established upon oath, to believe that an
offence has been committed and that there is evidence to be found at the place of the
search, constitutes the minimum standard consistent with s. 8 of the Charter for
authorizing searches and seizures
o Subsections 10(1) and 10(3) of the Act do not embody such a requirement and
therefore, do not measure up to the standard imposed by s. 8 of the Charter.
The Court will not attempt to save the Act by reading into it the appropriate standards for issuing
a warrant
 It should not fall to the courts to fill in the details necessary to render legislative lacunae
constitutional
Dickson J refrained from carrying out a s.1 analysis, because the parties had not submitted any
evidence to support it
Lecture notes
 This was one of the first cases to discuss the restriction of state action in individual’s
property interests
 What is being protected from intrusion is not property per se, but the reasonable
expectation of privacy
 A proper warrant, wherever feasible, process is necessary to justify the state’s intrusion
 The conditions for a warrant to be constitutionally valid were also laid out in the case
o The person granting it must be aware of the competing interests between the
individual and the state
o The assessment must be done in advance of the search
o The search must be authorized by law, the law must be reasonable and must be
executed reasonably
o It must be given under oath
 Prior authorization is required, but all this case states about an exception to this, is that it
is not necessary when it is not feasible
53
Hunter recognized an exception to the warrant requirement: where it is not feasible to get one
 Under this case the search of any premise (including a person or vehicle) is whether the
warrant was feasible
 Under Rao, however, the focus is on reasonableness, and a warrant is only a critical
factor
o This seems to suggest that under the Rao standard courts are less likely to assert a
warrant requirement for searches of vehicles
A s.8 challenge against a search can be made against a search power (like Rao, Hunter and
Noble) or against the way in which the search is exercised, as will be illustrated below
R. v. Collins, [1987] 1 SCR 265
Facts
 The accused had been under surveillance by two members of the RCMP Drug Squad
 Having arrested her husband who was found in possession of heroin in his, a police
officer approached her in a pub, laid hold of her identifying himself as a police officer,
grabbed her throat and pulled her to the floor
 The officer directed her to let go of an object clenched in her hand, which was a balloon
containing heroin
 The search was found to be unlawful and therefore unreasonable and in violation of s. 8
of the Charter but the evidence was nevertheless admitted because the accused failed to
satisfy the judge that it should be excluded under s. 24(2) of the Charter
 The Court of Appeal unanimously dismissed the accused's appeal
Issue: Was the search unreasonable?
Holding: Yes; the appeal was allowed and a new trial was ordered
Lamer J
The accused bears the burden of persuading the court on a balance of probabilities that a
Charter right has been infringed
 That appears from the wording of s. 24(1) and (2), and most courts which have
considered the issue have come to that conclusion
 The burden of persuasion is the balance of probabilities
The courts have also developed certain presumptions, in particular, the SCC held in Hunter v.
Southam Inc. that a warrantless search was prima facie "unreasonable"
o This shifts the burden of persuasion from the appellant to the Crown: once the appellant
has demonstrated that the search was a warrantless one, the Crown has the burden
of showing that the search was, on a balance of probabilities, reasonable
A SEARCH WILL BE REASONABLE IF authorized by law, if the law itself is reasonable,
and in the manner in which the search was carried out is reasonable
 As the accused did not challenge the constitutionality of s. 10(1) of the Act, the issues that
remain to be decided here are whether the search was unreasonable because the officer
did not come within s. 10 of the Act, or whether, while being within s. 10, he carried out
54

the search in a manner that made the search unreasonable
The Crown here was not able to prove the search reasonable because it did not establish
under s. 10 of the Narcotic Control Act that the officer had reasonable and probable
grounds for believing there were narcotics in the place where the person was searched
The nature of the belief will also determine whether the manner in which the search was
carried out was reasonable
o If the officer is lawfully searching a person whom he believes on reasonable grounds to
be a "drug handler", then the "throat hold" would not be unreasonable
Because the failure to establish the grounds for the search was due to an error by the trial judge, a
new trial should be ordered if the evidence would be excluded on the record as it now stands.
It is important to note that there is no blanket exception to the warrant requirement for motor
vehicle searches, and in fact an assessment of reasonableness of the search may be carried out
 See R. v. D. (I.D) (1987) and R. v. Grant (1993)
R. v. Edwards, [1996] 1 SCR 128
Facts
 The accused was convicted of possession of drugs for purposes of trafficking
 He had been suspected of drug dealing out of his car using a cellular phone and of
keeping the drugs at his residence or at his girlfriend's apartment
 The police arrested him on a traffic offence
 Two officers later called at his girlfriend's apartment and gained her cooperation through
a number of statements, some of which were lies and half-truths
o Once inside, the accused's girlfriend directed them to the location of a significant
cache of drugs. She was arrested a short time later but the charges against her
were later dropped
o At no time prior to being taken into custody was she advised of her right to refuse
entry to the police or of her right to counsel
o At the police station, she gave a statement naming the accused as the person who
put the drugs in her apartment
 At trial and on appeal, the accused denied being the owner of the drugs
 The accused's appeal from conviction was dismissed with a dissenting opinion which
found a reasonable expectation of privacy giving rise to the possibility of an infringement
of his s. 8 Charter rights against unreasonable search or seizure
Issue: Did Edwards hold a reasonable expectation of privacy against the search and seizure of his
girlfriend’s place?
Holding: No; the appeal was dismissed
Cory J
A reasonable expectation of privacy is to be determined on the basis of the totality of the
circumstances, but without reference to the conduct of the police during the impugned search
 The factors to be considered may include
o Presence at the time of the search
o Possession or control of the property or place searched
55
o
o
o
o
o
Ownership of the property or place
Historical use of the property or item
The ability to regulate access
The existence of a subjective expectation of privacy
The objective reasonableness of the expectation
If an accused person establishes a reasonable expectation of privacy, the inquiry must
proceed to the second stage, to determine whether the search was conducted in a
reasonable manner
 The intrusion of the privacy rights of a third person may be relevant in the second stage
of this analysis
o The invasion of third party right, however, is not determinative of the
unreasonableness of the search
The accused had no privacy interest in the goods seized as he had denied that the drugs were his
 Furthermore, taking all the circumstances of the case into account, he demonstrated no
expectation of privacy in his girlfriend's apartment, which was the only other relevant
privacy interest
o His girlfriend described him as "just a visitor"
o He only had a few personal belongings in the apartment and did not contribute to
rent
o While he had keys to the apartment, he lacked the authority to regulate access to
the premises
The police conduct did not affect a personal right of the accused
 A claim for relief under s. 24(2) of the Charter can only be made by the person
whose Charter rights have been infringed
o Like all Charter rights, s. 8 is a personal right. It protects people and not
places
The reasonable expectation of privacy concept has worked well in Canada
 It has proved to be reasonable, flexible and viable, and should not be abandoned in favour
of the discredited rule of automatic standing
La Forest J
While concurring with the majority in the result, disagreement with their reasons was expressed
on the ground that their effect was to diminish drastically the public's interest in being left alone,
guaranteed by s. 8 of the Charter, in a manner inconsistent with previous statements of this Court
 As I see it, the protection accorded by s. 8 is not in its terms limited to searches of
premises over which an accused has a personal right to privacy in the sense of some
direct control or property
 Rather the provision is intended to afford protection to all of us to be secure against
intrusion by the state or its agents by unreasonable searches or seizures, and is not solely
for the protection of criminals
 It is important for everyone, not only an accused, that the police or other state agents do
56
not break into private premises without warrant.
R. v. Tessling, [2004] 3 SCR 432
Facts
 The RCMP, relying on confidential information from two sources, was investigating
whether the accused was involved in a marijuana growing operation
 The RCMP used an airplane equipped with a Forward Looking Infra-Red (“FLIR”)
camera to overfly properties owned by the accused without first obtaining judicial
warrant. FLIR technology records images of thermal energy or heat radiating from a
building
o The police had contacted hydro to determine whether there was an unusual use of
electricity in the property, but that wasn’t the case
 The RCMP were able to obtain a search warrant for the accused’s home based on the
results of the FLIR image coupled with information supplied by two informants that
pointed to the fact that he accused was growing and trafficking marijuana
 In the house, the RCMP found a large quantity of marijuana and several guns. The
accused was charged with a variety of drug and weapons offences
 At trial, he unsuccessfully argued that the FLIR overflight was a violation of his right to
be free from unreasonable search and seizure guaranteed by s. 8 of the Charter, and was
convicted
 The Court of Appeal set aside the convictions
Issue: Does the use of Forward Looking Infra-Red violate the right to unreasonable search and
seizure?
Holding: No; the appeal was allowed
Binnie J
The freedom from unreasonable search and seizure is fundamental to the relationship
between the state and the citizen
 Few things are as important to our way of life as the amount of power allowed the police
to invade our home, privacy and even bodily integrity
 S. 8 of the Charter creates for “everyone” certain areas of personal autonomy where the
state, including the police, cannot trespass
 At the same time, social and economic life creates competing demands; the community
wants privacy but it also insists on protection
 Thus s. 8 of the Charter accepts the validity of reasonable searches and seizures.
The difficult issue is where the “reasonableness” line should be drawn
 The distinction between informational and territorial privacy is of assistance in the
current factual situation
o Whereas the CA treated the FLIR imaging as equivalent to a search of the home,
it is more accurately characterized as an external surveillance
o FLIR is not equivalent to entry but provides information
o Everything shown in the FLIR photograph exists on the external surfaces of the
building and, in that sense, FLIR records only information exposed to the
public
57
o FLIR heat profile did not expose any intimate details of the accused’s lifestyle or
part of his core biographical data
When one considers the “totality of the circumstances”, the use of FLIR technology did not
intrude on the reasonable sphere of privacy of the accused
 Patterns of heat distribution on the external surfaces of a house are not a type of
information in which, objectively speaking, the accused had a reasonable expectation of
privacy
Comments
 FLIR technology did not reveal specific information about the lifestyle and personal
choices of the individual, so there is no expectation of privacy
 In Tessling the SCC distanced itself from the US Supreme Court in Kyllo v. United States
(2001), where FLIR imaging was unconstitutional
 The SCC does express two caveats:
o FLIR information alone is insufficient to obtain a search warrant
o If FLIR technology gets better, the constitutional issues will be reconsidered
R. v. Kang-Brown, 2008 SCC 18
Facts
 An RCMP officer involved in a special operation designed to detect drug couriers at bus
stations spotted an individual who seemed suspicious
 The officer eventually approached the accused, identified himself and told him that he
was not in any trouble and was free to go at any time
 He asked him if he was carrying narcotics and the accused said no
 The officer then asked to look in the accused’s bag, but when went to touch the bag the
accused pulled it away, looking nervous
 At that point, the officer signaled another officer with a sniffer dog to approach and the
dog sat down, indicating the presence of drugs in the bag
 The accused was arrested for possession of and/or trafficking in drugs, he was searched
and drugs were found on his person and in his bag
 The trial judge found that the accused was neither arbitrarily detained nor unlawfully
searched and entered a conviction
o She held that the odours from the bag, which emanated freely in a public
transportation facility, did not constitute information in which the accused had a
reasonable expectation of privacy and that s. 8 of the Charter
 The Court of Appeal upheld the conviction
Issue: Was the sniffer dog’s search lawful?
Holding: No; the appeal was allowed
Lebel J (majority)
The use of sniffer dogs constitutes a search within the meaning of s. 8 of the Charter, and
absent justified authority for such a search in a statute or at common law, the sniffer-dog search
breached s. 8
It is undisputed that the search was not specifically authorized by statute
58
In determining whether the police were authorized at common law to conduct the search in
fulfilment of their general duty to investigate crime, the threshold for the exercise of police
powers should not be lowered to one of “reasonable suspicion” since, to do so, would
impair the important safeguards found in s. 8 against unjustified state intrusion
 The existing and well-established STANDARD OF “REASONABLE AND
PROBABLE GROUNDS” should be applied
 In this case, the search did not meet this standard
Any perceived gap in the present state of the law on police investigative powers arising from the
use of sniffer dogs is a matter better left for Parliament
 When rights and interests as fundamental as personal privacy and autonomy are at stake,
the constitutional role of the Court suggests that the creation of a new and more
intrusive power of search and seizure should be left to Parliament to set up and
justify under a proper statutory framework
Binnie J
A “sniff” amounts to a s. 8 search because of the significance and quality of the information
obtained about concealed contents, whether such contents are in a suspect’s belongings or carried
on his or her person
Because of the minimal intrusion, contraband-specific nature and pinpoint accuracy of a sniff, a
proper balance between an individual’s s. 8 rights and the reasonable demands of law
enforcement would be struck by permitting such “sniff” searches on a “REASONABLE
SUSPICION” standard without requiring prior judicial authorization
Sniffer dogs have been in common use by police forces in Canada for the last 30 years or more.
If the police have lawful authority to use sniffer dogs only when they already have reasonable
grounds to believe contraband is present, sniffer dogs would be superfluous and unnecessary
The issue raised by this appeal is not the existence of a police power to investigate crime using
sniffer dogs, but the extent to which the use of such animals is permitted by s. 8 of the Charter
 It is emphatically the duty of the courts, not Parliament, to resolve the issue of Charter
compliance
The “reasonable suspicion” standard is not a new juridical standard called into existence for the
purposes of this case
 Suspicion is an expectation that the targeted individual is possibly engaged in some
criminal activity
 A “reasonable” suspicion means something more than a mere suspicion and
something less than a belief based upon reasonable and probable grounds
 Because sniffer-dog searches are conducted without prior judicial authorization, the afterthe-fact judicial scrutiny of the grounds for the alleged “reasonable suspicion” must be
rigorous
59
It is common ground that what occurred at the bus station was a warrantless search, and therefore
presumptively unreasonable
 However, had the dog-sniff search been based on reasonable suspicion, the dog’s positive
alert would have given the police the grounds to proceed on the spot with a warrantless
search
The sniff in this case was an unreasonable search since the RCMP officer did not have grounds
for reasonable suspicion at the time the dog was called
 Given the dog alerts to the odour of narcotics, not to their actual presence, the arrest of
the accused in this case was premature
 The police should first have confirmed the presence of narcotics by a hand search of the
bag
 If reasonable cause had existed, and given the positive alert and the dog’s history of
accuracy, the RCMP would have been entitled to perform such a verification search on
the spot and without prior judicial authorization
Deschamps J (dissenting)
The use of a sniffer dog to check the accused’s bag in a public bus terminal on the basis of a
reasonable suspicion that evidence of an offence would be discovered was proper and did
not constitute an unreasonable search or seizure
The search in this case was justified on the basis of the reasonable suspicion standard
 This standard can be applicable only where there are circumstances that serve as
safeguards against unreasonable intrusions on privacy and ensure a balance that affords
proper protection
A reasonable suspicion standard may be sufficient where the investigative technique is
relatively non-intrusive and the expectation of privacy is not high
 To determine whether the reasonable suspicion standard is met in a given case, the
totality of the circumstances must be considered
 The sniffer dog’s intrusion on the accused’s right to informational privacy suggests that
the accused had an objectively reasonable expectation of privacy, yet nevertheless, the
accused’s objectively reasonable expectation of privacy in this case was not high
 The search in the present case was conducted in a public place and it was only minimally
intrusive
The police did not use sniffer dogs randomly at the bus terminal where the appellant was
searched
 They had obtained the permission of the terminal’s management to do so and it is after
having observed the appellant in this public place that the police determined that he was
a person of interest
Bastarache J (dissenting)
Although the accused did have a reasonable expectation of privacy in his luggage, it was
60
significantly reduced owing to the location at which the search occurred
 In a bus depot, a passenger is voluntarily using the terminal to access a public mode of
transportation, and he or she is aware that the state has an interest in ensuring that that
transportation system is both secure and not being used to further criminal activity
The requisite balancing of individual rights with the state interest in preventing and investigating
crime supports a finding that a sniff search of luggage using a police dog will not be in violation
of s. 8 of the Charter where the police are acting on a reasonable suspicion about the committal
of a crime
 The reduced expectation of privacy at public terminals, the minimal intrusion
caused by the search itself, and the effectiveness of sniffer-dog searches all support a
standard of “reasonable suspicion”.
Lecture notes
 How does Kang-Brown fair in light of Tessling
o The dogs identify particularly whether you have drugs or not, which could be part
of your lifestyle, while FLIR technology only indicates heat consumption
patterns, which in itself is not an offence
 Binnie J states that if you required reasonable and probable grounds to carry out the
sniffer search, the sniffer dogs become redundant
o His standard instead in the reasonable suspicion, which in this case was not met
given the attitude of the defendant
 Lebel J would leave the creation of a new police power to Parliament
o Binnie J’s answer to this is that since police powers have been extended under
other circumstances, not doing it do here could be inconsistent and courts would
become a space where police powers are extended depending on the feeling of the
court
R. v. A.M., 2008 SCC 19
Facts
 The police accepted a long-standing invitation by the principal of a high school to bring
sniffer dogs into the school to search for drugs
o The police had no knowledge that drugs were present in the school and would not
have been able to obtain a warrant to search the school
 The sniffer dog reacted to one of the unattended backpacks lined up against a wall
o Without obtaining a warrant, the police opened the backpack and found illicit
drugs
 The student who owned the backpack was charged with possession of cannabis
marihuana and psilocybin for the purpose of trafficking
 At trial, the accused brought an application for exclusion of the evidence, arguing that his
rights under s. 8 of the Charter had been violated
 The trial judge allowed the application, finding two unreasonable searches: the search
conducted with the sniffer dog and the search of the backpack, and he ordered an
acquittal
 The CA upheld the acquittal
Issue: Was the sniffer dog’s search lawful?
Holding: No; appeal dismissed
61
Lebel J (majority)
Since there was no authority in the statutes or at common law for the sniffer-dog search in this
case, the search violated s. 8 of the Charter
 Students are entitled to privacy in a school environment
 For the reasons stated in R. v. Kang-Brown, courts should not attempt to craft a legal
framework of general application for the use of sniffer dogs in schools
Binnie J
The dog sniff amounts to a search within s. 8 of the Charter
 The information provided when the dog is trained to alert to the presence of controlled
drugs permits inferences about the precise contents of the source that are of interest to the
police
 The subject matter of the sniff is not public air space, it is the concealed contents of
a backpack, which is a personal
 Teenagers may have little expectation of privacy from the searching eyes and fingers of
their parents, but they expect the contents of their backpacks not to be open to the random
and speculative scrutiny of the police
Although a warrantless sniffer-dog search is available where reasonable suspicion is
demonstrated, the sniffer-dog search of the students’ belongings in this case violated their
Charter rights under s. 8
 The dog-sniff search was unreasonably undertaken because there was no proper
justification
While the sniffer-dog search may have been seen by the police as an efficient use of their
resources, and by the principal of the school as an efficient way to advance a zero-tolerance
policy, these objectives were achieved at the expense of the privacy interest (and constitutional
rights) of every student in the school
 The Charter weighs other values, including privacy, against an appetite for police
efficiency
Where there are grounds of reasonable suspicion, the police should not have to take their
suspicions to a judicial official for prior authorization to use the dogs in an area where the
police are already lawfully present
 All “searches” do not have the same invasive and disruptive quality and prior judicial
authorization is not a universal condition precedent to any and all police actions
characterized as “searches” given that the touchstone of s. 8 is reasonableness
 Account must be taken in s. 8 matters of all the relevant circumstances including the
minimal intrusion, contraband-specific nature and high accuracy rate of a fly-by sniff
The warrantless search is, of course, presumptively unreasonable
 If the sniff is conducted on the basis of reasonable suspicion and discloses the presence of
illegal drugs, the police may confirm the accuracy of that information with a physical
search, again without prior judicial authorization
62
The importance of proper tests and records of particular dogs will be an important element in
establishing the reasonableness of a particular sniffer-dog search
 An important concern for the court is therefore the number of false positives
 Moreover, the sniff does not disclose the presence of drugs, but it discloses the presence
of an odour that indicates either the drugs are present or may have been present
In sniffer-dog situations, the police are generally required to take quick action guided by on-thespot observations
 In circumstances where this generally occurs, it is not feasible to subject the “sniffer
dog’s” sniff to prior judicial authorization
 Both the subject and his suspicious belongings would be long gone before the paperwork
could be done
The youth court judge here noted that the evidence of the drugs existed independently of the
Charter violation and that its admission, being non-conscriptive, would not affect trial fairness
 However, given the fact that the speculative sweep in this case appears to be the standard
practice of municipal police forces in Ontario, the youth court judge concluded that “the
Charter must not be seen as something to be swept away in the interests of expediency
 The court will not interfere with balance of competing values struck by the youth court
judge or his exclusion of the evidence
R. v. Wong, [1990] 3 SCR 36
Facts
 At issue was the admissibility of police video surveillance evidence of a large-scale
illegal gambling operation in a hotel
 The Ontario CA held that the protection under s.8 of the Charter was not available as
there was no reasonable expectation of privacy, given that 30 to 35 people had been
invited to gamble illegally in the room
 The SCC held that this reasoning could not be reconciled with Duarte
Issue: Is the reasonable expectation of privacy diminished by the fact that someone is committing
a crime?
Holding: No
La Forest J
The question should NOT be framed in terms of whether people engaging in illegal
activities in a hotel room have a reasonable expectation of privacy, but whether in our
society, people who retire to a hotel room and close the door behind them have a
reasonable expectation of privacy
 Otherwise, no only those engaging in criminal activities will risk warrantless
surveillances, but also all members of society renting rooms in a hotel
S. 8 of the Charter affords us a protection against warrantless video surveillances in a hotel,
just as these would be unlawful in our own homes
 The very reason we rent such rooms is to obtain a private enclave
63
[La Forest J also held that it was not appropriate for courts to authorize video surveillance]
Part IV.1 of the Code is designed to set strict limits on the ability of the agents of the state to
intercept private oral communications
 Courts would be forgetting their role as guardians of our fundamental liberties if they
were to usurp the role of Parliament and purport to give their sanction to video
surveillance by adapting for that purpose a code of procedure dealing with an altogether
different surveillance technology
It is for Parliament, and Parliament alone, to set out the conditions under which law enforcement
agencies may employ video surveillance technology in their fight against crime
 Moreover, the same holds true for any other technology which the progress of science
places at the disposal of the state in the years to come
Lecture by Sarah Henningsson - Provincial prosecutor
Fact pattern
 Flier goes out to Montreal police which has the picture of a black individual in his midtwenties who is wanted for home invasion, and is likely to be seen around the Lachine
area
 An officer who is on patrol around Lachine sees someone who fits the description of the
person in the flier
o In his mind he is the person
 He stops the person and asks him to identify himself
 Verifications are then made by the officer and he finds out that a warrant is out for his
arrest, but as he tries to arrest him he runs away
 Later on another officer sees a person who fits the same physical description riding a bike
o He asks him his name and he says the same last name as the other person
o He then grabs him by the arm and tells him that he is under arrest
o As soon as he is arrested he says that they have the wrong person, and that the
person they are looking for is his brother
o He does not have any identification, however
 He arrests him anyway because he feels that he has reasonable and probable grounds and
in his mind he is the person he was looking for
o He searches him as incidental to arrest and finds crack cocaine and $ 280 cash and
a cell phone
o He is also arrested for possession for the purpose of trafficking
 At the police station they find that he is in fact the brother of the person they were
originally looking for
For the arrest to be lawful the officer had to believe that this person had committed or was about
to commit a crime
 There is also an objective component based on information that would make the arrest
lawful on reasonable and probable grounds
64
The prosecutor argued that there were grounds for arrest and that the search should not be
excluded
 The judge found that the arrest was illegal because although there were subjective
grounds for the arrest, they were not objectively justifiable
o The police officer should have gone back to his car and made verification of the
information
o Is the judge, however, imposing a standard of certainty for arrest, which in turn is
contrary to Storrey?
Even if the search was illegal, the evidence should not be excluded according to 24(2)
 The argument here would be that the rocks would have been found anyway
Arrest
The arrest is a critical moment in the balance of power between the citizen and the State
 Across the spectrum of State intrusion on the individual’s liberty and privacy,
arrest is at the end as the most intrusive
o For example, search incident to arrest allows more search powers than any other
event in the common law
o Police can use as much force as necessary in effecting an arrest
As per R. v. Whitfield, one of the earliest sources of the description of arrest
 Arrest is the actual seizure or touching of a person’s body or pronouncing the words that
you are under arrest
An arrest often involves a prolonged interference on someone’s freedom of movement and an
intrusion into a person’s privacy
The powers of arrest are guided by a complex statutory regime
Generally speaking, there is NOT an obligation to obtain arrest warrants, as is the case of
searches
Citizen’s Power of Arrest
The power of citizen arrest is expressed in the Code and carefully circumscribed in s.494
s. 494 – Criminal Code
Arrest without warrant by any person
(1) Any one may arrest without warrant
(a) a person whom he finds committing an indictable offence; or
(b) a person who, on reasonable grounds, he believes
65
(i) has committed a criminal offence, and
(ii) is escaping from and freshly pursued by persons who have lawful authority
to arrest that person.
Arrest by owner, etc., of property
(2) Any one who is
(a) the owner or a person in lawful possession of property, or
(b) a person authorized by the owner or by a person in lawful possession of property,
may arrest without warrant a person whom he finds committing a criminal offence on or in
relation to that property.
Delivery to peace officer
(3) Any one other than a peace officer who arrests a person without warrant shall forthwith
deliver the person to a peace officer.
 Crown election offences are treated as indictable for the purpose of the law of arrest by
virtue of interpretation
 Paragraph 2 delineates the special power of arrest of property owners and possessors
Power of Peace Officers
Peace officers, as described in s.2 of the Code, have more extensive powers
s. 31 – Criminal Code
Arrest for breach of peace
(1) Every peace officer who witnesses a breach of the peace and every one who lawfully assists
the peace officer is justified in arresting any person whom he finds committing the breach of
the peace or who, on reasonable grounds, he believes is about to join in or renew the breach of
the peace.
Giving person in charge
(2) Every peace officer is justified in receiving into custody any person who is given into his
charge as having been a party to a breach of the peace by one who has, or who on reasonable
grounds the peace officer believes has, witnessed the breach of the peace.
 Notice that there is no restriction to an indictable offence here
s. 495 – Criminal Code
Arrest without warrant by peace officer
(1) A peace officer may arrest without warrant
(a) a person who has committed an indictable offence or who, on reasonable grounds, he
66
believes has committed or is about to commit an indictable offence;
(b) a person whom he finds committing a criminal offence; or
(c) a person in respect of whom he has reasonable grounds to believe that a warrant of
arrest or committal, in any form set out in Part XXVIII in relation thereto, is in force
within the territorial jurisdiction in which the person is found.
Limitation
(2) A peace officer shall not arrest a person without warrant for
(a) an indictable offence mentioned in section 553,
(b) an offence for which the person may be prosecuted by indictment or for which he is
punishable on summary conviction, or
(c) an offence punishable on summary conviction,
in any case where
(d) he believes on reasonable grounds that the public interest, having regard to all the
circumstances including the need to
(i) establish the identity of the person,
(ii) secure or preserve evidence of or relating to the offence, or
(iii) prevent the continuation or repetition of the offence or the
commission of another offence,
may be satisfied without so arresting the person, and
(e) he has no reasonable grounds to believe that, if he does not so arrest the person, the
person will fail to attend court in order to be dealt with according to law.
Consequences of arrest without warrant
(3) Notwithstanding subsection (2), a peace officer acting under subsection (1) is deemed to be
acting lawfully and in the execution of his duty for the purposes of
(a) any proceedings under this or any other Act of Parliament; and
(b) any other proceedings, unless in any such proceedings it is alleged and established
by the person making the allegation that the peace officer did not comply with the
requirements of subsection (2).
 The limitation set out in subsection 2 (mainly to summary convictions) is contingent on
the circumstances described in subsection 2(e)
What is the appropriate standard for a lawful arrest?
R. v. Storrey, [1990] 1 SCR 241
Issue: Was the arrest unlawful and arbitrary?
Holding: No; appeal dismissed
Cory J
Section 495(1) makes it clear that the police were required to have reasonable and
67
probable grounds that the appellant had committed the offence of aggravated assault
 In the case of an arrest made without a warrant, it is even more important for the police
to demonstrate that they have those same reasonable and probable grounds upon which
they base the arrest
The importance of this requirement to citizens of a democracy is self-evident, so as to not fall
prey to the abuses and excesses of a police state; yet society also needs protection from crime
 This need requires that there be a reasonable balance between the individual's right to
liberty and the need for society to be protected from crime
The Criminal Code then requires that an arresting officer must subjectively have reasonable
and probable grounds on which to base the arrest, but this is not sufficient
 It must be objectively established that those reasonable and probable grounds did
in fact exist
 That is to say a reasonable person, standing in the shoes of the police officer, would
have believed that reasonable and probable grounds existed to make the arrest
 On the other hand, the police need not demonstrate anything more than reasonable
and probable grounds, specifically they are not required to establish a prima facie
case for conviction before making the arrest.
Comments
 This case explains what reasonable grounds is as per s.495(1)(a)
 The court in Storrey appeared to be unaware that Parliament had deleted the words “and
probable” from what is now s.495
 Storrey did also say that a lawful arrest is not arbitrary
The peace officer’s power to arrest anyone found committing a criminal offence is s.495(1)(b)
was interpreted in R. v. Biron to mean “apparently” committing and offence
R. v. Biron, [1976] 2 SCR 56
Facts
 The Montreal police made an authorized raid on a bar in search of illegal firearms and
liquor
 Biron, who had been drinking, was at the bar while the raid was taking place
o He refused to co-operate with the police, verbally abusing them and later
refusing to his arrest
 He was charged with creating a disturbance in a public place by shouting and also for
resisting a peace officer
 The accused was convicted of both offences but a trial de novo acquitted him for the
offence of "creating a disturbance by shouting" on the ground that there was no evidence
he had been shouting as was alleged in the information
 He was later acquitted by the Quebec CA on the charge of resisting a police officer
 His claim is that since he was acquitted of the charge of creating disturbance, the officer
had no lawful reason to arrest him since he was committing no criminal offence, and as
such he could not be guilty from resisting the police officer
Issue: What does it entail to be found committing a criminal offence as per s.495(1)(b)?
68
Holding: To be “apparently” committing an offence; appeal allowed
Martland J
Paragraph (a) of s. 450(1) permits a peace officer to arrest without a warrant under the situation
where an indictable offence has already been committed or is expected to be committed
 The peace officer is not present at its commission, so she may have to rely upon
information received from others
 The paragraph therefore enables her to act on her belief, if based on reasonable and
probable grounds
Paragraph (b) applies in relation to ANY criminal offence and it deals with the situation
in which the peace officer herself finds an offence being committed
 Her power to arrest is based upon her own observation
 Because it is based on her own discovery of an offence actually being committed
there is no reason to refer to a belief based upon reasonable and probable grounds
 The validity of an arrest under this paragraph must be determined in relation to the
circumstances which were apparent to the peace officer at the time the arrest was
made
The words "committing a criminal offence" are not to be construed in the manner indicated in
the Pritchard case, whereby it is impossible to say that an offence is committed until the party
arrested has been found guilty by the courts
 Instead, the wording used in paragraph (b) means that the power to arrest without a
warrant is given where the peace officer herself finds a situation in which a person
is APPARENTLY committing an offence
In the present case Constable Maisonneuve observed an apparent offence being committed by
Biron
Laskin CJ (dissesnting)
It is astonishing that a provision concerned with a constable's criminal or other responsibility,
and which immunizes her in specified circumstances in respect of an arrest that she has made,
should become the vehicle for providing a basis upon which an accused may herself be
convicted of resisting the arrest
 [This is made in relation to s.25 of the Code which dictates when a police officer is
justified in doing what she is authorized to do]
If the word "apparently" is to be read into s. 450(1)(b) [now s.495(1)(b)], logical consistency
demand that the word be read into s. 449(1)(a) [s.494(1)(a)] which empowers any person to
arrest without warrant a person whom she "finds committing" an indictable offence
 On grounds of context when s. 449(1)(a) is read with s. 449(1)(b), the former could not
possibly embrace arrest without warrant on apparency or on reasonable and probable
grounds
While a constable's burden is a heavy when she has to make an on-the-spot decision as to an
69
arrest, we cannot go on a guessing expedition out of regret for an innocent mistake or a wrongheaded assessment
 The social and legal, and political, principle upon which our criminal law is based,
namely, the right of an individual to be left alone, to be free of private or public
restraint, save as the law provides otherwise
The position as it relates to resistance to unlawful arrest was established at common law as early
as 1709
 Our law has not, as I understand it, deprived the citizen of her right to resist unlawful
arrest
 Her resistance may be at her own risk if the arrest proves to be lawful, but too
must the police officer accept the risk of having effected a lawful arrest
Lecture Notes
 Requiring police officers to have reasonable and probable grounds that the person
committed the crime would entail asking them to be judges and determined whether the
person has committed the crime or not
 Laskin CJ (dissenting) feels that loosening the standard for citizen’s powers of arrest
could be problematic
o It can also be said, that the bigger the scope if for s.495(1), the smaller it will be
for s.495(2)
o Loosening the standard to apparently committing an offence, would also affect
all other sections which are based on this standard
In Roberge v. R .(1983), Lamer J for the SCC interpreted Biron to mean that apparently
committing was the same as having reasonable and probable grounds for believing an offence
has been committed
 Because of this, a requirement of reasonable and probable grounds is now read into
all forms of arrest powers
Constitutional Minimum Standards (Charter ss. 9 and 10(a))
There are also constitutional dimensions to the law of arrest
s. 9 – Canadian Charter of Rights and Freedoms
Detention or imprisonment
Everyone has the right not to be arbitrarily detained or imprisoned.
s. 10 – Canadian Charter of Rights and Freedoms
Arrest or detention
Everyone has the right on arrest or detention
a) to be informed promptly of the reasons therefor;
b) to retain and instruct counsel without delay and to be informed of that right; and
70
c) to have the validity of the detention determined by way of habeas corpus and to be
released if the detention is not lawful.
The SCC has not given a clear pronouncement of what an arbitrary detention is
 There is no equivalent of Hunter that establishes what an arbitrary detention entails
The Ontario CA held in Duguay that an unlawful arrest is not necessarily arbitrary under s. 9
of the Charter
R. v. Duguay (1985), 45 CR (3d) 140
Facts
 Following a break and enter at a residence, police were contacted and two experienced
detectives attended at the victim's home
 The evidence indicated that prior to the victim leaving his home that night, he had
noticed three young men drinking beer in the backyard of a neighbour's home
 The owner of the neighbouring house, after hearing a description of the three young
men, identified one of them and made arrangements for the three to attend back at the
premises
 When the three men arrived, they were arrested and put in a police cruiser
 As a result of questioning by one of the detectives, the police were directed to some of
the stolen goods and the three accused all made inculpatory statements
 The trial judge found that the detectives could not honestly believe that they had
reasonable and probable grounds to arrest the accused, and as such they had been
subjected to arbitrary detention in violation of s. 9 of the Charter and he excluded all the
evidence obtained following the arrest pursuant to s. 24(2) of the Charter
 The accused were acquitted
Issue: Is an unlawful arrest necessarily arbitrary and contrary to s.9 of the Charter?
Holding: No; appeal dismissed
MacKinnon ACJO
It cannot be that every unlawful arrest necessarily falls within the words "arbitrarily
detained"
The person making the arrest may honestly, though mistakenly, believe that reasonable and
probable grounds for the arrest exist and there may be some basis for that belief; in those
circumstances the arrest, though subsequently found to be unlawful, could not be said to
be capricious or arbitrary
On the other hand, the entire absence of reasonable and probable grounds for the arrest
could support an inference that no reasonable person could have genuinely believed that
such grounds existed
 In such cases, the conclusion would be that the person arrested was arbitrarily
detained
Between these two ends of the spectrum, the issue of whether an accused was arbitrarily
detained will depend on two considerations
71
1. The particular facts of the case
2. The view taken by the court with respect to the extent of the departure from the standard
of reasonable and probable grounds and the honesty of the belief and basis for the belief
in the existence of reasonable and probable grounds on the part of the person making the
arrest
On the facts of the present case, the arrest or detention was arbitrary, being for quite an
improper purpose
 They had neither grounds nor an honest belief that they had the necessary grounds
Zuber JA
The detention that follows an arrest based on something less than reasonable and probable
grounds is not necessarily arbitrary
 The arrest in this case was neither capricious nor random
 I have very serious doubts as to these two conclusions reached by the trial judge but
since they are bound up with issues of fact and since the appeal by the Crown is
dependent upon an issue of law alone, I am content to accept his conclusions on these
two issues and rest my judgment entirely on the third and last issue, i.e., the propriety of
excluding the evidence
Lecture Notes
 The case seems to constitutionalize the Storrey standard of reasonable grounds for
detention, but with some room for departure
 The court doesn’t state whether this is the standard to be used in all types of detention
In contrast to the position advanced in Duguay, the Saskatchewan Court held that an unlawful
detention is necessarily arbitrary and contrary to s.9 of the Charter; see R. v. Iron (1987)
Reasons for Arrest
Common law requirements respecting the duty to provide the reasons for an arrest were laid
down by the House of Lords in Christie v. Leachinsky (1947)
 The requirement does not means that technical an precise language must be used, but the
matters is a matter of substance
This pronouncement was followed by Canadian courts until a majority in the SCC held that the
duty was exhaustively codified in s.29(2) of the Criminal Code; see Gamracy v. R. (1974)
Section 10(a) of the Charter now establishes the right of everyone upon arrest or detention
to be informed promptly of the reasons therefor
R. v. Evans, [1991] 1 SCR 869
Facts
 The accused, a mentally challenged youth, was convicted of first degree murder in the
72
brutal killings of two women
 Initially, the police thought his brother had committed the murders and arrested the
appellant on a marijuana charge in the hope that he would be able to provide evidence
against his brother
 During the course of the interrogation that followed, Evans became the prime suspect in
the two murders
o The police did not formally advise the accused that he was then being detained
for murder
o The police investigation was aggressive and marked by their lying about finding
the appellant's fingerprint at one of the murder scenes
o Eventually incriminating statements were obtained from the appellant, and these
statements formed virtually the entire basis of his conviction for the two murders
 An appeal to the Court of Appeal was dismissed
 The accused continued to appeal that his rights under ss. 7, 10(a) and 10(b) of the
Charter were violated so that the resultant confessions should have been excluded
pursuant to s. 24(2) of the Charter
Issue: Was the accused properly informed of the reasons for his detention?
Holding: Yes; the appeal was nonetheless allowed for other reasons
McLachlin J
The right to be promptly advised of the reason for one's detention embodied in s. 10(a) of
the Charter is founded on the notion that one is not obliged to submit to an arrest if one
does not know the reasons for it
 A second aspect of the right lies in its role as an adjunct to the right to counsel conferred
by s. 10(b) of the Charter
When considering whether there has been a breach of s. 10(a), it is the substance of what
the accused can REASONABLY be supposed to have understood, rather than the
formalism of the precise words used, which must govern
 The question is whether what the accused was told, viewed reasonably in all the
circumstances of the case, was sufficient to permit him to make a reasonable decision to
decline to submit to arrest, or alternatively, to undermine his right to counsel under s.
10(b)
The accused’s response to the officer's statement that, while he had originally been arrested on
marijuana charges, things had now taken "quite a change", indicates that the accused was aware
that the focus of the questioning had changed and that he was then being questioned with
respect to the killings
 It might, therefore, be argued that he was given the facts relevant to determining
whether he should continue to submit to the detention
 Nor can any failure to comply with s. 10(b) be attributed to failure to advise the accused
of the reasons why his detention and questioning was continuing
 These considerations suggest that the requirements of s. 10(a) were met in the case at
bar
Sopinka J (dissenting on the issues of s.10(a) but concurring on the result)
73
The right to be informed of the true grounds for the arrest or detention is firmly rooted in the
common law which required that the detainee be informed in sufficient detail that he or she
"knows in substance the reason why it is claimed that this restraint should be imposed"
(Christie v. Leachinsky)
 The purpose of communicating this information to the accused is to enable the person
under arrest or detention to immediately undertake his or her defence, including a
decision as to what response, if any, to make to the accusation
In this case, the arresting officers were forewarned that they were dealing with a person of
subnormal intelligence, so it was incumbent on them to be scrupulous in ensuring that his
rights were respected
 Instead, they concocted a ground for the arrest in order to question him about the
involvement of his brother in the murders
 Having explicitly advised the appellant that he was in jeopardy for trafficking in
narcotics, the arresting officers were obliged to disabuse him of this false information
before seeking to elicit incriminatory evidence from him
The accused, whose mental development was equated to that of a 14-year-old, should not have
been required to deduce from the content of questions that the initial explicit reason for his
arrest had shifted to a far more serious ground
Entry into Premises
The law is particularly vague as to whether authorities have the power to enter premises to arrest
The Ouimet Report identified two distinct types of power
 The police officer can enter premises without a warrant to prevent the commission of
an offence which could cause immediate and serious injury to a person, based on a belief
grounded in reasonable and probable grounds
 The police officer can also enter premises without a warrant, by force if necessary, to
effect the arrest of a person who has been found committing a serious crime, and
who is being freshly pursued and seeks refuge in such premises
Before 1997, there was no obligation for the police to subject their intentions to prior judicial
scrutiny before arrest
 Feeney changed this when the arrest was carried out in the defendants premises
R. v. Feeney, [1997] 2 SCR 13
Facts
 An old man was found beaten to death in his home
 The police learned that the victim’s truck had been found abandoned in a ditch near the
crime scene
 A resident of the area had seen the accused walking away from the truck and carrying
something in his hand, while another witness told the police that the accused had stolen
74
a vehicle and crashed in the same location
 The police went to the house where the accused was staying and having received no
answer at the door, they entered, roused the accused and took him to the front of the
trailer for better lighting
o The police arrested him after seeing blood on his shirt
 Following a caution with respect to the right to counsel but not the right to immediate
counsel, the police asked the accused a couple of questions which he answered
 The accused's shirt was seized and he was taken to the police detachment where, before
the accused had consulted with counsel, further statements were taken
 The accused was convicted of second degree murder and his appeal to the British
Columbia CA was unanimously dismissed
o The CA was preoccupied that the blood in the shirt would be destroyed if the
arrest had not been carried
Issue: Was the police allowed to enter the accused’s premises to effect the arrest?
Holding: No; appeal allowed
Sopinka J
Under the pre-Charter common law, a warrantless arrest following a forced entry into
private premises is legal if
a) The officer has reasonable grounds to believe that the person sought is within the
premises
b) Proper announcement is made
c) The officer believes reasonable grounds for the arrest exist
d) Objectively speaking, reasonable and probable grounds for the arrest exist
 Except in exigent circumstances, police should give notice of presence by knocking or
ringing the doorbell, give notice of authority by identifying themselves as law
enforcement police officers and give notice of purpose by stating a lawful reason for
entry
o Before forcing entry, police should, at minimum, request admission and have
admission denied
Neither the subjective nor objective requirements for arrest were met, rendering the
arrest unlawful
 The arresting officer did not believe he had reasonable grounds to arrest prior to the
forcible entry, until after he saw the blood-stained shirt
 A reasonable person, standing in the shoes of the officer, would not have believed that
reasonable and probable grounds to make the arrest existed
o Any finding that the subjective test is not met will generally imply that the
objective test is not met, unless the officer is to be considered to have an
unreasonably high standard
The Landry test for warrantless searches, essentially a balancing between aiding the police in
their protection of society on the one hand and the privacy interests of individuals in their
dwellings on the other, no longer applies
 It must be adjusted to comport with Charter values which, notwithstanding the high
value on the security and privacy of the home at common law, significantly increase the
75

importance of the legal status of the privacy of the home
In general, the privacy interest now outweighs the interest of the police and
WARRANTLESS ARRESTS IN DWELLING HOUSES ARE PROHIBITED
The case of Hunter was used in the analysis
 Generally a warrant is required to make an arrest in a dwelling house, just like
warrantless searches are prima facie unlawful
 Nonetheless, there are exceptions with respect to the unreasonableness of warrantless
searches for things
o A warrantless search will respect s. 8 if authorized by law, and both the law and
the manner in which the search is conducted are reasonable
o In cases of hot pursuit, the privacy interest must give way to the interest of
society in ensuring adequate police protection
An arrest warrant alone is insufficient protection of the suspect's privacy rights
 Privacy rights under the Charter demand that the police, in general, obtain prior judicial
authorization of entry into the dwelling house in order to arrest the person
 If the Code currently fails to provide specifically for a warrant containing such
prior authorization, such a provision should be read in
 While the absence of such a provision could have a profound influence on the common
law power of arrest, its absence cannot defeat a constitutional right of the individual
 Once a procedure to obtain such prior authorization is created, the concern that suspects
may find permanent sanctuary in a dwelling house disappears.
Warrantless arrests in dwelling houses are in general prohibited
 Requiring a warrant prior to arrest avoids the ex post facto analysis of the
reasonableness of an intrusion and invasive arrests without a basis of reasonable and
probable grounds are prevented, rather than remedied after the fact
To summarize, the following requirements must be met before an arrest for an indictable
offence in a private dwelling is legal
1. A warrant must be obtained on the basis of reasonable and probable grounds to arrest
and to believe the person sought is within the premises in question
2. Proper announcement must be made before entering
3. An exception to this rule occurs where there is a case of hot pursuit
o Whether or not there is an exception for exigent circumstances generally has not
been fully addressed by this Court.
The arrest was unlawful both because the requirements for a warrantless arrest under s. 495 of
the Code were not met, and, in any event, the police cannot make warrantless arrests in private
dwellings unless exceptional circumstances exist (which were not present in this case)
o Consequently, the entry into the trailer and the search and seizure of the accused's
clothing violated s. 8 of the Charter
L’Heureux-Dubé J (dissenting on this issues but concurring on the result)
76
There were both subjective and objective grounds for the arrest
 The sole remaining factor to consider is whether a proper announcement was made
before the police entered the premises
o In some cases it would be contrary to common sense to announce the purpose of
entry once it was clear that the person inside was refusing or unable to answer
the request to enter
o We are clearly faced with such a case here, since before entering, the officers
were informed that the appellant was, in all likelihood, sound asleep
It is clear that the power of arrest is a crucial part of law enforcement, and for that reason, it is
unrealistic to suggest that the police can never enter private premises without a warrant for the
purposes of arrest
 Neither can I accept that it is only in circumstances of hot pursuit that the police are
permitted to enter a dwelling house without a warrant for the purpose of arrest
Preventing the removal or destruction of evidence is a legitimate law enforcement concern
which warrants setting aside the strict rules concerning the sanctity of the home
For all of these reasons, exigent circumstances were indeed present in the case at bar
 Where these circumstances exist, the common law authorizing entries onto private
premises constitutes a "reasonable" entry for the purposes of s. 8 of the Charter
Lecture Notes
 Sopinka J felt that the pre-Charter rule was not applicable because of the privacy
interests that were now protected by the Charter
On 1997, Parliament enacted new entry powers (see ss.529-529.3) including an exigent
circumstances exception for warrantless entry (s. 529.3)
s. 529 – Criminal Code
Including authorization to enter in warrant of arrest
(1) A warrant to arrest or apprehend a person issued by a judge or justice under this or any other
Act of Parliament may authorize a peace officer, subject to subsection (2), to enter a dwellinghouse described in the warrant for the purpose of arresting or apprehending the person if the
judge or justice is satisfied by information on oath in writing that there are reasonable grounds
to believe that the person is or will be present in the dwelling-house.
Execution
(2) An authorization to enter a dwelling-house granted under subsection (1) is subject to the
condition that the peace officer may not enter the dwelling-house unless the peace officer has,
immediately before entering the dwelling-house, reasonable grounds to believe that the person
to be arrested or apprehended is present in the dwelling-house.
s. 529.1 – Criminal Code
77
Warrant to enter dwelling-house
A judge or justice may issue a warrant in Form 7.1 authorizing a peace officer to enter a
dwelling-house described in the warrant for the purpose of arresting or apprehending a person
identified or identifiable by the warrant if the judge or justice is satisfied by information on oath
that there are reasonable grounds to believe that the person is or will be present in the dwellinghouse and that
(a) a warrant referred to in this or any other Act of Parliament to arrest or apprehend the
person is in force anywhere in Canada;
(b) grounds exist to arrest the person without warrant under paragraph 495(1)(a) or (b)
or section 672.91; or
(c) grounds exist to arrest or apprehend without warrant the person under an Act of
Parliament, other than this Act.
s. 529.2 – Criminal Code
Reasonable terms and conditions
Subject to section 529.4, the judge or justice shall include in a warrant referred to in section 529
or 529.1 any terms and conditions that the judge or justice considers advisable to ensure that the
entry into the dwelling-house is reasonable in the circumstances.
s. 529.3 – Criminal Code
Authority to enter dwelling without a warrant
(1) Without limiting or restricting any power a peace officer may have to enter a dwellinghouse under this or any other Act or law, the peace officer may enter the dwelling-house for the
purpose of arresting or apprehending a person, without a warrant referred to in section 529 or
529.1 authorizing the entry, if the peace officer has reasonable grounds to believe that the
person is present in the dwelling-house, and the conditions for obtaining a warrant under section
529.1 exist but by reason of exigent circumstances it would be impracticable to obtain a
warrant.
Exigent Circumstances
(2) For the purposes of subsection (1), exigent circumstances include circumstances in which
the peace officer
(a) has reasonable grounds to suspect that entry into the dwelling-house is necessary to
prevent imminent bodily harm or death to any person; or
(b) has reasonable grounds to believe that evidence relating to the commission of an
indictable offence is present in the dwelling-house and that entry into the dwellinghouse is necessary to prevent the imminent loss or imminent destruction of the evidence.
In Godoy, the SCC recognized a common law power to enter premises to investigate a
disconnected 911 call
R. v. Godoy, [1999] 1 SCR 311
78
Facts
 Two police officers received a call from radio dispatch concerning a 911 emergency call
originating from the accused's apartment in which the line had been disconnected before
the caller spoke
 Along with two back-up officers they arrived at the accused's apartment and knocked on
the door
o The accused partially opened the door and when asked if things were all right
inside responded that there was no problem
o One of the officers asked if they could enter the apartment to investigate but the
accused tried to close the door
o The officer prevented him from shutting the door and entered the dwelling
o As soon as they got inside, he heard a woman crying. They accused's common
law wife in their bedroom and the officer observed considerable swelling above
her left eye, which she stated was the result of the accused hitting her
 Based on these observations, the accused was placed under arrest for assaulting his wife
 The trial judge dismissed the charge, holding that the officers' entry into the accused's
apartment was unauthorized and that therefore all subsequent actions of the police,
including the arrest of the accused, were illegal
 The Ontario Court (General Division) allowed the Crown's appeal and ordered a new
trial
 The Court of Appeal upheld that decision
Issue: Was a warrantless entry and ensuing arrest following a disconnected 911 call lawful?
Holding: Yes; appeal dismissed
Lamer CJ
Public policy clearly requires that the police ab initio have the authority to investigate 911
calls, but whether they may enter dwelling houses in the course of such an investigation
depends on the circumstances of each case
The accepted test for evaluating the common law powers and duties of the police was set out in
Waterfield
 If police conduct constitutes a prima facie interference with a person's liberty or
property (which is the case of the forcible entry in the case at bar), the court must
consider two questions
1. Does the conduct fall within the general scope of any duty imposed by statute or
recognized at common law?
2. Does the conduct, albeit within the general scope of such a duty, involve an
unjustifiable use of powers associated with the duty?
The common law duties of the police (statutorily incorporated in s. 42(3) of the Ontario Police
Services Act) include the protection of life
 The police duty to protect life is engaged whenever it can be inferred that the 911
caller is or may be in some distress, including cases where the call is disconnected
before the nature of the emergency can be determined
 The importance of the police duty to protect life warrants and justifies a forced entry
into a dwelling in order to ascertain the health and safety of a 911 caller
79

There was no other reasonable alternative to ensure that the disconnected caller
received the necessary assistance in a timely manner
While residents have a recognized privacy interest within the sanctity of their home, the public
interest in maintaining an effective emergency response system is obvious and significant
enough to merit some intrusion on a resident's privacy interest
 However, the intrusion must be limited to the protection of life and safety; the
police do not have further permission to search premises or otherwise intrude on a
resident's privacy or property.
As for Feeney, this case was concerned solely with when the police can enter a dwelling
without a warrant to make an arrest, thus, the reasoning in Feeney does not apply to the case at
bar, which is unconcerned with powers of arrest
Meaning of Arrest
The leading case of what constitutes an arrest is now Latimer
R. v. Latimer, [1997] 1 SCR 217
Facts
 The accused was the father of Tracy, a severely disabled child who suffered from
extreme cerebral palsy and was quadriplegic
o Tracy was in constant pain, and despite the administration of medication,
experienced five or six seizures a day
 Tracy died while in the care of the accused, who advised the RCMP by telephone that
she had passed away in her sleep
o An autopsy found signs consistent with poisoning, and tests then indicated that
Tracy's blood was saturated with carbon monoxide
 The RCMP began to treat the matter as a homicide investigation and eventually the
accused was told that he was being detained for investigation into the death of his
daughter
o He was informed of his right to retain and instruct counsel without delay, of the
availability of Legal Aid duty counsel, and of his right to remain silent
o He was then taken to the police station, where he made a full confession
 The accused was convicted of second-degree murder
o His appeal was dismissed by the CA
 The accused appealed, inter alia, that he had been arbitrarily detained in his farm and
that his confession to the police should have been excluded
Issues: 1. Was the accused arbitrarily detained contrary to s.9 of the Charter?
2. Did the failure to inform the accused that he had been "arrested" and that he could be
charged with murder violate s. 10(a) of the Charter?
Holding: 1. No; 2. No; the appeal was allowed for other reasons
Lamer CJC
1. The RCMP officers who attended at the Latimer farm put Mr. Latimer under de facto arrest
80


The de facto arrest was entirely lawful because it was based on reasonable and probable
grounds that Mr. Latimer had taken his daughter's life
A de facto arrest which is lawful cannot be an arbitrary detention for the purposes
of s. 9
The appellant's strongest argument is that no arrest occurred because the officers deliberately
chose not to arrest Mr. Latimer
 Notwithstanding what the intention of the officers may have been, their
CONDUCT had the effect of putting Mr. Latimer under arrest
o In R. v. Whitfield, Judson J. held that an arrest consists either of (i) the actual
seizure or touching of a person's body with a view to his detention, OR (ii) the
pronouncing of "words of arrest" to a person who submits to the arresting officer
 With regards to this, what matters is the substance of what the accused
can reasonably be supposed to have understood, rather than the
formalism of the precise words used
On the facts of this case, a de facto arrest occurred through the use of words that conveyed
clearly that Latimer was under arrest, the conduct of the officers, and Mr. Latimer's
submission to the authority of the officers
 Mr. Latimer was told that he was being detained, and that he would be taken back to
North Battleford to be interviewed
 The police officers informed him of his right to silence and his right to counsel
 They accompanied him back into his house while he changed his clothes, telling him
that they were doing so because he was now in their custody
 Finally, at no point did Mr. Latimer protest or resist the police - he submitted to the
authority of the arresting officers
The fact that a de facto arrest occurred, however, is not sufficient to dispose of the matter,
because of the potential that his arrest was unlawful
 However, it is not necessary to address that question, because Mr. Latimer's arrest was
entirely lawful, and failing an attack against the legislative provision which authorized
the arrest, I do not see how a lawful arrest can contravene s. 9 of the Charter for being
arbitrary.
The trial judge made a specific finding that reasonable grounds for the arrest of Mr. Latimer
existed, and I see no reason to disturb that finding
 Subjectively, despite the fact that the officers decided not to arrest Mr. Latimer, it is
clear that they believed that they had reasonable grounds to arrest him
 Objectively, the reasonable person in the position of the arresting officer would have
concluded there were reasonable grounds for arrest
2. Section 10(a) of the Charter provides the right to be informed promptly of the reasons for
one's arrest or detention
 The purpose of this provision is to ensure that a person "understand generally the
jeopardy" in which he or she finds himself or herself
81
There are two reasons why the Charter lays down this requirement (see R. v. Evans)
1. Because it would be a gross interference with individual liberty for persons to have to
submit to arrest without knowing the reasons for that arrest
2. Because it would be difficult to exercise the right to counsel protected by s. 10(b) in a
meaningful way if one were not aware of the extent of one's jeopardy
There is no doubt that Mr. Latimer was not told that he was under "arrest", nor was he explicitly
told that he could be charged with murder
 However, when considering whether there has been a violation of s. 10(a), one must
look beyond the exact words used
 On the facts of this case, I have no doubt that the trial judge was right in finding that Mr.
Latimer understood the basis for his apprehension by the police and hence the extent of
his jeopardy
Appearance Notice
As part of “The Bail Reform Bill” one of the techniques for discouraging arrests adopted was a
new provision for appearance notices
s. 496 – Criminal Code
Issue of appearance notice by peace officer
Where, by virtue of subsection 495(2), a peace officer does not arrest a person, he may issue an
appearance notice to the person if the offence is
(a) an indictable offence mentioned in section 553;
(b) an offence for which the person may be prosecuted by indictment or for which he is
punishable on summary conviction; or
(c) an offence punishable on summary conviction.
 Indictable offences under s. 553 are those under absolute jurisdiction of provincial courts
s. 497 – Criminal Code
Release from custody by peace officer
(1) Subject to subsection (1.1), if a peace officer arrests a person without warrant for an offence
described in paragraph 496(a), (b) or (c), the peace officer shall, as soon as practicable,
(a) release the person from custody with the intention of compelling their appearance by
way of summons; or
(b) issue an appearance notice to the person and then release them.
Exception
(1.1) A peace officer shall not release a person under subsection (1) if the peace officer
believes, on reasonable grounds,
(a) that it is necessary in the public interest that the person be detained in custody or that
the matter of their release from custody be dealt with under another provision of this
82
Part, having regard to all the circumstances including the need to
(i) establish the identity of the person,
(ii) secure or preserve evidence of or relating to the offence,
(iii) prevent the continuation or repetition of the offence or the commission of
another offence, or
(iv) ensure the safety and security of any victim of or witness to the offence; or
(b) that if the person is released from custody, the person will fail to attend court in
order to be dealt with according to law.
Where subsection (1) does not apply
(2) Subsection (1) does not apply in respect of a person who has been arrested without warrant
by a peace officer for an offence described in subsection 503(3).
Consequences of non-release
(3) A peace officer who has arrested a person without warrant for an offence described in
subsection (1) and who does not release the person from custody as soon as practicable in the
manner described in that subsection shall be deemed to be acting lawfully and in the execution
of the peace officer’s duty for the purposes of
(a) any proceedings under this or any other Act of Parliament; and
(b) any other proceedings, unless in any such proceedings it is alleged and established
by the person making the allegation that the peace officer did not comply with the
requirements of subsection (1).
s. 498 – Criminal Code
Release from custody by officer in charge
(1) Subject to subsection (1.1), if a person who has been arrested without warrant by a peace
officer is taken into custody, or if a person who has been arrested without warrant and delivered
to a peace officer under subsection 494(3) or placed in the custody of a peace officer under
subsection 163.5(3) of the Customs Act is detained in custody under subsection 503(1) for an
offence described in paragraph 496(a), (b) or (c), or any other offence that is punishable by
imprisonment for five years or less, and has not been taken before a justice or released from
custody under any other provision of this Part, the officer in charge or another peace officer
shall, as soon as practicable,
(a) release the person with the intention of compelling their appearance by way of
summons;
(b) release the person on their giving a promise to appear;
(c) release the person on the person’s entering into a recognizance before the officer in
charge or another peace officer without sureties in an amount not exceeding $500 that
the officer directs, but without deposit of money or other valuable security; or
(d) if the person is not ordinarily resident in the province in which the person is in
custody or does not ordinarily reside within 200 kilometres of the place in which the
person is in custody, release the person on the person’s entering into a recognizance
before the officer in charge or another peace officer without sureties in an amount not
83
exceeding $500 that the officer directs and, if the officer so directs, on depositing with
the officer a sum of money or other valuable security not exceeding in amount or value
$500, that the officer directs.
Exception
(1.1) The officer in charge or the peace officer shall not release a person under subsection (1) if
the officer in charge or peace officer believes, on reasonable grounds,
(a) that it is necessary in the public interest that the person be detained in custody or that
the matter of their release from custody be dealt with under another provision of this
Part, having regard to all the circumstances including the need to
(i) establish the identity of the person,
(ii) secure or preserve evidence of or relating to the offence,
(iii) prevent the continuation or repetition of the offence or the commission of
another offence, or
(iv) ensure the safety and security of any victim of or witness to the offence; or
(b) that, if the person is released from custody, the person will fail to attend court in
order to be dealt with according to law.
Where subsection (1) does not apply
(2) Subsection (1) does not apply in respect of a person who has been arrested without warrant
by a peace officer for an offence described in subsection 503(3).
Consequences of non-release
(3) An officer in charge or another peace officer who has the custody of a person taken into or
detained in custody for an offence described in subsection (1) and who does not release the
person from custody as soon as practicable in the manner described in that subsection shall be
deemed to be acting lawfully and in the execution of the officer’s duty for the purposes of
(a) any proceedings under this or any other Act of Parliament; or
(b) any other proceedings, unless in any such proceedings it is alleged and established
by the person making the allegation that the officer in charge or other peace officer did
not comply with the requirements of subsection (1).
s. 499 – Criminal Code
Release from custody by officer in charge where arrest made with warrant
(1) Where a person who has been arrested with a warrant by a peace officer is taken into
custody for an offence other than one mentioned in section 522, the officer in charge may, if the
warrant has been endorsed by a justice under subsection 507(6),
(a) release the person on the person’s giving a promise to appear;
(b) release the person on the person’s entering into a recognizance before the officer in
charge without sureties in the amount not exceeding five hundred dollars that the officer
in charge directs, but without deposit of money or other valuable security; or
(c) if the person is not ordinarily resident in the province in which the person is in
84
custody or does not ordinarily reside within two hundred kilometres of the place in
which the person is in custody, release the person on the person’s entering into a
recognizance before the officer in charge without sureties in the amount not exceeding
five hundred dollars that the officer in charge directs and, if the officer in charge so
directs, on depositing with the officer in charge such sum of money or other valuable
security not exceeding in amount or value five hundred dollars, as the officer in charge
directs.
Interrogation – Right to Counsel and Arrest Informational duties
The law relating to interrogation presents a combination of three bodies of law
 The common law confessions rule
 The right to counsel in s. 10(b) of the Charter
 The right to silence in s. 7 of the Charter
The order to instruct accused individuals of their right to counsel is an underdeveloped area of
the law in the Code
 Appears to overlap with s. 29(2) of the Code which requires police to give reasons for
arrest
o McLachlin J discusses in Evans that the right to have the reasons of an arrest
given is an adjunct right to the right to counsel
What are the justifications behind the right to counsel?
 Allow people to understand their rights, including their right to counsel
 Protect individuals against self-incrimination, false confessions or other injustices
There are three basic questions linked to the right to counsel
 The triggers
 What are the police supposed to do to uphold this right
 What is state supposed to do to uphold it
Right to counsel (Charter s. 10(b))
s. 2(c) – Canadian Bill of Rights
Construction of law
2. Every law of Canada shall, unless it is expressly declared by an Act of the Parliament of
Canada that it shall operate notwithstanding the Canadian Bill of Rights, be so construed and
applied as not to abrogate, abridge or infringe or to authorize the abrogation, abridgment or
infringement of any of the rights or freedoms herein recognized and declared, and in particular,
no law of Canada shall be construed or applied so as to
...
(c) deprive a person who has been arrested or detained
(i) of the right to be informed promptly of the reason for his arrest or detention,
85
(ii) of the right to retain and instruct counsel without delay, or
(iii) of the remedy by way of habeas corpus for the determination of the validity
of his detention and for his release if the detention is not lawful;
s. 10 – Canadian Charter of Rights and Freedoms
Arrest or detention
10. Everyone has the right on arrest or detention
a) to be informed promptly of the reasons therefor;
b) to retain and instruct counsel without delay and to be informed of that right; and
c) to have the validity of the detention determined by way of habeas corpus and to be
released if the detention is not lawful.
1. Triggering mechanisms
Under both the Bill and the Charter, the right to counsel is not absolute, but it is only available
under arrest or detention
 Under the Bill, much of the debate in the courts consisted on whether a person who was
subject to a breath test was under detention and thus subject to right to counsel
 The leading decision on this was Brownridge v. R. (1972) where the SCC held that the
denial of a right to counsel constituted a reasonable excuse for failing to comply with a
breath sample, as long as the accused is not asserting her right to counsel for the purpose
of delay
o This was backtracked in Chromiak v. R (1980), where the SCC held that a person
subjected to a roadside test was not under detention
o Every CA, except for the majority in the Saskatchewan CA (in Therens), held that
the detention in Chromiak was still determining under the Charter
o The SCC unanimously rejected Chromiak
A textual interpretation of triggers would look at determining what arrest or detention mean
A purposive interpretation looks into what are the reasons why a right of counsel is available
and what type of situations ought to trigger these rights
 E.g. the unequal power relationship between the State and the accused
R. v. Therens, [1985] 1 SCR 613
Facts
 The accused lost control of his car and it collided with a tree
 A police officer demanded that he provided breath samples for analysis pursuant to s.
235(1) of the Criminal Code
 The accused accompanied the officer to the police station, complied with the demand,
and was subsequently charged with driving a motor vehicle while having an excessive
blood alcohol level contrary to s. 236(1)
 At trial, the accused objected to the admission of the certificate of analysis and applied,
86
pursuant to s. 24 of the Charter, for its exclusion on the ground that he had been denied
the right to be informed, upon arrest or detention, of his right to retain and instruct
counsel without delay
o The trial judge allowed the application and dismissed the charge for lack of other
evidence
 The majority of the Saskatchewan CA upheld the decision
Issue: Did the roadside alcohol test constitute detention under s. 10 of the Charter?
Holding: Yes; appeal dismissed
Le Dain J (dissenting on the remedy)
The accused’s rights under s. 10(b) of the Charter were violated
 A person who complied with a demand, pursuant to s. 235(1) of the Code, to accompany
a police officer to a police station and to submit to a breathalyzer test is "detained"
within the meaning of s. 10 of the Charter and that person is therefore entitled to be
informed of his right to retain and instruct counsel without delay
The word "detention" in s. 10 is directed to a restraint of liberty of varying duration other
than arrest in which a person may REASONABLY require the assistance of counsel and
might be prevented or impeded from retaining and instructing counsel without delay but
for the constitutional guarantee
In addition to the case of deprivation of liberty by physical constraint, there is also a
"detention" within s. 10 when a police officer assumes control over the movement of a
person by a demand or direction which may have significant legal consequence and which
prevents or impedes access to counsel
There must, however, be some form of compulsion or coercion
 Any criminal liability for failure to comply with a demand or direction of a police
officer is sufficient to make compliance involuntary
 Under s. 235(2), a refusal to comply with a s. 235(1) demand without reasonable excuse
is a criminal offence
Notwithstanding any similarity to s. 10 of the Charter, the meaning of the word "detained" in s.
2(c) of the Canadian Bill of Rights as adopted in Chromiak was not determinative of the issue
 The premise that the framers of the Charter must be presumed to have intended that the
words used by it should be given the meaning which had been given to them by judicial
decisions at the time the Charter was enacted is not a reliable guide to its interpretation
and application
 The Charter must use general language which is capable of development and adaptation
by the court
Estey J was content to simply state that the accused was detained within the meaning of s. 10(b)
when the officers administered the breathalyzer
Lecture Notes
 According to Le Dain J the person may be detained even when there are no criminal
consequences, but if the person feels that she reasonably had no choice other than
87
submit
The uncertainty left in the judgment issued in Therens, where Le Dain J spoke for only half of
the court, was cleared up in Thomsen
 Here the court held that someone under a roadside test (under what is now s. 254(2) is
detained
 It later held that there was no right to counsel since the provision constituted a
demonstrably justified reasonable limit on s. 10(b)
Thomsen v. R, [1988] 1 SCR 640
Facts
 A police officer engaged in spot checks of motor vehicles stopped the accused's vehicle
because it had a defective headlight
 The officer detected an odour of alcohol on the accused's breath and made a formal
demand that he provide a breath sample for the roadside screening device
 The accused refused and was charged with refusing to comply to give a breath sample
 At no time did the officer inform the accused that he had a right to retain and instruct
counsel without delay
Issue: Did the roadside alcohol test constitute detention under s. 10(b) of the Charter?
Holding: Yes
Le Dain J
Restating the elements of detention that appear in Therens
1. In its use of the word "detention", s. 10 of the Charter is directed to a restraint of liberty
other than arrest in which a person may reasonably require the assistance of counsel but
might be prevented or impeded from retaining and instructing counsel without delay but
for the constitutional guarantee
2. In addition to the case of deprivation of liberty by physical constraint, there is a
detention within s. 10 of the Charter, when a police officer or other agent of the state
assumes control over the movement of a person by a demand or direction which may
have significant legal consequence and which prevents or impedes access to counsel
3. The necessary element of compulsion or coercion to constitute a detention may arise
from criminal liability for refusal to comply with a demand or direction, or from a
reasonable belief that one does not have a choice as to whether or not to comply
4. Section 10 of the Charter applies to a great variety of detentions of varying duration and
is not confined to those of such duration as to make the effective use of habeas corpus
possible
The demand by the police in the case at bar fell under what constitutes detention
 The way in which the officer assumed control over the movement of the appellant was
one which might have significant legal consequence
o Given the criminal liability under s. 234.1(2) for refusal, without reasonable
excuse, the situation was one in which a person might reasonably require
the assistance of counsel
 The criminal liability for refusal also constituted the necessary compulsion or
coercion to make the restraint of liberty a detention
88
[It was later also held that failing to provide right to counsel was demonstrably justified
under s.1]
The obiter in Thomsen that detention includes psychological detention where the accused
reasonably believes that the only choice is to comply with the police, continues to trouble
courts
The following decisions show that there has been a focus beyond the mind on the accused, in
order to determine whether there is a detention within s.10
 For example, the intention of the police officers when interviewing a person
R. v. Moran (1987), 36 CCC (3d) 225
Facts
 The accused was convicted of murder
 Amongst his grounds of appeal were that the trial judge had admitted certain statements
made to the police in contravention of s. 10(b)
 A police officer telephone the accused, who was a friend of the deceased, and he agreed
to come to a station where he made some exculpatory statements
 Some days later, when pressed by the police, he admitted he had lied and gave another
exculpatory statement
o At neither of these interviews was he advised of his right to counsel, but the trial
judge held that he was not detained within the meaning of s. 10(b) at either
interview
 His appeal was dismissed by the CA
Issue: Did the questioning of the accused during the interviews at the station constitute a
detention within s. 10(b)?
Holding: No; appeal dismissed
Martin JA
As stared in R. v. Bazinet, acquiescence in a "demand" or "direction" is essential to constitute
"psychological" detention
When determining whether a person who subsequently is an accused was detained at the
time she was questioned at a police station by the police, the following non-exhaustive list
of factors is relevant
1. The precise language used by the police and whether the accused was given a choice or
expressed a preference that the interview be conducted at the Police station, rather than
at her home
2. Whether the accused was escorted to the police station by a police officer or came
herself
3. Whether the accused left at the conclusion of the interview or whether she was arrested
4. The stage of the investigation, that is, whether the questioning was part of the general
investigation of a possible crime or whether the police had already decided that a crime
had been committed and that the accused was the perpetrator and the questioning was
conducted for the purpose of obtaining incriminating statements
89
5. Whether the police had reasonable and probable grounds to believe that the accused had
committed the crime being investigated
6. The nature of the questions: whether they were questions of a general nature designed to
obtain information or whether the accused was confronted with evidence pointing to her
guilt
7. The subjective belief by an accused that she is detained, although relevant, is not
decisive, because the issue is whether she reasonably believed that she was detained
 Personal circumstances relating to the accused, such as low intelligence,
emotional disturbance, youth and lack of sophistication are to be considered in
determining the subjective belief
As for the first interview, the officer testified that he asked the appellant whether he would
prefer to come to the police station or would prefer that the police come to see him
 At that time the police were conducting a general investigation of the circumstances of
the deceased's death
The issue of whether the accused was when he was interviewed again is more complex
 Even if, in a broad sense, the appellant may have been a suspect at that time, that fact
alone is not determinative of the question whether he was detained
 The officer did not suggest that the appellant had killed the deceased nor did he attempt
to obtain an admission from the appellant that he had killed the deceased
 Finally, after making the statement the appellant told the police: "I feel a lot better now";
He went home, and was not arrested until more than two months later
R. v. Mickey (1988), 46 CCC (3d) 278
Facts
 The accused was convicted of a murder that took place near his residence
 He originally told the police that he had witnessed the murder and agreed to go to the
station to make a statement
 Throughout the statement it became apparent to the police that he may be the perpetrator
of the murder, so he turned the accused over t two other officers for interrogation
o He was not advised of his s. 10(b) right
o He made a number of statements on which the Crown later relied
Issue: Was the accused under detention within s. 10(b) when he was interrogated by the two
new officers?
Holding: Yes; appeal allowed and a new trial ordered
Macfarlane JA
The status of the accused changed when it appeared that he might have knowledge which
only the police and the assailant had
 The police officer on learning that fact, no longer treated him as a witness
 If he had treated him as a witness he would have taken a statement from him and let him
go, but instead, he turned him over for questioning to experienced interrogators
The appellant was detained from the moment the other two officers took charge of him
90

He was no longer free to leave; he was a suspect and he was being detained at the
police station for the purpose of having the police investigate a suspected offence
Lecture Notes
 This case can be seen as shifting the attention in determining what detention means,
from subjective feelings of the accused in to the purpose of the investigation
R. v. Hawkins (1992), 14 CR (4th) 286
Facts
 A complain of sexual assault was made by the daughter of a friend of the accused
 The police contacted him requesting an interview and he chose to go to the police
station
 He was advised of the nature of the complaint and his right to remain silent
 He agreed to make a written statement with some inculpatory remarks
 The accused later testified that he was at no time given the impression that the complaint
was serious and would amount to anything
 The accused was eventually convicted of sexual assault
Issue: Was the written statement given in violation of s. 10(b)?
Holding: Yes; appeal allowed
Marshall JA
While suspicion will be an important element in assessing the approach taken by an
investigating officer, it is not in itself enough to substantiate a detention within the meaning of
s. 10(b)
 A police officer investigating whether an offence has been committed is entitled to
question anybody, whether suspected or not
It is when these SUSPICIONS BECOME CRYSTALLIZED, and the investigator's
approach to the encounter is changed from a questioning of the individual to an
examination with an intent to charge her with the offence, that a detention must be
deemed to have arisen engaging that person's rights under s. 10(b)
The right to be informed of entitlement to counsel must arise at precisely the point where the
individual is subject to "the coercive power of the State"
 It is at this moment that her liberty and security is placed in peril, and, the Charter
mandates that the individual be informed not only of her right to silence but also
be afforded the additional protection of being advised of the right to consult
counsel
The act creating the adversarial relationship and TRIGGERING these individual rights
can be identified in the centering by the investigating officer upon the individual as the
offender and the confining of the interview to attempts to discern her culpability and to
elicit from the individual evidence to be used against her
 This pertains even though the person interviewed may be unconscious of any
compulsion in the sense of feeling she has no choice but to respond
o It will generally suffice to regard that, from the individual's point of view, an
91
ambience of compulsion permeated the entire interview and it is not necessary to
establish that the incriminating statement itself was specifically made because
the person felt a lack of choice to do otherwise
Lecture Notes
 The ruling seems to imply that a detention takes place when there is a reasonable
suspicion in the mind of the police
R. v. Feeney supra, [1997] 2 SCR 13
Facts
 While investigating a murder, the police went to the accused’s home and entered his
residence having received no answer at the door
 After bringing the accused to better lighting where it was apparent that he was covered
in blood, he was arrested
 Following caution with respect to the right o counsel, the police asked the accused some
questions which he answered
 He was eventually convicted
Issue: Were the accused’s statements obtained in violation of s. 10(b)?
Holding: Yes
Sopinka J
Detention began once the officer touched the appellant's leg and ordered him to rise, thus
his s. 10(b) rights were violated at the time of his initial detention
 The accused was not given any caution at this time but only after the appellant had been
escorted to the light, thus failing to satisfy the informational requirements of s. 10(b)
 Furthermore, the accused was not given an adequate opportunity to consult with counsel
before being questioned
L’Heureux-Dubé J (dissenting on this matter)
The police are not obliged to read an accused his s.10(b) rights the instant he is detained
or arrested
 The police must be permitted the latitude to assess and gain control of the situation and
determine whether a potentially dangerous situation exists
 In addition, the officer was walking into a dark room and the appellant was sleeping,
and surely, the officer was not supposed to read the appellant his rights while he was
asleep
The caution was given at the first reasonable opportunity
 The delay in this case between the time the officer grabbed the accused’s leg to wake
him up and the time he was read his right to counsel was no more than a few minutes
Notwithstanding the SCC’s unanimous ruling in Hawkins, some courts continued to insist that
the authority is Moran
The SCC’s interpretation of the content of the s.10 (b) guarantee has been dominated by the
distinguishing approach between informational and implementation duties in Bartle
92
R. v. Bartle, [1994] 3 SCR 173
Lamer CJ
The purpose of the right to counsel guaranteed by s. 10(b) of the Charter is to provide
detainees with an opportunity to be informed of their rights and obligations under the law
and, most importantly, to obtain advice on how to exercise those rights and fulfil those
obligations
 This opportunity is made available because, when an individual is detained by state
authorities, she is put in a position of disadvantage relative to the state
 Not only has this person suffered a deprivation of liberty, but she may also be at risk of
incriminating herself
S. 10(b) of the Charter imposes the following duties on state authorities who arrest or
detain a person
1. To inform the detainee of her right to retain and instruct counsel without delay and of
the existence and availability of legal aid and duty counsel
2. If a detainee has indicated a desire to exercise this right, to provide the detainee with a
reasonable opportunity to exercise the right (except in urgent and dangerous
circumstances)
3. To refrain from eliciting evidence from the detainee until she has had that reasonable
opportunity (again, except in cases of urgency or danger)
The first duty is an informational one
 The second and third duties are more in the nature of implementation duties and are not
triggered unless and until a detainee indicates a desire to exercise her right to counsel
The right to counsel under s. 10(b) is not absolute
 Unless a detainee invokes the right and is reasonably diligent in exercising it, the
correlative duty on the police to provide a reasonable opportunity and to refrain from
eliciting evidence will either not arise in the first place or will be suspended
The rights guaranteed by s. 10(b) may be waived by the detainee, although the standard for
waiver will be high, especially in circumstances where the alleged waiver has been implicit
Moran and Therens still largely remain the authority with regards to the triggers to the
right to counsel
 The difference with later cases lies on the importance given to the subjective feelings of
the accused regarding the detention as compared to the subjective feelings of the police
2. Informational duties
Unlike the Bill, s. 10(b) of the Charter expressly confers the right to be informed of the right
to retain and instruct counsel
 This is mandatory on arrest and detention
93
Informational duties refer to stating to the accused of her right to counsel
R. v. Brydges, [1990] 1 SCR 190
Facts
 The accused, a resident of Alberta, was arrested in Manitoba in connection with a
murder which took place in Edmonton
 He was charged with second degree murder and informed without delay of his right to
retain and instruct counsel
 The accused asked the investigating officer if they had Legal Aid in Manitoba because
he could not afford a private lawyer
o The officer, who was from Edmonton, answered that he imagined that they had
such a system in Manitoba
o The officer then asked the accused if he felt there was a reason for him to
wanting to talk to a lawyer. The accused answered "Not right now, no"
 During the interrogation which followed, the accused made a number of statements
 He later interrupted the questioning and requested a Legal Aid lawyer and the lawyer
contacted by the police advised the accused not to say anything more and the
interrogation ended
 At trial, the judge found that at the beginning of the interrogation the accused essentially
requested the assistance of counsel and because the police did not assist the accused in
exercising his right to counsel, by determining the availability of Legal Aid at that time,
the accused's rights under s. 10(b) of the Charter were violated
 As a result, the accused was acquitted
 A majority of the CA set aside the acquittal and ordered a new trial
Issue: Was s. 10(b) violated by failing to inform the accused of the availability of legal aid?
Holding: Yes; appeal allowed
Lamer J
Once the accused requested the assistance of counsel it was incumbent on the police officer to
facilitate contact with counsel by giving the appellant a reasonable opportunity to exercise his
right to counsel
 There is a duty on the police to inform him of the existence of duty counsel and the
ability to apply for Legal Aid
It is not always the case that immediately upon detention an accused will be concerned about
retaining the lawyer that will eventually represent him at a trial, if there is one
 Rather, one of the important reasons for retaining legal advice without delay upon being
detained is linked to the protection of the right against self-incrimination
The failure of the police to inform the appellant of the existence of Legal Aid or duty counsel at
the time that he first indicated a concern about his ability to pay a lawyer, was a restriction on
the appellant's right to counsel, in so far as the appellant was left with an erroneous
impression of the nature and extent of his s. 10(b) rights
It is consistent with the purpose underlying s. 10(b) of the Charter to impose a duty on the
94
police to inform accused individual of the existing of legal aid in all cases of detention,
regardless of whether the accused expresses a concern with affording an attorney or not
 Otherwise police officers would be put in the difficult position of having to judge, on the
spot, whether a person has expressed concerns about affordability or whether there
should be further inquiries made in this respect
Before concluding, it is my view that in light of the imposition of the additional duty on the
police as part of the information component of the s. 10(b) caution, a transition period is
appropriate
 This transition period is needed to enable the police to properly discharge their new
burden, more specifically to take into account the reality that police officers often use
printed cards from which they read the caution given to detainees
 A period of thirty days from the date of this judgment is sufficient time for the police
forces to react, and to prepare new cautions
Lecture Notes
 Does this mean that it is now part of police procedure to advise of legal aid when
making reference to the right to counsel?
R. v. Bartle (1994), 33 CR (4th) 1
Facts
 The accused was charged with impaired driving after he failed a breathalyzer test early
in Saturday morning
 A section 10(b) Charter caution was read to him from a card, advising of the right to
apply for free legal aid
o The officer did not, however, refer to the fact that there was free and immediate
preliminary legal advice available from duty counsel, who could be reached by
calling a toll free number
 After being taken to the station he was twice asked if he wanted to speak to a lawyer,
but once again he was not informed of this toll free number to free duty counsel
 On both occasions the accused declined
Issue: Was the defendant’s right in s. 10(b) Charter violated by the absence of information
about the free duty counsel?
Holding: Yes; the appeal was allowed and the acquittal restored
Lamer CJ
It is critical that the information component of the right to counsel be comprehensive in
scope and that it be presented by police authorities in a "timely and comprehensible"
manner
Unless they are clearly and fully informed of their rights at the outset, detainees cannot be
expected to make INFORMED and MEANINGFUL choices and decisions about whether
or not to contact counsel and, in turn, whether to exercise other rights, such as their right
to silence (see Hebert)
 A person who does not understand his or her right cannot be expected to assert it
 Likewise, before an accused can be said to have waived her right to counsel, she must be
possessed of sufficient information to allow her to make an informed choice as regards
95
exercising the right
In light of the rule that, absent special circumstances, police are not required to assure
themselves that a detainee fully understands the s. 10(b) caution, it is important that the
standard caution given to detainees be as instructive and clear as possible
 Particularly given that subsequent duties on the state are not triggered unless and until a
detainee expresses a desire to contact counsel
The purpose of the right to counsel would be defeated if police were only required to
advise detainees of the existence and availability of Legal Aid and duty counsel after some
triggering assertion of the right by the detainee
 A detainee is entitled under the information component of s. 10(b) of the Charter to be
advised of whatever system for free, preliminary legal advice exists in the
jurisdiction and of how such advice can be accessed
In the case at bar, section 10(b) required that the existence and availability of this duty counsel
system and how to access it be routinely communicated by police in a timely and
comprehensible manner to detainees
 The accused was not properly informed of his rights under s. 10(b) and as a result, he
may have been misled about the nature and extent of his right to counsel
 The s. 10(b) caution that the appellant received, both at the roadside and at the police
station, failed to convey the necessary sense of immediacy and universal availability of
legal assistance, which the majority in Brydges said must be conveyed as part of the
standard s. 10(b) warning in jurisdictions where such a service exists
o Reference to Legal Aid was confusing in so far as it implied that free legal
advice, while available, was contingent on applying for it once charged
 The 1-800 number, or at least the existence of a toll-free telephone number, should have
been conveyed to the appellant upon his arrest at the roadside even though there were no
telephones available
 Furthermore, the appellant did not waive his right to receive a caution that fully
informed him of his right to counsel
o The validity of a waiver of a procedural right is dependent upon it being clear
and unequivocal that the person is waiving the procedural safeguard and is doing
so with full knowledge of the rights the procedure was enacted to protect
One thing that INFORMATIONAL DUTIES DO NOT REQUIRE, is that the accused in
fact understands the right to counsel
 In Baig the SCC placed the onus on the accused to show that she did not understand the
right
 The IMPLEMENTATION DUTIES, on the other hand, do require that the accused
understands her right
2. Implementation duties
96
Laskin’s minority concurring judgment in Brownridge is still referred to in this respect, stating
that the right to counsel raises a correlative obligation on the police to facilitate contact with
counsel, such as providing the accused with a phone
Some courts have held that privacy should be afforded by the police to the accused in contacting
counsel, while the SCC in Jumaga v. R (1977), held that it is not a requirement
 The positive wording of s. 10(b) has been relied upon to distinguish Jumaga and hold that
privacy must be afforded whether or not it is requested (see R. v. Playford (1987))
R. v. Manninen, [1987] 1 SCR 1233
Facts
 The accused was arrested for theft and possession of a stolen car and armed robbery
 The arresting officer read him his rights from a card twice because of a dismissive
remark respondent had made following the first reading
 Even though the accused indicated that he was not going to say anything until he saw his
lawyer, the officers continued to question him
 He did not directly request to use the telephone and the officers did not volunteer the use
of it
o He did not speak to his lawyer until his lawyer called him at the police station
that evening
 The trial judge, in convicting the accused, relied on a statement made by him in reply to
a barbed question asked before his lawyer had made contact with him
 The trial judge held that, even if the right to counsel had been infringed, the admission
of the statements would not bring the administration of justice into disrepute
 The CA unanimously allowed the accused’s appeal, quashed the convictions and
ordered a new trial
Issue: Was the accused’s right to counsel infringed?
Holding: Yes; appeal dismissed
Lamer J
S.10(b) imposes at least TWO duties on the police in addition to the duty to inform the
detainee of her rights
1. The police must provide the detainee with a reasonable opportunity to exercise the
right to retain and instruct counsel without delay
 This means allowing her upon his request to use the telephone for that purpose if
one is available
2. The police must cease questioning or otherwise attempting to elicit evidence from the
detainee until she has had a reasonable opportunity to retain and instruct counsel
 The purpose of the right to counsel is to allow the detainee not only to be informed
of her rights and obligations under the law but, equally if not more important, to
obtain advice as to how to exercise those rights
 An example of this is the right to remain silent
S. 10(b) requires at least that the authorities inform the detainee of her rights and not
prevent her in any way from exercising them
97

Also, where a detainee is required to provide evidence which may be incriminating and
refusal to comply is punishable as a criminal offence, s. 10(b) also imposes a duty not
to call upon the detainee to provide that evidence without first informing her of her
s. 10(b) rights and providing him with a reasonable opportunity and time to retain and
instruct counsel
The first aspect of the right to counsel was clearly infringed in this case
 There was a telephone immediately at hand in the office, which the officers used for
their own purposes
o It was not necessary for the respondent to make an express request to use the
telephone
 Of course, there may be circumstances in which it is particularly urgent that the
police continue with an investigation before it is possible to facilitate a
detainee's communication with counsel, yet there was no urgency in this case
The second aspect of the respondent's right to counsel was also infringed
 Immediately after the respondent's clear assertion of his right to remain silent and his
desire to consult his lawyer, the police officer commenced his questioning as if the
respondent had expressed no such desire
 The police proceed with their questioning of the detainee before providing him with a
reasonable opportunity to retain and instruct counsel, but there was no such urgency in
this case.
The Crown contends that there was no infringement of the right to counsel because the
respondent had waived his right by answering the police officer's questions
 While a person may implicitly waive their rights under s. 10(b), the standard will
be very high (see Clarkson)
 The accused’s conduct in this case did not constitute an implied waiver of his right to
counsel
o He did not intend to waive his right, as he clearly asserted it at the beginning and
at the end of the questioning
o The respondent had the right not to be asked questions, and he must not be held
to have implicitly waived that right simply because he answered the questions
 Otherwise, the right not to be asked questions would only exist where the
detainee refused to answer
Lecture Notes
 By talking to the police regardless of having asked for counsel, the accused was not
waiving his rights
In Manninen the accused had clearly asserted his right to counsel so it was not necessary to
decide whether the police correlative duty only arises where such an assertion is made
 This issues was discussed in Baig
Baig v. R., [1987] 2 SCR 537
Facts
 The accused was arrested for murder and promptly informed of his right to counsel
98

When asked if he understood his right, the accused replied "How can you prove this
thing?"
 At the police station, he made an inculpatory statement and the statement form indicated
that the accused gave an affirmative answer to the question of whether he understood
that he had the right to retain and instruct counsel without delay
 At trial, the trial judge ruled that the accused's right under s. 10(b) of the Charter had
been violated and excluded the statement
 The accused was acquitted on a directed verdict
 The Court of Appeal quashed the acquittal and ordered a new trial
Issue: Are the police correlative duties in relation to s. 10(b) triggered if they accused has not
yet expressed a desire to exercise the right to counsel?
Holding: No; appeal dismissed
The Court
The judges agreed with Tarnopolsky J.A. in R. v. Anderson (1984) in that absent proof of
circumstances indicating that the accused did not understand her right to retain counsel
when she was informed of it, the onus has to be on her to prove that she asked for the
right but it was denied or she was denied any opportunity to even ask for it
No such evidence was put forth in this case
 Absent such circumstances, once the police have complied with s. 10(b), by advising the
accused without delay of her right to counsel, there are no correlative duties triggered
and cast upon them until the accused, if she so chooses, has indicated his desire to
exercise her right to counsel
Comments
 Can Baig be reconciled with Manninen?
 Should the right of counsel once invoked by a suspect, limit subsequent questioning by
the police?
Leclair and Ross v. R., [1989] 1 SCR 3
Facts
 Three youth were arrested at night and charged with break, enter and theft
 They were advised of their right to counsel and tried unsuccessfully to contact counsel
by telephone at 2 am
 Leclair was asked if he wished to call another lawyer and he said no
 The police conducted a line-up an hour later and the accused were not advised that they
were under no obligation to participate
Issue: Were the implementation duties of the police successfully exercised?
Holding: No
Lamer J
As expressed in Manninen s. 10(b) imposes at least two duties on the police in addition to the
duty to inform detainees of their rights
 The first is that the police must give the accused or detained person who so wishes a
reasonable opportunity to exercise the right to retain and instruct counsel without delay
99


The second is that the police must refrain from attempting to elicit evidence from the
detainee until the detainee has had a reasonable opportunity to retain and instruct
counsel
The police fulfilled neither duty in this case
1. Affording a Reasonable Opportunity
It was highly unlikely that the accused would be able to contact their counsel outside the normal
office hours
The fact that Leclair did not want to call another lawyer cannot be viewed as a waiver of
his right to retain counsel
 He merely asserted his right to counsel and to counsel of his choice
 As per R. v. Tremblay (1987), a detainee must be reasonably diligent in the exercise of
the right to counsel, otherwise the correlative duties imposed on the police and set out in
Manninen are suspended
 Accused or detained persons have a right to choose their counsel and it is only if
the lawyer chosen cannot be available within a reasonable time that the detainee or
the accused should be expected to call another lawyer
 Once Leclair asserted his right to instruct counsel, and absent a clear indication that he
had changed his mind, it was unreasonable for the police to proceed as if he had waived
his right to counsel
As stated in Korponay v. Attorney General of Canada (1982), any WAIVER "is dependent
upon it being clear and unequivocal that the person is waiving the procedural safeguard
and is doing so with full knowledge of the rights the procedure was enacted to protect and of
the effect the waiver will have on those rights in the process"
In the case of Ross, there is no evidence that the police even asked whether he wanted to call
another lawyer
There was NO URGENCY justifying that the police proceed immediately and it cannot
be said that the appellants had a real opportunity to retain and instruct counsel
2. Refraining from Taking Further Steps
As held in Manninen, the police have a duty to cease questioning or otherwise attempting to
elicit evidence from the detainee until she has had a reasonable opportunity to retain and
instruct counsel
 This also means that, once an accused or detained person has asserted that right,
the police cannot, in any way, compel the detainee or accused person to make a
decision or participate in a process which could ultimately have an adverse effect in
the conduct of an eventual trial until that person has had a reasonable opportunity to
exercise that right
 In the case at bar, it cannot be said that the appellants had a real opportunity to retain
and instruct counsel before the line-up was held
100

Nor can it be said that there was any urgency or other compelling reason justifying
proceeding so precipitously
The Crown also submitted that there was no violation of the right to counsel because the
appellants did not have the right to have their lawyers present during the line-up
 But, even if the appellants could not have their lawyers present during the line-up, this
does not imply that counsel is of no assistance to a suspect
o The accused could also have been told how a well-run line-up is conducted
 There is further no legal obligation to participate in a line-up
Furthermore, that the accused did not refuse to participate in the line-up cannot by itself amount
to a waiver of the right to counsel
 It would contradict the purpose of this right to conclude that a detained or accused
person has waived the right to counsel simply by submitting, before being
instructed by counsel, to precisely those attempts from which the police should
refrain
R. v. Burlingham, [1995] 2 SCR 206
Facts
 The accused, who had been charged with one murder and was suspected in a second,
was subjected to an intensive and often manipulative interrogation by the police
o He was systematically questioned notwithstanding his stating repeatedly that he
would not speak unless he could consult with his lawyer
 The police offered the accused a "deal" he would be charged with second degree murder
if he provided the police with the location of the gun and other ancillary information
related to that murder
o When he refused to accept the "deal" without consulting his lawyer, the officers
continued to badger him about the usefulness of his lawyer and informed him
this "one-time" chance would be kept open only for the weekend -- the period
when appellant's counsel was unavailable
o He eventually agreed, despite his being advised by another lawyer not to talk to
the police, and fulfilled his part of the deal by giving police a full confession,
bringing them to the murder site, and telling them where the murder weapon had
been thrown
 A misunderstanding arose as to the deal, since the accused understood that he would be
allowed to plead not guilty to a charge of second degree murder whereas the Crown
insisted that he would have to plead guilty to that charge
o The trial judge found as a fact that the police officers had made an honest
mistake
 The accused was charged with first degree murder
 The trial judge found that appellant's right to counsel had been breached and held that
appellant's confession, his disclosure of the location of the weapon and his directions
and gestures to the police were inadmissible
o He, nonetheless, admitted the fact of finding the gun, the actual gun and the
testimony of some witnesses
 The accused was convicted of the first degree murder and the CA affirmed that decision.
101
Issue: Was the accused’s denied of his right to counsel?
Holding: Yes; appeal allowed
Iacobucci J
The Supreme Court has consistently given a broad interpretation to s. 10(b)
 In the case at bar, there were several ways in which the appellant's right was denied
1. The police continually questioned him despite his repeated statements that he would say
nothing absent consultation with his lawyer
 Section 10(b) requires, barring urgent circumstances, that the police refrain from
attempting to elicit incriminatory evidence once a detainee has asserted his or her
right to counsel
2. S. 10(b) specifically prohibits the police, as they did in this case, from belittling an
accused's lawyer with the express goal of undermining the accused's confidence in and
relationship with defence counsel
3. S. 10(b) was violated when the officers pressured the accused into accepting the "deal"
without first having the opportunity to consult with his lawyer
 Allowing the appellant to call a random lawyer is, given the seriousness of the situation
he faced and the circumstances of this case, insufficient for the officers to discharge
their responsibilities under s. 10(b)
 Although it is clear that s. 10(b) does not guarantee an accused the right to the counsel
of his or her choice at all times, in a situation such as the appellant's, the offer should
have been made at a point in time when the accused's lawyer was available or the police
should have kept it open to a point in time when the accused's counsel would
reasonably be considered to be available
As discussed in Evans, the police have the duty to advise a suspect of the right to counsel
where there is a fundamental and discrete change in the purpose of an investigation which
involves a different and unrelated offence or a significantly more serious offence than that
contemplated at the time of the original instruction of the right to counsel
S. 10(b) mandates the Crown or police, whenever offering a plea bargain, to tender that
offer either to accused's counsel or to the accused while in the presence of her counsel,
unless the accused has expressly waived the right to counsel.
It is important to emphasize that, in the case at bar, there was no urgency to the matter
 Mere expediency or efficiency is not sufficient to create enough "urgency" to
permit a s. 10(b) breach
Lecture Notes
 One of the issues raised in this case was that when a plea bargaining is offered, the
accused should be advised of her right again
In R. v. Prosper (1994), the SCC held that there is no constitutional obligation on
governments to provide a free duty counsel on arrest or detention
102

Lamer J further held for the majority that once the right is asserted, s. 10(b) is violated
where the accused is not given a reasonable opportunity to consult counsel, where
the lack of availability of duty counsel is to be taken into consideration by the court
3. Waiver and duty to be reasonably diligent in exercise of right
Generally speaking, the Supreme Court has been generous to the accused when characterizing
the issues of waiver of the right to counsel
 It is far less generous when insisting on the duty to assert the s. 10(b) right with
reasonable diligence
 It seems unclear, however, which ruling will be adopted in each case, thus leading to
inconsistencies
There is a difference between the standards used in the right to counsel and the right to
silence
 The later uses an operating mind test
 The former involves an awareness of the consequences test
R. v. Clarkson, [1986] 1 SCR 383
Facts
 The accused was very intoxicated when she was charged with her husband's murder, as
well as when she was given the customary police warning and informed of her right to
counsel
 She said there was "no point" in having counsel and underwent police questioning while
still drunk and very emotional
 The interrogation continued in spite of the efforts of an aunt to have it postponed and to
convince appellant to stop talking until counsel was present
 The trial judge excluded inculpatory statements finding that she did not appreciate the
consequences of making such statements
 The CA rejected the trial judge's test of admissibility and ordered a new trial
Issue: Could the accused effectively waive her right to counsel while being intoxicated?
Holding: No; appeal allowed
Wilson J
Given the concern for fair treatment of an accused which underlies the right to counsel in s.
10(b), it is evident that any alleged waiver of this right by an accused must be carefully
considered and that the accused's awareness of the consequences of what she was saying is
crucial
As per Korponay v. Attorney General of Canada (1982), any waiver "is dependent upon it
being clear and unequivocal that the person is waiving the procedural safeguard and is
doing so with full knowledge of the rights the procedure was enacted to protect and of the
effect the waiver will have on those rights in the process"
103
US jurisprudence shows that an accused must knowingly intelligently and with a full
understanding of the implications, waive her constitutional rights to counsel
 The US Supreme Court has gone so far as to indicate that not only must an accused
person be cognizant of the consequences of waiving the constitutional right to counsel in
a general way, but she must be aware of the legal specificities of her own case
It is clear that the waiver of the s. 10(b) right by an intoxicated accused must pass some
form of "AWARENESS OF THE CONSEQUENCES" test
 Any voluntary waiver in order to be valid and effective must be premised on a true
appreciation of the consequences of giving up the right
Unlike the confession itself, there is no room for an argument that the court in assessing
such a waiver should only be concerned with the probative value of the evidence
 Rather, the purpose of the right, as indicated in Therens, is to ensure that the accused is
treated fairly in the criminal process
Application to the case
 The trial judge found as a fact that the appellant's confession could not pass the
"awareness of the consequences" test, so then presumably neither could the waiver of
the s. 10(b) right to counsel
 At the very minimum it was incumbent upon the police to delay their questioning
and the taking of the appellant's statement until she was in a sufficiently sober state
to properly exercise her right to retain and instruct counsel or to be fully aware of the
consequences of waiving this right
Lecture Notes
 The standard of awareness of the consequences appears to be higher than the “operating
mind” used in the right to silence cases
Bartle and the companion cases also hold that a waiver of the informational component of
s. 10(b) will have to be explicit and require a reasonable belief that the accused knew about
the right to counsel and how to exercise it
R. v. Smith, [1989] 2 SCR 368
Facts
 The accused was arrested at his home around 7:00 p.m., charged with robbery and
informed of his right to retain and instruct counsel
 After various stops made to accommodate the accused, the police officers and the
accused arrived at the police station at approximately 9:00 p.m
 The accused requested the opportunity to communicate with his lawyer and the police
gave him a telephone and a telephone book
o Because it was late and the only telephone number appearing in the telephone
book was his lawyer's office number, the accused decided not to call and advised
the police that he would contact his lawyer in the morning
o The police suggested that he try to make the call but he refused
 He was placed in a police cell for approximately one hour and then taken to an interview
room
104
o The accused told the police that he would not answer questions concerning the
robbery until he could speak with his lawyer, but the police pursued their
questioning and tried to obtain a statement
o The accused indicated in two other occasions that he wanted to speak to his
lawyer but he finally made a statement, specifying that it was made "off the
record"
 The trial judge ruled that the accused's rights under s. 10(b) had not been violated and
admitted the statement
 The accused was subsequently convicted and the majority of the CA upheld the
conviction
Issue: Was the accused given reasonable opportunity to retain and instruct counsel?
Holding: Yes; appeal dismissed
Lamer J
As per R. v. Tremblay (1987), the duties imposed on the police as stated in Manninen are
suspended when the arrested or detained person is not reasonably diligent in the exercise
of her rights
This limit on the right to counsel is essential because without it, it would be possible to delay
needlessly and with impunity an investigation and even, in certain cases, to allow for an
essential piece of evidence to be lost, destroyed or rendered impossible to obtain
The rights set out in the Charter, and in particular the right to retain and instruct counsel, are
not absolute and unlimited rights
 They must be exercised in a way that is reconcilable with the needs of society
The situation would be very different in a case like R. v. Ross, if the accused had tried to contact
her lawyer but had failed in her attempt
 In these circumstances, the accused would have been justified to ask or a delay until the
opening of offices in the morning
 The burden of proving that it was impossible for the accused to communicate with his
lawyer when the police offered him the opportunity to do so was on the accused
An arrested or detained person who has had a reasonable opportunity to communicate with
counsel but who was not diligent in the exercise of this right cannot, subsequently, require the
police to suspend, one more time, the investigation or the questioning
 This does not apply, however, when the circumstances that exist when she asks
subsequently to exercise the right to counsel are substantially different from those which
existed when he had the opportunity to communicate with a lawyer (e.g. when she
originally was accused of something and is later accused of another crime)
L’Heureux-Dubé J
The circumstances of time and place as well as the responsible behaviour of the police officers
clearly gave the accused more than a reasonable opportunity to communicate with counsel of
his choice, or at the very least, to obtain legal advice before answering the questions
105

The accused elected not to avail himself of this opportunity
Sopinka J
In this case the accused was most casual in asserting his right to counsel
The accused had been afforded a reasonable opportunity to retain and instruct counsel
 This determination is largely a question of fact
 What is a reasonable opportunity is determined by reference to all the circumstances of
the case, including the action of the accused
 Whether an opportunity is reasonable must be judged in part in light of the diligence
with which the accused seeks to avail herself of the right
La Forest J (dissenting)
Following Clarkson, it cant be disputed that the accused waived his right to counsel
 The accused positively and repeatedly asserted his desire to exercise his right to counsel
 Answering the questions put to him does not constitute a waiver of his right to counsel
in these circumstances
 It is clear that these answers were given after he had asserted his desire to speak with his
lawyer
 It is also clear that he believed they were given "off the record",
 In these circumstances, it could not be said that there was an awareness of the
consequences of speaking in the absence of his lawyer
The appellant did not have a reasonable opportunity to contact counsel
 He decided not to make the call thinking, quite reasonably that at that time of night there
would be no one in the lawyer's office
 The evidence also discloses that the police actively dissuaded the accused from his
resolve not to speak until he had talked with his lawyer
The accused should be able to wait and get in touch with his lawyer, rather than with any
lawyer
 If the investigation needed urgently to be pursued, the position might be different, but it
cannot be said that there was any urgency in this case
This case is governed by Ross
 There was no reason to proceed before the appellant had the opportunity to speak with
counsel the next morning
The only factor I can find in the police's favour in this case is that the appellant did not attempt
to call his lawyer
 However, I cannot hold it against the appellant that he was willing to spend the night in
jail when the likelihood was that he would not have been able to talk to his lawyer that
night anyway
106
Right to Silence
This right has been very poorly defined in Canada and did not exist for a long time
 Until recently, the confession rule was the only standard restricting the admissibility of
some incriminating statements
The reasons behind exercising this right are the following
 Unequal power balance between the State and the accused
 Risk of a false confession
Confession rule
 Comes from Ibrahim
o Confessions made to persons in authority under promises or threats may be
unreliable
In Hebert, the idea of voluntariness was expanded to include the reputation of the administration
of justice
On the burden of proof
 Voluntariness has to be proved by the Crown beyond a reasonable doubt
 Showing that the administration of justice was brought into disrepute has to be done by
the accused, as any other Charter violation
o Then the burden shifts to the Crown
What is the Crown requested to show under the voluntary confession rule? (assuming that
the person who heard the confession was a person in authority) – See Oickle
 No threats of promises
 Operating mind
o Does not require more than knowledge of what the accused is saying
 No oppression
o Though Oickle set a very high bar for oppression
 No police trickery
o This forms part of a distinct inquiry, which deals with the integrity of the justice
system
 It seems like the first three deal with the reliability of the confession
o The standard is the shock of the community
o Police informants seem to be admissible within this high standard of trickery
As for the right to silence under s.7 of the Charter
 Police persuasion is acceptable
 After detention
o Voluntary statements to cellmates are allowed
o Undercover officers to observe are allowed, but not to elicit information
107
s. 7 – Canadian Charter of Rights and Freedoms
Life, liberty and security of person
Everyone has the right to life, liberty and security of the person and the right not to be deprived
thereof except in accordance with the principles of fundamental justice.
R. v. Hebert, [1990] 2 SCR 151
Facts
 The accused was arrested on a charge of robbery and informed upon arrest of his right to
counsel. At the police station, after consulting counsel, he advised the police that he did
not wish to make a statement
 The accused was then placed in a cell with an undercover police officer posing as a
suspect under arrest by police. The officer engaged the accused in conversation, during
which the accused made various incriminating statements implicating him in the robbery
 Prior to trial, there was a voir dire to determine the admissibility of these statements
o The judge held that the accused's right to counsel under s. 10(b) of the Charter
and his right to remain silent asserted under s. 7 of the Charter had been violated
and excluded the statements pursuant to s. 24(2) of the Charter
o The Crown offered no evidence, and the accused was later acquitted
 The Court of Appeal set aside the accused's acquittal and ordered a new trial. The Court
found that the police conduct did not violate the accused's right to counsel or his right to
remain silent
Issue: Did a statement elicited by an undercover police officer once the accused expressed his
intention not to speak to the police violate his Charter rights?
Holding: Yes; the appeal was allowed and the acquittal was restored
McLachlin J
Section 7 of the Charter accords a detained person a pre-trial right to remain silent which
extends beyond the narrow formulation of the confessions rule
 The rules relating to the right to remain silent and the privilege against self-incrimination,
suggest that the scope of the right in the pre-trial detention period must be based on the
fundamental concept of the suspect's right to freely choose whether to speak to the
authorities or remain silent
 This concept, accompanied by a concern with the repute and integrity of the judicial
process, is consistent with the right to counsel and the right against self-incrimination
affirmed by the Charter. It is also consistent with the Charter's approach to the question of
improperly obtained evidence under s. 24(2) and with the underlying philosophy and
purpose of the procedural guarantees the Charter enshrines -- in particular in s. 7
 Under s. 7, the state is not entitled to use its superior power to override the suspect's
will and negate his choice to speak to the authority or to remain silent
o The test to determine whether the suspect's choice has been violated is
ESSENTIALLY OBJECTIVE
The scope of the right to silence, however, does not go as far as to prohibit police from
obtaining confessions in all circumstances
108

The s. 7 right to silence retains the objective approach to the confessions rule and would the
following limits
o There is nothing that prohibits the police from questioning an accused or a suspect in
the absence of counsel after he has retained counsel
 Police persuasion, short of denying the suspect the right to choose or of
depriving him of an operating mind, does not breach the right to silence
o The right applies only after detention
o The right does not affect voluntary statements made to fellow cell mates
o A distinction must be made between the use of undercover agents to observe
the suspect, and the use of undercover agents to actively elicit information in
violation of the suspect's choice to remain silent
o Even where a violation of the suspect's right is established, the evidence may, where
appropriate, be admitted.
 Only if the court is satisfied that its reception would be likely to bring the
administration of justice into disrepute can the evidence be rejected under
s. 24(2) of the Charter
o Where the police have acted with due care for the suspect's rights, it is unlikely that
the statements they obtain will be held inadmissible.
Here, the accused exercised his choice not to speak to the police and the police violated his right to
remain silent under s. 7 of the Charter by using a trick to negate his decision. Section 1 of the
Charter was inapplicable because the police conduct was not a limit "prescribed by law" within the
meaning of that section
 The evidence obtained in breach of the accused's right under s. 7 should be excluded
pursuant to s. 24(2) of the Charter, otherwise the reception of the evidence would render the
trial unfair and the accused would be deprived of his presumption of innocence
Lectures notes
 A confession is not voluntary when the behavior of the police is likely to bring the
administration of justice into disrepute or when the person cannot exercise her choice to
remain silent
R. v. Oickle, [2000] 2 SCR 3
Facts
 Between Feb 94 and April 95, there were 8 fires in 4 buildings and 2 cars in Waterville,
Nova Scotia
 One car belonged to the accused’s dad and one belonged to his fiancée, Tanya
 Fires appeared to have been deliberately set, with the possible exception of Tanya’s car
 Cops did extensive investigation including asking 7 or 8 people to sit for polygraphs. 5 or
6 did and passed and were removed from list of suspects
 Oickle agreed to sit for the test and went to a motel at 3 pm one day for it. The police
audiotaped the events and Sgt Taker administered the test
 The accused was informed of rights, and that he could leave at any time. He was given
pamphlet to review and signed a consent form
 In a lengthy pre-test interview, an exculpatory statement was taken, which formed the
basis for the polygraph test
 The sergeant then did the test, which lasted only a few minutes. At its conclusion Oickle
109
was told that he had failed. He was reminded of his rights and then questioned for more
than an hour
 At one point he asked:“what if I admit to the car? Then I call walk out of here and it’s
over.” The officer replied he could walk out whenever
 Time later Corporal Deveau took over and reminded Oickle of his rights. After 30-40
minutes, he confessed to torching Tanya’s car
 He was emotionally distraught, the police took a written statement and he continued to
deny involvement in any other fires
 The accused was then arrested, advised of his rights and taken to a police station. He was
very upset and crying
 He was put in room and videotaped while questioned for longer, and after a while he said
he was tired and wanted to go home and he was told that he was under arrest and couldn’t
do so
 Another Constable, Bogle, took over the interrogation, which had so far lasted 11 hours,
and he confessed to 7 of the 8 fires. He denied any involvement in his dad’s van
 He was very distraught. The police reminded him of his rights and took a written
statement, which lasted until 1 am. He was then put in a cell to sleep at 2:45 am and at
6am Corp Deveau asked if he would agree to a re-enactment. He was taken around the
town explaining how he set each fire
 He was charged with 7 counts of arson.
 At trial the voir dire was held by judge to determine if the statements and video reenactment were admissible. She held that they were voluntary and admissible and
convicted on all counts
 The CA allowed the appeal on grounds that statements were not voluntary. They entered
acquittals
Issue: Where the statements voluntary?
Holding: Yes; the appeal was allowed and the convictions restored
Iacobucci J
There are 2 strands to the confessions rule
 Under one approach any statement made by cops where there is an explicit promise or
threat to accused should be excluded (as in Ibrahim). This gives the accused a
negative right - i.e. the right not to be tortured or coerced into making a statement by
threats or promises held out to him by a person who is and whom he subjectively believes
to be a person in authority
 The decision in Hebert recognized a broader approach whereby the absence of violence
was not dispositive of the case – you still need the necessary mental element of
deciding between alternative options
The Charter does not subsume these common law rules regarding confessions. The
confessions rule has a broader scope than the Charter
 The Charter also has a different standard and burden of proof. Remedies under the charter
are different as well
Confessions rule today is concerned with voluntariness, broadly defined
110
 This is rooted in a concern about reliability
 It has twin goals: protecting the accused’s rights while not unduly limiting society’s
need to investigate and solve crimes
 Both of these objectives must be borne in mind in an analysis
One of the key concerns of the criminal justice system is that the innocent must not be convicted,
so the confession rule prevents the solicitation of false confessions
5 categories of confessions
 Voluntary
 Stress-compliant confessions
o The person will say anything to get rid of the stress
 The coerced compliant
o Coercion by threats or promises to confess
o Most cases of false confessions come out of these
o Most cases develop in relation to this
 Non-coerced persuaded
o Take someone who does not recall the events well and lead her to confess to
something didn’t happen
 Someone with mental challenges, who was intoxicated at the time, etc
o One of the ways to go about persuading someone to confess is through the
fabrication of false facts
o The would likely get the most false confessions from vulnerable people and those
who are easily influenced to agree with others
o You have to look at the strength of mind of the accused
 Coerced-persuaded
Confessions rule today must be applied contextually. The following factors should be
considered by trial judges in reviewing confessions:
 Threats or promises
o Reduced charge
o Minimizing the seriousness of the crime
o Offer psychological help
o Moral or spiritual inducements
 Oppression
o Denial of food, clothes, rest, medical attention, intimidating questioning
o Has a large impact on false confessions
 Operating mind
o Inspired by principle of voluntariness and the accused being aware of what she is
saying
 Other police trickery
o Shock to the community criteria (as per Lamer in Rothman)
Because of the overall concern about not convicting the innocent, a confession will not
admissible if it is made under circumstances that raise a reasonable doubt as to voluntariness
 Oppressive conditions and inducements can operate together to exclude confessions
111

Trial judges must pay attention to all circumstances when making this determination
In this case, there was nothing wrong with the confession and how it came about
 Oickle was fully aware of his rights at all times, he was never subjected to harsh,
aggressive or overbearing interrogation and not deprived of food, sleep or drink and was
not offered any improper inducements.
 With regards to the claim from CA that the cops minimized the seriousness of the crimes,
this was not a problem. Any sort of package deal was raised by Oickle and not the cops.
While the cops minimized the moral significance of the crimes, there was no suggestion
that they would minimize the legal consequences thereof
 CA held that the cops improperly offered psychological help in return for a confession.
Iacobucci points out that there was no suggestion that he would only be able to get help if
he confessed. Although there were moral inducements made, such as Oickle would feel
better if he confessed, there were no implied threats in this.
 With regards to alleged threats against his fiancée, although the cops did say they might
have to interview her, she was not threatened with harm
 CA also found the use of the polygraph test problematic. Merely failing to tell an accused
that the polygraph is inadmissible will not automatically produce an involuntary
confession. Courts need to do a 2-stage process. (I) it should be excluded if the police
deception shocks the community and (II) use of deception should be considered within
the rest of the overall voluntariness analysis. In this case, the police made it clear what
was admissible and what was not
 The police did exaggerate the validity of the polygraph but not to the extent that the
respondent was overwhelmed by the polygraph results
Lecture Notes
 Oickle has been seen as formally expanding the right to silence rule and defining the
operating mind standard from Hebert
o Yet the Court seems to place the bar very high for the accused to rely on this
right
 The Court gave some examples of instances when the confessions may be thrown out
o E.g. when a person is made a promise in relation to someone else
o Telling a person that if they do not confess they will go to hell
R. v. Singh, [2007] SCC 48 [Extra reading]
Facts
 The accused was arrested for second degree murder in respect of the shooting death of an
innocent bystander who was killed by a stray bullet while standing just inside the
doorway of a pub
 The accused was advised of his right to counsel under s. 10(b) of the Charter and
privately consulted with counsel
 During the course of two subsequent interviews with police, the accused stated on
numerous occasions that he did not want to talk about the incident. The interviewing
officer persisted in trying to get him to make a statement
 While the accused never confessed to the crime, he made a number of admissions which,
when taken together with other evidence, later became probative of the issue of
112
identification at trial
 On the voir dire to determine the admissibility of the statements made by the accused, the
trial judge held that the admission came freely and did not result from the police
systematically breaking down his operating mind or undermining his right to silence
 The probative value of the statements was held to outweigh their prejudicial effect and
the trial judge thus ruled them admissible
 The accused was subsequently convicted by a jury
 The Court of Appeal upheld the trial judge’s ruling and affirmed the conviction
 In the CA and in this appeal to the SCC the accused did not contest the trial judge’s
findings of fact nor his conclusion that the statements were voluntary; his appeal solely
concerns the s. 7 Charter right to silence
Issue: Did the admission of these statements violate the accused’s right to silence as per s. 7?
Holding: No; the appeal should be dismissed
Charron J
Although historically the confessions rule was more concerned with the reliability of
confessions than the protection against self-incrimination, this no longer holds true in the
post-Charter era
 The modern view of the confessions rule includes the right of the detained person to
make a meaningful choice whether or not to speak to state authorities
 On the question of voluntariness, the focus is on the conduct of the police and its effect
on the accused’s ability to exercise her free will
o The test is an objective one, but the individual characteristics of the accused
are obviously relevant considerations in applying this objective test
o After detention, once the accused cannot walk away, she is in a more vulnerable
position
In the context of an interrogation of a detainee by an obvious person in authority, a finding of
voluntariness will be determinative of the s. 7 issue
 In such circumstances, the confessions rule effectively subsumes the constitutional
right to silence because the two tests are functionally equivalent
o If a statement has survived a thorough inquiry into voluntariness, the accused’s
Charter application alleging a violation of the pre-trial right to silence under s. 7
cannot succeed
o Conversely, if circumstances are such that the accused can show on a balance of
probabilities that the statement was obtained in violation of her constitutional
right to remain silent, the Crown will be unable to prove voluntariness beyond a
reasonable doubt
It is not appropriate to impose a rigid requirement that police refrain from questioning a
detainee who states that he or she does not wish to speak to police
 Under both the common law and Charter rules, police persistence in continuing an
interview, despite repeated assertions by the detainee that he wishes to remain silent, may
well raise a strong argument that the subsequently obtained statement was not the product
of a free will to speak to authorities
 The trial judge in this case was very much alive to this risk. His ultimate judgment call
113
on this issue is supported by the record and is entitled to deference
Fish J
The interrogator understood very well that the accused had chosen not to speak with the police
but nonetheless disregarded the accused’s repeated assertions of his right to silence
 In his relentless pursuit of a confession “no matter what”, the interrogator urged the
accused, subtly but unmistakably, to forsake his counsel’s advice
 While detainees who have asserted their right to silence are entitled to change their
minds, they cannot be compelled to do so by the persistent disregard of that asserted
choice
A confession may be “voluntary” under the common law rule and yet be obtained by state action
that infringes s. 7 of the Charter
 A confession that meets these common law standards does not invariably represent a free
and meaningful choice for the purposes of the Charter
Why is there a doctrine of waiver to right to counsel, but not to the right to silence?
 According to McLachlin, in Hebert, in order to be able to waive a right, you need to be
able to understand the full circumstances surrounding the waiver
 If there was a doctrine of waiver in the right of silence, this would undermine the power
of the police to use undercover police officers or other types of trickery
Unfortunately Singh did not resolve the discrepancies surrounding this issue
 The court found that the voluntary confessions rules and the right to counsel were the
same
Bail Hearings
At some point the investigation of a crime would allow the State to lay charges against someone
 Here the State will try to find a way to compel the person to appear in court
In lieu of arresting the person until the train, the state can offer a notice to appear or a
recognizance
 A recognizance is an acknowledgment of the debt that person will owe if they do not
appear in court
After this procedure happens, an information will be laid in front of a judge, which establishes
the reasons for the person’s arrest
 The judge must receive this information as part of her administrative functions
o She cannot amend it
 Only then can the judge confirm the processes issued by the police earlier
o E.g. the requirement to appear
 Once the judge has issued this order, the accused will appear in court for arraignment or
the first appearance
114
o People who are not offered bail remain in custody until the trial
o The term used in the Code is judicial interim release, though bail is more largely
used commonly
o Generally speaking the burden is on the Crown to prove that the person should not
be released, though under some circumstances the onus can be reversed
 Depending on the offence, a decision will be reached by the judge
The rational for the Bail Reform Act of 1961 was mainly twofold
 Minimize incidence of arrest
 Minimize detention before trial
 Research found that when people were being held without bail (independent of the
seriousness of the offence) convictions were more common, as well as being sentenced to
serve time in prison
o Pre-trial detention could lead to more accused people pleading guilty and agreeing
to harsher sentences
o Pre-trial detention may limit the ability of the accused to gather evidence and
build her case
o Someone who is incarcerated cannot earn an income, thus may not be able to
afford a good lawyer
Originally the onus was always on the Crown to show why the accused should be held in prison
 The Reform Act removed this presumption for accused individuals charged with specific
offences
Central criteria for judicial interim release are set out in s. 515(10)
 The presumption in s.515 is that the individual should be let out without any
conditions, except if it is shown that conditions are necessary because of matters of:
o Flight risk
o Public safety
o Public confidence on the administration of justice
S. 515 (2) lists the types of bail orders that may made, depending on how onerous they are
 In terms of a money order that will need to be paid, the Code is silent as to its amount
 The onus is always on the Crown to show why a less restrictive means is not adequate
Intake Procedures
s. 503(1) – Criminal Code
Taking before justice
A peace officer who arrests a person with or without warrant or to whom a person is delivered
under subsection 494(3) or into whose custody a person is placed under subsection 163.5(3) of
the Customs Act shall cause the person to be detained in custody and, in accordance with the
following provisions, to be taken before a justice to be dealt with according to law:
115
(a) where a justice is available within a period of twenty-four hours after the person
has been arrested by or delivered to the peace officer, the person shall be taken before a
justice without unreasonable delay and in any event within that period, and
(b) where a justice is not available within a period of twenty-four hours after the person
has been arrested by or delivered to the peace officer, the person shall be taken before a
justice as soon as possible,
unless, at any time before the expiration of the time prescribed in paragraph (a) or (b) for taking
the person before a justice,
(c) the peace officer or officer in charge releases the person under any other provision of
this Part, or
(d) the peace officer or officer in charge is satisfied that the person should be released
from custody, whether unconditionally under subsection (4) or otherwise conditionally
or unconditionally, and so releases him.
s. 504 – Criminal Code
In what cases justice may receive information
Any one who, on reasonable grounds, believes that a person has committed an indictable
offence may lay an information in writing and under oath before a justice, and the justice shall
receive the information, where it is alleged
(a) that the person has committed, anywhere, an indictable offence that may be tried in
the province in which the justice resides, and that the person
(i) is or is believed to be, or
(ii) resides or is believed to reside,
within the territorial jurisdiction of the justice;
(b) that the person, wherever he may be, has committed an indictable offence within the
territorial jurisdiction of the justice;
(c) that the person has, anywhere, unlawfully received property that was unlawfully
obtained within the territorial jurisdiction of the justice; or
(d) that the person has in his possession stolen property within the territorial jurisdiction
of the justice.
s. 505 – Criminal Code
Time within which information to be laid in certain cases
Where
(a) an appearance notice has been issued to an accused under section 496, or
(b) an accused has been released from custody under section 497 or 498,
an information relating to the offence alleged to have been committed by the accused or relating
to an included or other offence alleged to have been committed by him shall be laid before a
justice as soon as practicable thereafter and in any event before the time stated in the
appearance notice, promise to appear or recognizance issued to or given or entered into by the
116
accused for his attendance in court.
s. 506 – Criminal Code
Forms
An information laid under section 504 or 505 may be in Form 2.
s. 507 – Criminal Code
Justice to hear informant and witnesses — public prosecutions
(1) Subject to subsection 523(1.1), a justice who receives an information laid under section 504
by a peace officer, a public officer, the Attorney General or the Attorney General’s agent, other
than an information laid before the justice under section 505, shall, except if an accused has
already been arrested with or without a warrant,
(a) hear and consider, ex parte,
(i) the allegations of the informant, and
(ii) the evidence of witnesses, where he considers it desirable or necessary to do
so; and
(b) where he considers that a case for so doing is made out, issue, in accordance with
this section, either a summons or a warrant for the arrest of the accused to compel the
accused to attend before him or some other justice for the same territorial division to
answer to a charge of an offence.
Process compulsory
(2) No justice shall refuse to issue a summons or warrant by reason only that the alleged offence
is one for which a person may be arrested without warrant.
Procedure when witnesses attend
(3) A justice who hears the evidence of a witness pursuant to subsection (1) shall
(a) take the evidence on oath; and
(b) cause the evidence to be taken in accordance with section 540 in so far as that
section is capable of being applied.
Summons to be issued except in certain cases
(4) Where a justice considers that a case is made out for compelling an accused to attend before
him to answer to a charge of an offence, he shall issue a summons to the accused unless the
allegations of the informant or the evidence of any witness or witnesses taken in accordance
with subsection (3) discloses reasonable grounds to believe that it is necessary in the public
interest to issue a warrant for the arrest of the accused.
No process in blank
(5) A justice shall not sign a summons or warrant in blank.
117
Endorsement of warrant by justice
(6) A justice who issues a warrant under this section or section 508 or 512 may, unless the
offence is one mentioned in section 522, authorize the release of the accused pursuant to section
499 by making an endorsement on the warrant in Form 29.
Promise to appear or recognizance deemed to have been confirmed
(7) Where, pursuant to subsection (6), a justice authorizes the release of an accused pursuant to
section 499, a promise to appear given by the accused or a recognizance entered into by the
accused pursuant to that section shall be deemed, for the purposes of subsection 145(5), to have
been confirmed by a justice under section 508.
Issue of summons or warrant
(8) Where, on an appeal from or review of any decision or matter of jurisdiction, a new trial or
hearing or a continuance or renewal of a trial or hearing is ordered, a justice may issue either a
summons or a warrant for the arrest of the accused in order to compel the accused to attend at
the new or continued or renewed trial or hearing.
 Subsection 4 shows that summons are preferred to a warrant of arrest unless necessary in
the public interest
o See also s. 515
s. 507.1 – Criminal Code
Referral when private prosecution
(1) A justice who receives an information laid under section 504, other than an information
referred to in subsection 507(1), shall refer it to a provincial court judge or, in Quebec, a judge
of the Court of Quebec, or to a designated justice, to consider whether to compel the appearance
of the accused on the information.
Summons or warrant
(2) A judge or designated justice to whom an information is referred under subsection (1) and
who considers that a case for doing so is made out shall issue either a summons or warrant for
the arrest of the accused to compel him or her to attend before a justice to answer to a charge of
the offence charged in the information.
Conditions for issuance
(3) The judge or designated justice may issue a summons or warrant only if he or she
(a) has heard and considered the allegations of the informant and the evidence of
witnesses;
(b) is satisfied that the Attorney General has received a copy of the information;
(c) is satisfied that the Attorney General has received reasonable notice of the hearing
118
under paragraph (a); and
(d) has given the Attorney General an opportunity to attend the hearing under paragraph
(a) and to cross-examine and call witnesses and to present any relevant evidence at the
hearing.
Appearance of Attorney General
(4) The Attorney General may appear at the hearing held under paragraph (3)(a) without being
deemed to intervene in the proceeding.
Information deemed not to have been laid
(5) If the judge or designated justice does not issue a summons or warrant under subsection (2),
he or she shall endorse the information with a statement to that effect. Unless the informant, not
later than six months after the endorsement, commences proceedings to compel the judge or
designated justice to issue a summons or warrant, the information is deemed never to have been
laid.
Information deemed not to have been laid — proceedings commenced
(6) If proceedings are commenced under subsection (5) and a summons or warrant is not issued
as a result of those proceedings, the information is deemed never to have been laid.
New evidence required for new hearing
(7) If a hearing in respect of an offence has been held under paragraph (3)(a) and the judge or
designated justice has not issued a summons or a warrant, no other hearings may be held under
that paragraph with respect to the offence or an included offence unless there is new evidence in
support of the allegation in respect of which the hearing is sought to be held.
Subsections 507(2) to (8) to apply
(8) Subsections 507(2) to (8) apply to proceedings under this section.
Non-application — informations laid under sections 810 and 810.1
(9) Subsections (1) to (8) do not apply in respect of an information laid under section 810 or
810.1.
Definition of “designated justice”
(10) In this section, “designated justice” means a justice designated for the purpose by the chief
judge of the provincial court having jurisdiction in the matter or, in Quebec, a justice designated
by the chief judge of the Court of Quebec.
s. 508 – Criminal Code
119
Justice to hear informant and witnesses
(1) A justice who receives an information laid before him under section 505 shall
(a) hear and consider, ex parte,
(i) the allegations of the informant, and
(ii) the evidence of witnesses, where he considers it desirable or necessary to do
so;
(b) where he considers that a case for so doing is made out, whether the information
relates to the offence alleged in the appearance notice, promise to appear or
recognizance or to an included or other offence,
(i) confirm the appearance notice, promise to appear or recognizance, as the case
may be, and endorse the information accordingly, or
(ii) cancel the appearance notice, promise to appear or recognizance, as the case
may be, and issue, in accordance with section 507, either a summons or a
warrant for the arrest of the accused to compel the accused to attend before him
or some other justice for the same territorial division to answer to a charge of an
offence and endorse on the summons or warrant that the appearance notice,
promise to appear or recognizance, as the case may be, has been cancelled; and
(c) where he considers that a case is not made out for the purposes of paragraph (b),
cancel the appearance notice, promise to appear or recognizance, as the case may be,
and cause the accused to be notified forthwith of the cancellation.
Procedure when witnesses attend
(2) A justice who hears the evidence of a witness pursuant to subsection (1) shall
(a) take the evidence on oath; and
(b) cause the evidence to be taken in accordance with section 540 in so far as that
section is capable of being applied.
s. 515 – Criminal Code
Order of release
(1) Subject to this section, where an accused who is charged with an offence other than an
offence listed in section 469 is taken before a justice, the justice shall, unless a plea of guilty by
the accused is accepted, order, in respect of that offence, that the accused be released on his
giving an undertaking without conditions, unless the prosecutor, having been given a
reasonable opportunity to do so, shows cause, in respect of that offence, why the detention
of the accused in custody is justified or why an order under any other provision of this section
should be made and where the justice makes an order under any other provision of this section,
the order shall refer only to the particular offence for which the accused was taken before the
justice.
Release on undertaking with conditions, etc.
(2) Where the justice does not make an order under subsection (1), he shall, unless the
prosecutor shows cause why the detention of the accused is justified, order that the
accused be released
120
(a) on his giving an undertaking with such conditions as the justice directs;
(b) on his entering into a recognizance before the justice, without sureties, in such
amount and with such conditions, if any, as the justice directs but without deposit of
money or other valuable security;
(c) on his entering into a recognizance before the justice with sureties in such amount
and with such conditions, if any, as the justice directs but without deposit of money or
other valuable security;
(d) with the consent of the prosecutor, on his entering into a recognizance before the
justice, without sureties, in such amount and with such conditions, if any, as the justice
directs and on his depositing with the justice such sum of money or other valuable
security as the justice directs; or
(e) if the accused is not ordinarily resident in the province in which the accused is in
custody or does not ordinarily reside within two hundred kilometres of the place in
which he is in custody, on his entering into a recognizance before the justice with or
without sureties in such amount and with such conditions, if any, as the justice directs,
and on his depositing with the justice such sum of money or other valuable security as
the justice directs.
Power of justice to name sureties in order
(2.1) Where, pursuant to subsection (2) or any other provision of this Act, a justice, judge or
court orders that an accused be released on his entering into a recognizance with sureties, the
justice, judge or court may, in the order, name particular persons as sureties.
Alternative to physical presence
(2.2) Where, by this Act, the appearance of an accused is required for the purposes of judicial
interim release, the appearance shall be by actual physical attendance of the accused but the
justice may, subject to subsection (2.3), allow the accused to appear by means of any suitable
telecommunication device, including telephone, that is satisfactory to the justice.
Where consent required
(2.3) The consent of the prosecutor and the accused is required for the purposes of an
appearance if the evidence of a witness is to be taken at the appearance and the accused cannot
appear by closed-circuit television or any other means that allow the court and the accused to
engage in simultaneous visual and oral communication.
Idem
(3) The justice shall not make an order under any of paragraphs (2)(b) to (e) unless the
prosecution shows cause why an order under the immediately preceding paragraph should not
be made.
Conditions authorized
121
(4) The justice may direct as conditions under subsection (2) that the accused shall do any
one or more of the following things as specified in the order:
(a) report at times to be stated in the order to a peace officer or other person designated
in the order;
(b) remain within a territorial jurisdiction specified in the order;
(c) notify the peace officer or other person designated under paragraph (a) of any change
in his address or his employment or occupation;
(d) abstain from communicating, directly or indirectly, with any victim, witness or other
person identified in the order, or refrain from going to any place specified in the order,
except in accordance with the conditions specified in the order that the justice considers
necessary;
(e) where the accused is the holder of a passport, deposit his passport as specified in the
order;
(e.1) comply with any other condition specified in the order that the justice considers
necessary to ensure the safety and security of any victim of or witness to the offence;
and
(f) comply with such other reasonable conditions specified in the order as the justice
considers desirable.
Condition prohibiting possession of firearms, etc.
(4.1) When making an order under subsection (2), in the case of an accused who is charged with
(a) an offence in the commission of which violence against a person was used,
threatened or attempted,
(a.1) a terrorism offence,
(b) an offence under section 264 (criminal harassment),
(b.1) an offence under section 423.1 (intimidation of a justice system participant),
(c) an offence relating to the contravention of subsection 5(3) or (4), 6(3) or 7(2) of the
Controlled Drugs and Substances Act,
(d) an offence that involves, or the subject-matter of which is, a firearm, a cross-bow, a
prohibited weapon, a restricted weapon, a prohibited device, ammunition, prohibited
ammunition or an explosive substance, or
(e) an offence under subsection 20(1) of the Security of Information Act, or an offence
under subsection 21(1) or 22(1) or section 23 of that Act that is committed in relation to
on offence under subsection 20(1) of that Act,
the justice shall add to the order a condition prohibiting the accused from possessing a firearm,
cross-bow, prohibited weapon, restricted weapon, prohibited device, ammunition, prohibited
ammunition or explosive substance, or all those things, until the accused is dealt with according
to law unless the justice considers that such a condition is not required in the interests of the
safety of the accused or the safety and security of a victim of the offence or of any other person.
Surrender, etc.
(4.11) Where the justice adds a condition described in subsection (4.1) to an order made under
subsection (2), the justice shall specify in the order the manner and method by which
122
(a) the things referred to in subsection (4.1) that are in the possession of the accused
shall be surrendered, disposed of, detained, stored or dealt with; and
(b) the authorizations, licences and registration certificates held by the person shall be
surrendered.
Reasons
(4.12) Where the justice does not add a condition described in subsection (4.1) to an order made
under subsection (2), the justice shall include in the record a statement of the reasons for not
adding the condition.
Additional conditions
(4.2) Before making an order under subsection (2), in the case of an accused who is charged
with an offence referred to in subsection (4.3), the justice shall consider whether it is desirable,
in the interests of the safety and security of any person, particularly a victim of or witness to the
offence or a justice system participant, to include as a condition of the order
(a) that the accused abstain from communicating, directly or indirectly, with any victim,
witness or other person identified in the order, or refrain from going to any place
specified in the order; or
(b) that the accused comply with any other condition specified in the order that the
justice considers necessary to ensure the safety and security of those persons.
Offences
(4.3) The offences for the purposes of subsection (4.2) are
(a) a terrorism offence;
(b) an offence described in section 264 or 423.1;
(c) an offence in the commission of which violence against a person was used,
threatened or attempted; and
(d) an offence under subsection 20(1) of the Security of Information Act, or an offence
under subsection 21(1) or 22(1) or section 23 of that Act that is committed in relation to
an offence under subsection 20(1) of that Act.
Detention in custody
(5) Where the prosecutor shows cause why the detention of the accused in custody is justified,
the justice shall order that the accused be detained in custody until he is dealt with according to
law and shall include in the record a statement of his reasons for making the order.
Order of detention
(6) Notwithstanding any provision of this section, where an accused is charged
(a) with an indictable offence, other than an offence listed in section 469,
(i) that is alleged to have been committed while at large after being released in
123
respect of another indictable offence pursuant to the provisions of this Part or
section 679 or 680,
(ii) that is an offence under section 467.11, 467.12 or 467.13, or a serious
offence alleged to have been committed for the benefit of, at the direction of, or
in association with, a criminal organization,
(iii) that is an offence under any of sections 83.02 to 83.04 and 83.18 to 83.23 or
otherwise is alleged to be a terrorism offence,
(iv) an offence under subsection 16(1) or (2), 17(1), 19(1), 20(1) or 22(1) of the
Security of Information Act, or
(v) an offence under subsection 21(1) or 22(1) or section 23 of the Security of
Information Act that is committed in relation to on offence referred to in
subparagraph (iv),
(b) with an indictable offence, other than an offence listed in section 469 and is not
ordinarily resident in Canada,
(c) with an offence under any of subsections 145(2) to (5) that is alleged to have been
committed while he was at large after being released in respect of another offence
pursuant to the provisions of this Part or section 679, 680 or 816, or
(d) with having committed an offence punishable by imprisonment for life under
subsection 5(3), 6(3) or 7(2) of the Controlled Drugs and Substances Act or the offence
of conspiring to commit such an offence,
the justice shall order that the accused be detained in custody until he is dealt with according to
law, unless the accused, having been given a reasonable opportunity to do so, shows cause
why his detention in custody is not justified, but where the justice orders that the accused be
released, he shall include in the record a statement of his reasons for making the order.
Order of release
(7) Where an accused to whom paragraph 6(a), (c) or (d) applies shows cause why the
accused’s detention in custody is not justified, the justice shall order that the accused be
released on giving an undertaking or entering into a recognizance described in any of
paragraphs (2)(a) to (e) with the conditions described in subsections (4) to (4.2) or, where the
accused was at large on an undertaking or recognizance with conditions, the additional
conditions described in subsections (4) to (4.2), that the justice considers desirable, unless the
accused, having been given a reasonable opportunity to do so, shows cause why the conditions
or additional conditions should not be imposed.
Idem
(8) Where an accused to whom paragraph (6)(b) applies shows cause why the accused’s
detention in custody is not justified, the justice shall order that the accused be released on
giving an undertaking or entering into a recognizance described in any of paragraphs (2)(a) to
(e) with the conditions, described in subsections (4) to (4.2), that the justice considers desirable.
Sufficiency of record
124
(9) For the purposes of subsections (5) and (6), it is sufficient if a record is made of the reasons
in accordance with the provisions of Part XVIII relating to the taking of evidence at preliminary
inquiries.
Justification for detention in custody
(10) For the purposes of this section, the detention of an accused in custody is justified only
on one or more of the following grounds:
(a) where the detention is necessary to ensure his or her attendance in court in order to
be dealt with according to law;
(b) where the detention is necessary for the protection or safety of the public, including
any victim of or witness to the offence, having regard to all the circumstances including
any substantial likelihood that the accused will, if released from custody, commit a
criminal offence or interfere with the administration of justice; and
(c) on any other just cause being shown and, without limiting the generality of the
foregoing, where the detention is necessary in order to maintain confidence in the
administration of justice, having regard to all the circumstances, including the apparent
strength of the prosecution’s case, the gravity of the nature of the offence, the
circumstances surrounding its commission and the potential for a lengthy term of
imprisonment.
Detention in custody for offence listed in section 469
(11) Where an accused who is charged with an offence mentioned in section 469 is taken before
a justice, the justice shall order that the accused be detained in custody until he is dealt
with according to law and shall issue a warrant in Form 8 for the committal of the
accused.
Order re no communication
(12) A justice who orders that an accused be detained in custody under this section may include
in the order a direction that the accused abstain from communicating, directly or indirectly, with
any victim, witness or other person identified in the order, except in accordance with such
conditions specified in the order as the justice considers necessary.
 The offences listed under s. 469 include treason, intimidating Parliament, and murder
o Subsection 10 requires custody when an accused is charged with these
o The procedure under s. 522 stipulates when an accused may be released when
charged with these types of offences
 Subsection 6 reverses the onus and forces the defendant to justify release if the indictable
offence (other than those listed in s. 469) was committed within the context of certain
circumstances
s. 516 – Criminal Code
Remand in custody
(1) A justice may, before or at any time during the course of any proceedings under section 515,
125
on application by the prosecutor or the accused, adjourn the proceedings and remand the
accused to custody in prison by warrant in Form 19, but no adjournment shall be for more than
three clear days except with the consent of the accused.
Detention pending bail hearing
(2) A justice who remands an accused to custody under subsection (1) or subsection 515(11)
may order that the accused abstain from communicating, directly or indirectly, with any victim,
witness or other person identified in the order, except in accordance with any conditions
specified in the order that the justice considers necessary.
s. 517 – Criminal Code
Order directing matters not to be published for specified period
(1) If the prosecutor or the accused intends to show cause under section 515, he or she shall so
state to the justice and the justice may, and shall on application by the accused, before or at any
time during the course of the proceedings under that section, make an order directing that the
evidence taken, the information given or the representations made and the reasons, if any, given
or to be given by the justice shall not be published in any document, or broadcast or transmitted
in any way before such time as
(a) if a preliminary inquiry is held, the accused in respect of whom the proceedings are
held is discharged; or
(b) if the accused in respect of whom the proceedings are held is tried or ordered to
stand trial, the trial is ended.
Failure to comply
(2) Every one who fails without lawful excuse, the proof of which lies on him, to comply with
an order made under subsection (1) is guilty of an offence punishable on summary conviction.
s. 518 – Criminal Code
Inquiries to be made by justice and evidence
(1) In any proceedings under section 515,
(a) the justice may, subject to paragraph (b), make such inquiries, on oath or otherwise,
of and concerning the accused as he considers desirable;
(b) the accused shall not be examined by the justice or any other person except
counsel for the accused respecting the offence with which the accused is charged,
and no inquiry shall be made of the accused respecting that offence by way of
cross-examination unless the accused has testified respecting the offence;
(c) the prosecutor may, in addition to any other relevant evidence, lead evidence
(i) to prove that the accused has previously been convicted of a criminal offence,
(ii) to prove that the accused has been charged with and is awaiting trial for
another criminal offence,
(iii) to prove that the accused has previously committed an offence under section
145, or
126
(iv) to show the circumstances of the alleged offence, particularly as they relate
to the probability of conviction of the accused;
(d) the justice may take into consideration any relevant matters agreed on by the
prosecutor and the accused or his counsel;
(d.1) the justice may receive evidence obtained as a result of an interception of a private
communication under and within the meaning of Part VI, in writing, orally or in the
form of a recording and, for the purposes of this section, subsection 189(5) does not
apply to that evidence;
(d.2) the justice shall take into consideration any evidence submitted regarding the need
to ensure the safety or security of any victim of or witness to an offence; and
(e) the justice may receive and base his decision on evidence considered credible or
trustworthy by him in the circumstances of each case.
Release pending sentence
(2) Where, before or at any time during the course of any proceedings under section 515, the
accused pleads guilty and that plea is accepted, the justice may make any order provided for in
this Part for the release of the accused until the accused is sentenced.
 Subsection 1 addresses the procedure during the bail hearing
 Subscetion2 discusses the issue of release pending evidence
s. 519 – Criminal Code
Release of accused
(1) Where a justice makes an order under subsection 515(1), (2), (7) or (8),
(a) if the accused thereupon complies with the order, the justice shall direct that the
accused be released
(i) forthwith, if the accused is not required to be detained in custody in respect of
any other matter, or
(ii) as soon thereafter as the accused is no longer required to be detained in
custody in respect of any other matter; and
(b) if the accused does not thereupon comply with the order, the justice who made the
order or another justice having jurisdiction shall issue a warrant for the committal of the
accused and may endorse thereon an authorization to the person having the custody of
the accused to release the accused when the accused complies with the order
(i) forthwith after the compliance, if the accused is not required to be detained in
custody in respect of any other matter, or
(ii) as soon thereafter as the accused is no longer required to be detained in
custody in respect of any other matter
and if the justice so endorses the warrant, he shall attach to it a copy of the order.
Discharge from custody
(2) Where the accused complies with an order referred to in paragraph (1)(b) and is not required
to be detained in custody in respect of any other matter, the justice who made the order or
127
another justice having jurisdiction shall, unless the accused has been or will be released
pursuant to an authorization referred to in that paragraph, issue an order for discharge in Form
39.
Warrant for committal
(3) Where the justice makes an order under subsection 515(5) or (6) for the detention of the
accused, he shall issue a warrant for the committal of the accused.
s. 520 – Criminal Code
Review of order
(1) If a justice, or a judge of the Nunavut Court of Justice, makes an order under subsection
515(2), (5), (6), (7), (8) or (12) or makes or vacates any order under paragraph 523(2)(b), the
accused may, at any time before the trial of the charge, apply to a judge for a review of the
order.
Notice to prosecutor
(2) An application under this section shall not, unless the prosecutor otherwise consents, be
heard by a judge unless the accused has given to the prosecutor at least two clear days notice in
writing of the application.
Accused to be present
(3) If the judge so orders or the prosecutor or the accused or his counsel so requests, the accused
shall be present at the hearing of an application under this section and, where the accused is in
custody, the judge may order, in writing, the person having the custody of the accused to bring
him before the court.
Adjournment of proceedings
(4) A judge may, before or at any time during the hearing of an application under this section,
on application by the prosecutor or the accused, adjourn the proceedings, but if the accused is in
custody no adjournment shall be for more than three clear days except with the consent of the
accused.
Failure of accused to attend
(5) Where an accused, other than an accused who is in custody, has been ordered by a judge to
be present at the hearing of an application under this section and does not attend the hearing, the
judge may issue a warrant for the arrest of the accused.
Execution
(6) A warrant issued under subsection (5) may be executed anywhere in Canada.
128
Evidence and powers of judge on review
(7) On the hearing of an application under this section, the judge may consider
(a) the transcript, if any, of the proceedings heard by the justice and by any judge who
previously reviewed the order made by the justice,
(b) the exhibits, if any, filed in the proceedings before the justice, and
(c) such additional evidence or exhibits as may be tendered by the accused or the
prosecutor,
and shall either
(d) dismiss the application, or
(e) if the accused shows cause, allow the application, vacate the order previously made
by the justice and make any other order provided for in section 515 that he considers is
warranted.
Limitation of further applications
(8) Where an application under this section or section 521 has been heard, a further or other
application under this section or section 521 shall not be made with respect to that same
accused, except with leave of a judge, prior to the expiration of thirty days from the date of the
decision of the judge who heard the previous application.
Application of sections 517, 518 and 519
(9) The provisions of sections 517, 518 and 519 apply with such modifications as the
circumstances require in respect of an application under this section.
s. 521 – Criminal Code
Review of order
(1) If a justice, or a judge of the Nunavut Court of Justice, makes an order under subsection
515(1), (2), (7), (8) or (12) or makes or vacates any order under paragraph 523(2)(b), the
prosecutor may, at any time before the trial of the charge, apply to a judge for a review of the
order.
Notice to accused
(2) An application under this section shall not be heard by a judge unless the prosecutor has
given to the accused at least two clear days notice in writing of the application.
Accused to be present
(3) If the judge so orders or the prosecutor or the accused or his counsel so requests, the accused
shall be present at the hearing of an application under this section and, where the accused is in
custody, the judge may order, in writing, the person having the custody of the accused to bring
129
him before the court.
Adjournment of proceedings
(4) A judge may, before or at any time during the hearing of an application under this section,
on application of the prosecutor or the accused, adjourn the proceedings, but if the accused is in
custody no adjournment shall be for more than three clear days except with the consent of the
accused.
Failure of accused to attend
(5) Where an accused, other than an accused who is in custody, has been ordered by a judge to
be present at the hearing of an application under this section and does not attend the hearing, the
judge may issue a warrant for the arrest of the accused.
Warrant for detention
(6) Where, pursuant to paragraph (8)(e), the judge makes an order that the accused be detained
in custody until he is dealt with according to law, he shall, if the accused is not in custody, issue
a warrant for the committal of the accused.
Execution
(7) A warrant issued under subsection (5) or (6) may be executed anywhere in Canada.
Evidence and powers of judge on review
(8) On the hearing of an application under this section, the judge may consider
(a) the transcript, if any, of the proceedings heard by the justice and by any judge who
previously reviewed the order made by the justice,
(b) the exhibits, if any, filed in the proceedings before the justice, and
(c) such additional evidence or exhibits as may be tendered by the prosecutor or the
accused,
and shall either
(d) dismiss the application, or
(e) if the prosecutor shows cause, allow the application, vacate the order previously
made by the justice and make any other order provided for in section 515 that he
considers to be warranted.
Limitation of further applications
(9) Where an application under this section or section 520 has been heard, a further or other
application under this section or section 520 shall not be made with respect to the same accused,
except with leave of a judge, prior to the expiration of thirty days from the date of the decision
130
of the judge who heard the previous application.
Application of sections 517, 518 and 519
(10) The provisions of sections 517, 518 and 519 apply with such modifications as the
circumstances require in respect of an application under this section.
s. 522 – Criminal Code
Interim release by judge only
(1) Where an accused is charged with an offence listed in section 469, no court, judge or
justice, other than a judge of or a judge presiding in a superior court of criminal
jurisdiction for the province in which the accused is so charged, may release the accused
before or after the accused has been ordered to stand trial.
Idem
(2) Where an accused is charged with an offence listed in section 469, a judge of or a judge
presiding in a superior court of criminal jurisdiction for the province in which the accused is
charged shall order that the accused be detained in custody unless the accused, having
been given a reasonable opportunity to do so, shows cause why his detention in custody is
not justified within the meaning of subsection 515(10).
Order re no communication
(2.1) A judge referred to in subsection (2) who orders that an accused be detained in custody
under this section may include in the order a direction that the accused abstain from
communicating, directly or indirectly, with any victim, witness or other person identified in the
order except in accordance with such conditions specified in the order as the judge considers
necessary.
Release of accused
(3) If the judge does not order that the accused be detained in custody under subsection (2), the
judge may order that the accused be released on giving an undertaking or entering into a
recognizance described in any of paragraphs 515(2)(a) to (e) with such conditions described in
subsections 515(4), (4.1) and (4.2) as the judge considers desirable.
Order not reviewable except under section 680
(4) An order made under this section is not subject to review, except as provided in section
680.
Application of sections 517, 518 and 519
(5) The provisions of sections 517, 518 except subsection (2) thereof, and 519 apply with such
131
modifications as the circumstances require in respect of an application for an order under
subsection (2).
Other offences
(6) Where an accused is charged with an offence mentioned in section 469 and with any other
offence, a judge acting under this section may apply the provisions of this Part respecting
judicial interim release to that other offence.
 The offences listed under s. 469 include treason, intimidating Parliament, and murder
o Subsection 2 lays down the procedure under which an accused may be released
when charged with these types of offences (satisfying the reversed onus imposed
on her)
s. 795 – Criminal Code
Application of Parts XVI, XVIII, XX and XX.1
The provisions of Parts XVI and XVIII with respect to compelling the appearance of an accused
before a justice, and the provisions of Parts XX and XX.1, in so far as they are not inconsistent
with this Part, apply, with such modifications as the circumstances require, to proceedings
under this Part.
 This section appears under Part XXVII: Summary Convictions
Show cause hearings
The new legislation’s emphasis on release is nowhere more evident than in s. 515
 Previously the accused would have to petition for her release
 Now the legislation requires that the accused should be release without conditions, unless
the prosecutor shows cause why the release should not be ordered
o The section follows a ladder effect
R. v. Thompson (1972), 18 CRNS 102 (BC SC)
Facts
 The accused lost control of his car and it collided with a tree
Issue: Did the roadside alcohol test constitute detention under s. 10 of the Charter?
Holding: Yes; appeal dismissed
Anderson J
Whereas under the old system the onus was on an accused to show that he was entitled to
be released on bail, the new legislation places on the Crown the burden of justifying the
detention of an accused
 The legislation as a whole is to be liberally interpreted so as to carry out the intention of
Parliament
132
The fact that a person has no roots in the community is not, of itself, a sufficient ground
for denying a release without conditions, nor is the question of deterrence relevant in
considering release pending trial
A justice who makes an order refusing to release an accused is required to give his reasons
therefor, both for the protection of the accused and for the assistance of a reviewing court
Lecture Notes
 At common law, only flight risk and public safety were reasons for denying bail
 The third ground in Canada is anomalous amongst developed nations
o Does not exist in the UK or the US
 Before 1967, there were two grounds
o Compelling appearance
o Public interest
 It can then be argued that the third ground that exists now derives from
this second former legal ground
Grounds for Detention
Re Powers and R. (1973), 20 CRNS 23 (Ont. HC)
Lerner J
Under s. 457(7), detention for the purpose of ensuring attendance in court includes
consideration of such things as residence, fixed place of abode, employment or occupation,
marital and family status, previous criminal record, proximity of close friends and relatives,
character witnesses, facts relating to the allegations of the offence, personal history or vitae
In the instant case the accused was alleged to have committed a series of drug offences while on
bail such that it could not be said that she would likely commit an offence involving serious
harm or interference with the administration of justice
The expression, "detention is necessary in the public interest", in s. 457(7)(b), is a separate
ground
Public interest" includes the "public image" of the Criminal Code, the bail amendments,
the apprehension and conviction of criminals, deterrence of crime and public protection
 The application of the bail provisions should not appear to be a mockery
 The public must be allowed to feel safe and secure
The plain lessons of human experience must be recognized and applied to these bail procedures.
Lectures notes
 The court interprets public interest as something including deterrence and people not
feeling that the application of our criminal laws are a mockery
R. v. Graham, Ont. Dist. Ct., August 2 1990
133
Fleury J
In our modern society, where all sectors have become aware of the prevalence of sexual abuse,
it is clearly in the public interest to discourage any type of sexual abuse
The context of reasonable bail as outlined in the Charter must be understood in the context
of the present provisions of the Criminal Code
It is clear that there are circumstances where there is no substantial likelihood of the
commission of a criminal offence which might still justify a detention order in the public
interest
The principles outlined in Powers have not changed despite the advent of the Charter
Lectures notes
 Here the court interprets public interest as something based on how much the particular
offence is considered a social problem
R. v. Lamothe (1990), 77 CR (3d) 236
Facts
 The accused was charged with complicity after the fact in a case of murder
 Under s. 522 of the Code, he had to convince the court that his detention was not
required for either of the reasons in s. 515(10)
 The Superior Court was satisfied that his detention was not required to assure his
presence in court, but concluded that he should be detained because of the probability
that he would be convicted of a very serious crime
 The accused appealed.
Issue: Does the seriousness of the crime justify a detention in the public interest?
Holding: No; appeal allowed
Baudoin J
In applying s. 515(10) a judge has a wide discretionary power because of the importance of the
circumstances of each case in the decision-making process
 However, this discretionary power must be exercised judicially, taking into account
the fundamental principles of criminal law, and particularly the rights guaranteed by the
Charter
The right to be presumed innocent should not be considered only to decide the question of guilt
or innocence
 The same right applies to s. 520, 521 or 522, and does not vary according to the nature
of the crime charged
Even if s. 522 shifts the burden of proof, the accused is still presumed innocent
 Perron authoritatively defined the notion of public interest in that context
o That criterion implies two aspects: the perception of the public if the
accused is released from custody, and its reaction
134
The perception of the public is often negative, sometimes emotionally so, towards criminals
 However, the criminal law has an educative goal to achieve
 The judge must take into account the perception of the INFORMED PUBLIC who
understand that the right to be presumed innocent at every stage of the criminal
process is not a theoretical notion, but is the price to pay for living in a free and
democratic society
 The judge should also take into account the reaction the informed public would have if
the accused were released from custody before trial
 Would that release discredit the administration of justice?
Lectures notes
 The court recognizes that it would not want to be entertaining public views when they
are not aware of the purpose of the Bail Reform Act and that bail should not be use as a
means of denunciation
 The court lists some criteria which could be relevant to interpreting what the reasonable
response of the public may be
o As for recidivism, what is at stake here is the public perception of recidivism and
not public safety
s. 11(e) – Canadian Charter of Rights and Freedoms
Proceedings in criminal and penal matters
Any person charged with an offence has the right
...
e) not to be denied reasonable bail without just cause
R. v. Morales, [1992] 3 SCR 711
Issues:
1. Did the criterion of “public interest” as a basis for pre-trial detention violate s. 11(e) of the
Charter?
2. Is the “public safety” component unconstitutional?
Holding: 1.Yes 2. No; appeal allowed
Lamer CJC
The criterion of "public interest" violates s. 11(e) of the Charter because it authorizes
detention in terms which are vague and imprecise
As per R. v. Nova Scotia Pharmaceutical Society, the doctrine of vagueness is a consideration
both as a principle of fundamental justice under s. 7 and in applying s. 1
 It is a principle of fundamental justice that laws may not be too vague
 Vagueness can also be raised under s. 1 of the Charter in limine, on the basis that an
enactment is so vague as not to satisfy the requirement that a limitation on Charter
rights be "prescribed by law"
 A law will be found unconstitutionally vague if it so lacks in precision as not to give
sufficient guidance for legal debate
135
There cannot be just cause for denial of bail within the meaning of s. 11(e) if the statutory
criteria for denying bail are vague and imprecise
Nova Scotia Pharmaceutical Society identified two rationales for the doctrine of vagueness,
namely fair notice to the citizen and limitation of law enforcement discretion
 While fair notice is not relevant to a provision like s. 515(10)(b) which does not
prohibit conduct, limitation of law enforcement discretion is still a relevant factor
o Is the statute so pervasively vague that it permits a ‘standardless sweep’ allowing
law enforcement officials to pursue their personal predilections?
 Since pre-trial detention is extraordinary in our system of criminal justice,
vagueness in defining the terms of pre-trial detention may be even more invidious
than is vagueness in defining an offence
A provision does not violate the doctrine of vagueness simply because it is subject to
interpretation
 Nevertheless, the term "public interest" is incapable of framing the legal debate in
any meaningful manner or structuring discretion in any way
 Nor would it be possible to give the term "public interest" a constant or settled meaning
This violation is not justified under s. 1
 Even if the term "public interest" is capable of passing the threshold test under s. 1 of
being a limit which is "prescribed by law", it cannot be justified under the Oakes test
o It can be justified under the first branch of the Oakes test given its objectives: 1)
prevent those who have been arrested from committing criminal offences;
2) prevent those who have been arrested from interfering with the administration
of justice
o However, it does not meet the proportionality branch of the Oakes test
 First, there is no rational connection between the measure and the
objectives of preventing crime and preventing interference with the
administration of justice
 Second, the measure does not impair rights as little as possible
 Third, there is no proportionality between the effects of the measure and
its objectives
Gonthier J (dissenting in part)
It is first significant to recognize the general sense of the phrase “public interest”, which is a
reference to the special set of values which are best understood from the point of view of
the aggregate good and are of relevance to matters relating to the well-being of society
 The Charter is an expression of the fundamental values which direct the public interest
 The concept of public interest is indeed broad but it is not meaningless, nor is it
vague
At issue are the reasons for detention, for which the Charter requires a just cause
 This implies two elements: (1) a cause or reason and (2) proportionality between the
reason and the deprivation of liberty that makes the cause "just"
136
Under s. 515(10), two main elements exist in relation to the operation of the public interest
criterion
 One of them is a requirement of necessity
 The other is the content of the considerations which may be included within the public
interest criterion
o The concept of public interest is broader than that of protection or safety of the
public, and includes interests which may not be properly included within the
categories of public health or safety
o The aim of avoiding interference with the administration of justice is one such
example
o Other examples of a public interest could be the protection of the accused from
suicide or from the actions of others, the prevention of activities which involve
the possession of or dealing in small quantities of illegal narcotics, or the
preparation of reports for the court which require the presence of the accused
o Public interest is capable of encompassing circumstances which have not been
foreseen
One must be wary of using the doctrine of vagueness to prevent or impede State action in
furtherance of valid social objectives, by requiring the law to achieve a degree of precision to
which the subject-matter does not lend itself
 A delicate balance must be maintained between societal interests and individual rights
As stated by Lamer CJC flexibility and vagueness are not synonymous
 The former is a quality necessary to the administration of justice and s.515(10)(b)
provides for this
 Its dual requirements of public interest and necessity which itself predicates a
public interest of a serious nature have meaning, give rise to legal debate and,
though broad, are not vague but provide an adequate framework and limit for the
exercise of judicial discretion
Lectures notes
 This is the first time that the court carefully considered what the purpose of the former
grounds for pre-trial detention was, in particular the secondary ground in the old
legislation
 The purpose of the secondary ground according to Lamer J was to prevent those being
arrested from interfering with the administration of justice
 One thing in which the majority and minority seem to agree is that deterrence and
denunciation does not play a role in determining bail
Five years after Morales Parliament amended the Criminal Code, including an amendment
to s. 515
 S. 515(10) now consists of three circumstances which justify the pre-trial detention of the
accused
o Flight risk
o Public safety
o Maintaining public confidence in the administration of justice
137

This third criteria, however, includes the words “on any other just cause”
which was in fact deemed unconstitutional (see Hall)
R. v. Hall (2002), 4 CR (6th) 197
Facts
 In 1999, a woman's body was found with 37 wounds to her hands, forearms, shoulder,
neck and face. It also appeared that her assailant had tried to cut off her head
 The murder caused significant public concern and a general fear that a killer was at large
 Based on compelling evidence the accused was charged with first degree murder and
applied for bail
 The bail judge held that pre-trial detention was not necessary "to ensure ... attendance in
court" nor for the "safety of the public"
o He denied bail, however, under s. 515(10)(c) in order "to maintain confidence in
the administration of justice"
 A superior court judge dismissed the accused's habeas corpus application challenging
the constitutionality of s. 515(10)(c)
 The CA affirmed the decision
Issue: Does s. 515(10)(c) violate s. 11(e) of the Charter?
Holding: No; appeal dismissed
McLachlin CJC
The portion of s. 515(10)(c) permitting detention "on any other just cause being shown" is
unconstitutional
 The impugned phrase confers an open-ended judicial discretion to refuse bail and is
inconsistent with both s. 11(e) of the Charter and the presumption of innocence
 It is a fundamental principle of justice that an individual cannot be detained by virtue
of a vague legal provision
o Parliament must lay out narrow and precise circumstances in which bail can be
denied
 The impugned phrase is not justified under s. 1 of the Charter as its generality impels its
failure of the proportionality branch of the Oakes test
The next phrase in s. 515(10)(c) "without limiting the generality of the foregoing" is also void
since it only confirms the generality of the preceding phrase
The balance of s. 515(10)(c), which authorizes the denial of bail in order "to maintain
confidence in the administration of justice", is valid
 It provides a basis for denying bail not covered by s. 515(10)(a) and (b)
 Although the circumstances in which recourse to this ground may not arise frequently,
when they do it is essential that a means of denying bail be available because public
confidence is essential to the proper functioning of the bail system and the justice
system as a whole
Denial of bail "to maintain confidence in the administration of justice" having regard to the
factors set out in s. 515(10)(c) complies with s. 11(e) of the Charter
138




This ground is narrower and more precise than the old public interest ground
which was struck down as vague in 1992 and provides an intelligible standard for
debate and for the exercise of discretion
The means chosen do not go further than necessary to achieve Parliament's purpose
of maintaining public confidence in the bail system and the justice system as whole
Parliament has hedged the provision with important safeguards: a judge can only
deny bail if satisfied that in view of the four specified factors and related circumstances,
a REASONABLE MEMBER of the community would be satisfied that denial of bail is
necessary to maintain confidence in the administration of justice
The provision is not overbroad but strikes an appropriate balance between the rights of
the accused and the need to maintain justice in the community.
The appropriate remedy in this case is to sever the phrase "on any other just cause being
shown, and without limiting the generality of the foregoing"
 The balance of s. 515(10)(c) can stand alone as a functioning whole without doing
damage to Parliament's intention
Iacobucci J (dissenting)
At the heart of a free and democratic society is the liberty of its subjects and our justice
system must minimize unwarranted denials of liberty
 In the criminal law context, this freedom is embodied generally in the right to be
presumed innocent until proven guilty and specifically in the right to bail
Fear that a bail judge will be unable to protect the public without s. 515(10)(c) is without
reasonable foundation
 There is no evidence that the bail system was lacking in any way before the introduction
of the provision in 1997
Section 515(10)(c) must be assessed as a whole
 Yet even if these components are considered independently, neither can withstand
constitutional scrutiny
It is impossible to hold that the phrase "any other just cause" provides for the denial of bail
under a narrow set of circumstances
 The phrase is deliberately open-ended and is vaguer than the old "public interest"
ground
Nor does the "confidence in the administration of justice" component of s. 515(10)(c)
provide a sufficiently precise standard
 The specific factors listed in s. 515(10)(c) provide little more than a facade of precision
 It is impermissibly vague because of the failure to establish a plausible and valid ground
for denying bail that would serve the proper administration of the bail system and that is
not already covered under the more specific grounds in s. 515(10)(a) and (b)
 Section 515(10)(c) essentially revives the old "public interest" ground and invokes
similarly vague notions of the public image of the criminal justice system
139
The bail judge erred in considering the subjective fears of the public after determining there was
no risk of flight nor any threat to the public
 The problem with s. 515(10)(c) is that it allows the subjective fears of the public and illinformed emotional impulses extraneous to the bail system to form a sole basis for
denying bail
Section 515(10)(c) cannot be saved under s. 1 of the Charter
 It does not address a pressing issue and fails the proportionality tests
The only available remedy is to strike down s. 515(10)(c) in its entirety
 Although the section may encompass instances where bail could be denied without
offending s. 11(e), the Court cannot sufficiently narrow the scope of that section without
seriously intruding into the legislative sphere
 Even if the administration of justice portion of the provision were constitutional, s.
515(10)(c) could not be read down to include only this portion
o The second component of s. 515(10)(c) is but an example of denying bail "on
any other just cause"
o To convert a single example of an explicitly open-ended rule into the rule itself
substantially changes the significance of the provision and contravenes
Parliament's clear intention not to limit the generality of "any other just cause"
Lecture notes
 A criticism of the “confidence in administration of justice” criteria is that it adds many
elements to the bail determination which go beyond the basic issues of public safety and
flight risk
o Proving these may require witnesses and a vast amount of evidence
There seems to be a trend in the jurisprudence to use the third ground to deny bail (s. 515(10)(c))
sparingly
 For some examples of where this ground has been invoked see p.631 of textbook
 Most judgment have place emphasis on McLachlin’s acceptance that the reasonable
person, in making an assessment of the need to maintain public confidence, must be
properly informed about the philosophy of the legislative provision, Charter values
and the actual circumstance of the case
3. The Judge Must Decide
Under the previous legislation, judges were to inquire into the sufficiency of sureties and make a
decision based on the merits of the case
 Empirical studies showed that a decision-making process was not followed in most cases,
but rather an inflexible procedure of cash or property in advanced based on the nature of
the crime
 While the new provisions provide for judicial review on these matters, and the legislature
has tried to be as specific as possible, the success of the system depends on whether or
not it has support at the grass-roots levels
140
Do judges under the new system make real decisions in determining bail on their own, or
like before, they simply abide by the wishes of the police and prosecutors?
R. v. Major (1990), 76 CR (3d) 104 (Ont. Dist. Ct.)
Facts
 The judge consented to a detention order
 Judge Charron found this to be in error
Charron DJC
When an accused is brought before a justice of the peace under s. 524, it is quite clear that
the justice, of the peace has a duty to inquire into the alleged misconduct, make a finding in
this respect and then make an appropriate order as set out in the section
 The facts relied upon by the Crown may well be presented by way of admission
where the accused so consents but evidence must be heard in order to support a
finding by the presiding justice
 An order must be based on such finding and cannot simply be based on a bare
consent without more
It is clear that the justice of the peace has a judicial function to perform in the determination
of appropriate conditions and that function cannot be delegated to the prosecutor
The justice of the peace must exercise his or her discretion as set out in the Code
4. Reasonable Conditions on Release
A judge must not impose so stringent condition on release, so that it amounts, in fact, to
detention
 The conditions imposed must be reasonable
 See Re Keenan and R., 1979
o The conditions imposed must relate to the criteria for detention in s. 515(10
R. v. Anoussis (2008) Criminal Chamber - Court of Quebec
 Accused was charged with causing bodily harm
 The Crown was asking for conditions made on the judicial interim release
o The first condition asked for was a cash deposit
o Healy J felt that this was not required and encouraged defence counsels to raise
objections to these conditions more often
 Cash deposits are meant to be exceptional
 The word bail was abandoned and replace with judicial interim release
because of its pecuniary connotations
 One view, however, is that this provides the easiest way for the accused to
be let out on interim release
 What if the accused agrees with the cash deposit?
141



Should this override the ladder approach to determining conditions
for judicial interim release?
Healy J also talks about the unequal treatment that may result from making cash deposits
the basis of judicial interim release
While judges must intervene, there is a reason to respect the choices made by some
particular defendants
5. Charter Standards
While one of the cornerstones of the Bail Reform Act of 1976 was to place the onus on the
Crown to justify a detention, the legislature created a number of situations where the onus would
be placed on the accused to justify release
Pearson deals with one of these situations, where the accused is charged with trafficking drugs
under the Controlled Drugs and Substances Act (then the Narcotic Control Act)
R. v. Pearson, [1992] 3 SCR 665
Issue: Does s. 515(6)(d) violate ss. 7, 9, 11(d) or 11(e) of the Charter?
Holding: No; appeal allowed
Lamer CJC
There is just cause for the denial of bail under s. 515(6)(d)
Bail is denied only in a narrow set of circumstances
 S. 515(6)(d) applies only to a very small number of offences, all of which involve the
distribution of narcotics
 The section does not deny bail for all persons who are charged with these offences, but
rather denies bail only when these persons are unable to demonstrate that detention is
not justified
The denial of bail is necessary to promote the proper functioning of the bail system and is
not undertaken for any purpose extraneous to the bail system
 S. 515(6)(d) establishes a set of special bail rules in circumstances where the normal bail
process is incapable of functioning properly
 The unique characteristics of the offences subject to s. 515(6)(d) suggest that those
offences are committed in a very different context than most other crimes
o Most offences, unlike the trafficking of narcotics, are not committed systematically
o There is also a marked danger that an accused charged with these offences will not
appear for trial
In the Court of Appeal, Proulx J.A. expressed concern about the scope of s. 515(6)(d) and he felt
that it is inequitable to treat a person who distributes a few joints of marijuana in the same manner
as a person running a sophisticated network to traffic cocaine
 While these concerns are legitimate, these lesser offenders will normally have no difficulty
142
justifying their release and obtaining bail
McLachlin J (dissenting in part)
S. 515(6)(d) section is very broad
 It applies to everyone who commits the specified offences
 In addition, the offences are very broad
o As a result, s. 515(6)(d) catches not only large-scale drug dealers, but the friend
who shares a joint of marijuana at a party or gives it to a friend for safekeeping
o All trafficking is repugnant and hence criminally punishable, but when the issue
is denial of bail, the different dangers associated with the different types of
trafficking, may require different treatment
While there may be "just cause" for denying bail to persons charged with serious, large-scale or
commercial trafficking, the reasons offered by Lamer CJC do not apply to other traffickers
 His reasons for a just cause apply only to the organized commercial trafficker
o The "small-time" trafficker may not be motivated by money or profit, nor be a
participant in a "highly sophisticated commercial setting"
o Neither is she very likely to flee justice
S. 515(6)(d) is not "demonstrably justified" in a free and democratic society
 While avoiding repeat offences and escaping justice are important objectives, the section
goes further than is necessary to achieve those objectives
Other jurisdictions, like the United States and Australia, distinguish between major and minor
trafficking
Lecture notes
 The court seems to agree that the presumption of innocence in 11(d) of the Charter
requires a burden on the Crown to justify pre-trial detention
The Report of the Commission on Systemic Racism in the Ontario Criminal Justice System
(1995) suggest that the practical effect of presuming detention for persons charged with
trafficking narcotics results in racism and the disproportionate detention of small-scale
offenders
R. v. Cantave, Court o f Quebec Criminal Chamber – September 9 2008
 Man in possession of two loaded firearms and had been ordered to not be in possession of
firearms before
 Healy J finds that pre-trial detention was justified under the second and third ground
given the gravity of these types of offences
Disclosure and Discovery
143
Except for narrow exceptions, it is generally not obliged for the accused to disclose her case
to the Crown
 The main focus is on the obligations for the Crown, justified as a matter of fairness
There is a distinction between disclosure and production
 Disclosure cases involved whether the Crown properly shared the evidence in its
possession
 Production cases involve potentially relevant materials not in the possession of the
Crown, but in the hands of so-called third parties
The charging instrument given the accused an outline of the case he will have to meet at trial
An amendment to the Code in 1985 now provides for pre-hearing conferences
s. 625.1 – Criminal Code
Pre-hearing conference
(1) Subject to subsection (2), on application by the prosecutor or the accused or on its own
motion, the court, or a judge of the court, before which, or the judge, provincial court judge or
justice before whom, any proceedings are to be held may order that a conference between the
prosecutor and the accused or counsel for the accused, to be presided over by the court, judge,
provincial court judge or justice, be held prior to the proceedings to consider the matters that, to
promote a fair and expeditious hearing, would be better decided before the start of the
proceedings, and other similar matters, and to make arrangements for decisions on those
matters.
Mandatory pre-trial hearing for jury trials
(2) In any case to be tried with a jury, a judge of the court before which the accused is to be
tried shall, before the trial, order that a conference between the prosecutor and the accused or
counsel for the accused, to be presided over by a judge of that court, be held in accordance with
the rules of court made under sections 482 and 482.1 to consider any matters that would
promote a fair and expeditious trial. .
Disclosure
Royal Commission on the Donald Marshall Jr. Prosecution
While the courtroom is adversarial, the Crown prosecutor must make sure the criminal justice
system is itself fair
Disclosure
 Anything less than complete disclosure by the Crown falls short of decency and fair
play
 This is linked to the fundamental justice of the accused to make full answer and defence
144
 The principle of fair trial requires that the accused receive all information know to
the Crown that might reasonably be considered useful to the accused
 The Crown should have a positive continuing duty to make disclosure to the defence
 It is immaterial whether the defence fails to request disclosure
 It is not realistic to assume that prosecutor does not have an interest in the outcome of
the trial
 An obligation to disclose all relevant information is not inconsistent with the adversarial
system
 Whether or not to disclose information should not be a decision left to the discretion
of the prosecution
o A statutory obligation to disclose is necessary
The law exists to protect society against lawbreakers and to safeguard the liberty and freedom
of individuals against the power of the State
R. v. Stinchcombe, [1991] 3 SCR 326
Facts
 The accused, a lawyer, was charged with breach of trust, theft and fraud
 A former secretary of his was a Crown witness at the preliminary inquiry, where she
gave evidence apparently favourable to the defence
 After the preliminary inquiry but prior to trial, the witness was interviewed by an RCMP
officer and a tape‑recorded statement was taken. Later, during the course of the trial,
the witness was again interviewed by a police officer and a written statement taken
 Defence counsel was informed of the existence but not of the content of the statements
 His requests for disclosure were refused
 During the trial defence counsel learned conclusively that the witness would not be
called by the Crown and sought an order that the witness be called or that the Crown
disclose the contents of the statements to the defence
 The trial judge dismissed the application and the accused was eventually convicted of
breach of trust and fraud
 The Court of Appeal affirmed the convictions without giving reasons
Issue: Is there a constitutional right to disclosure by the Crown as per s. 7 of the Charter?
Holding: Yes; appeal allowed and a new trial ordered
Sopinka J
The Crown has a legal duty to disclose all relevant information to the defence
The fruits of the investigation which are in its possession are not the property of the
Crown for use in securing a conviction but the property of the public to be used to ensure
that justice is done
The obligation to disclose is subject to a discretion with respect to the withholding of
information and to the timing and manner of disclosure
 Crown counsel has a duty to respect the rules of privilege and to protect the identity of
informers
145
 Discretion must also be exercised with respect to the relevance of information
 The Crown's discretion is reviewable by the trial judge, who should be guided by the
general principle that information should not be withheld if there is a reasonable
possibility that this will impair the right of the accused to make full answer and
defence
The absolute withholding of information which is relevant to the defence can only be justified
on the basis of the existence of a legal privilege which excludes the information from disclosure
 This privilege is reviewable on the ground that it is not a reasonable limit on the right to
make full answer and defence in a particular case
Counsel for the accused must bring to the trial judge's attention at the earliest opportunity any
failure of the Crown to comply with its duty to disclose of which counsel becomes aware
 This will enable the trial judge to remedy any prejudice to the accused if possible and
thus avoid a new trial
Initial disclosure should occur before the accused is called upon to elect the mode of trial
or plead
 Subject to the Crown's discretion, all relevant information must be disclosed, both that
which the Crown intends to introduce into evidence and that which it does not, and
whether the evidence is inculpatory or exculpatory
 All statements obtained from persons who have provided relevant information to the
authorities should be produced, even if they are not proposed as Crown witnesses

Where statements are not in existence, other information such as notes should be
produced
o If there are no notes, all information in the prosecution's possession relating to
any relevant evidence the person could give should be supplied
Crown counsel was not justified in refusing disclosure here on the ground that the witness was
not worthy of credit: whether the witness is credible is for the trial judge to determine after
hearing the evidence
 The trial judge ought to have examined the statements
 Since the information withheld might have affected the outcome of the trial, a new trial
was ordered
A 2002 amendment to the Code requires defence counsel to give notice of an expert it intends to
call, plus once the Crown has closed its case, a report on which the opinion will be based
The Stinchcombe duty on the Crown is not absolute
 It is only triggered on request and subject to Crown discretion not to disclose irrelevant
material or that which is subject to privilege
Production of Third Party Records
These new principles of disclosure have been highly controversial in cases of sexual assault or
when medical records are involved
146
In L.L.A. v. A.B. (1995), which was handed down with O’Connor, the SCC unanimously decided
that production should not be determined by class or case-by-case privilege (resembling the
Wigmore criteria)\
 The better approach was one of balancing competing Charter rights
R. v. O’Connor, [1995] 4 SCR 411
Facts
 A bishop was charged with a number of sexual offences
 The charges were stayed when it became apparent that the Crown had not fully
complied with a sweeping pre-trial order to disclose which included the information
regarding the identity of psychiatrists or counsellors who had treated the complainants
with respect to the allegations of sexual abuse, as well as the records of these therapy
sessions
Issue: Was there an obligation to disclose third party records from the complainants therapy?
Holding: Yes; appeal allowed and a new trial ordered
L'Heureux‑Dubé , La Forest, Gonthier and McLachlin saw the need to balance the accused’s
right to a fair trial and full answer and defence with the complainant’s right to privacy
and to equality without discrimination
 The majority determined that the accused’s right should be balanced with the
complainants’ right to privacy under ss. T and 8 of the Charter
The Court agreed that there should be a two-stage procedure but was divided 5-4 as to the
precise test
 The majority decided that when the defence seeks information in the hands of a
third party the onus should be on the accused to satisfy the judge that the
information is likely to be relevant
1. In order to initiate the procedure the accused must bring an application setting out the
grounds for production (this formal application can be waived by the court)
 Notice must be given to the third party in possession of these documents
 The meaning of relevance, in the context of disclosure, is whether the information might
be useful to the defence
o In the context of production the test for relevance should be higher, the trial
judge must be satisfied that there is a REASONABLE possibility that the
information is logically probable to an issue at trial or the competence of a
witness
o The majority disagreed with L'Heureux‑Dubé, however, that such records will
only be relevant in rare cases
o The majority gave an example of the therapy being relevant as to how the
complaint unfolded, the use of therapy to influence memory and information
bearing on credibility
2. Upon production of the documents, the judge should examine the records and
determine whether, and to what extent, they should be produced to the accused
147


Here, the judge must weigh the valuable and deleterious effects of a production order
and consider whether non-production would constitute a reasonable limit on the ability
of the accused to make full answer and defence
For the minority, at the second stage of deciding whether the records should be
produced to the accused, the court needed to decide in a manner that was respectful to
Charter values of the right to privacy and equality
The majority was also on the view that the Crown’s disclosure obligation in Stinchcombe was
unchanged by the nature of the records where the records where in possession of the
Crown
LOST EVIDENCE CASES
R.v. Carosella (1997)
 Since the Stinchcombe and O’Connor was met, a stay should be granted as remedy for
the violation of the accused’s ’right to make a full answer and defence as per s. 7 of the
Charter
R. v. La; R. v. Vu
 Seems like the SCC took a different approach when evidence was lost rather than
destroyed
 When the evidence is lost, the Crown has a duty to explain the loss
o When the court is satisfied that the evidence was not lost due to unacceptable
negligence, the duty to disclose has not been breached
 As the relevance of evidence increases, so does the degree of care for its preservation
 A stay may be appropriate under exceptional circumstances where the loss of evidence is
so prejudicial to the right to make full answer and defence
BILL C-46
Following O’Connor, Parliament passed legislation to restrict the production of records in sexual
offence proceedings
 This is contained in ss. 278.1-278.9
 The legislation largely reflects the minority position in O’Connor
 This reform was held to be constitutional in Miills
R. v. Mills, [1999] 3 SCR 668
Issue: Is the restriction to disclose the production of records in sexual offence proceedings
constitutional?
Holding: Yes
McLachlin and Iacobucci JJ
To challenge the constitutionality of the impugned legislation, the accused need not prove that
the legislation would probably violate his right to make full answer and defence
148

It is sufficient that he establish that the legislation is unconstitutional in its general
effect
Although the procedure governing the production of private records of complainants in sexual
assault proceedings set out in Bill C-46 differs significantly from the O’Connor regime, it does
not follow that Bill C-46 is unconstitutional
 A posture of respect towards Parliament has been adopted by the courts and the
relationship between the courts and the legislature should be one of dialogue
 The courts do not hold a monopoly on the protection and promotion of rights and
freedoms
Two principles of fundamental justice seem to conflict: the right to full answer and
defence and the right to privacy
 Neither right may be defined in such a way as to negate the other and both sets of rights
are informed by the equality rights at play in this context
 A contextual approach to the interpretation of rights should be adopted as they often
inform, and are informed by, other rights at issue in the circumstances
The right of an accused to make full answer and defence is crucial to ensuring that the
innocent are not convicted
 The accused’s right to make full answer and defence must be understood in light of
other principles of fundamental justice which may embrace interests and perspectives
beyond those of the accused
 In this context, full answer and defence does not include the right to evidence that would
distort the search for truth inherent in the trial process.
An order for the production of records made pursuant to ss. 278.1 to 278.91 of the Criminal
Code is a seizure within the meaning of s. 8 of the Charter
 The therapeutic relationship is one that is characterized by trust, an element of which is
confidentiality
 The protection of the complainant’s reasonable expectation of privacy in her therapeutic
records protects the therapeutic relationship and the mental integrity of complainants
and witnesses
 The right to privacy is one which may be limited as reasonable searches and seizures are
permitted by s. 8 of the Charter
 The accused will have no right to the records in question insofar as they contain
information that is either irrelevant or would serve to distort the search for truth
o On the other hand, the accused’s right must prevail where the lack of disclosure
or production of the record would render him unable to make full answer and
defence
o Full answer and defence will be more centrally implicated where the information
contained in a record is part of the case to meet or where its potential probative
value is high
o Privacy rights will be most directly at stake where a record concerns aspects of
one’s individual identity or where confidentiality is crucial to a therapeutic or
trust-like relationship
149
Equality concerns must also inform the contextual circumstances in which the rights of
full answer and defence and privacy will come into play
 An appreciation of myths and stereotypes in the context of sexual violence is essential to
delineate properly the boundaries of full answer and defence
The definition of the records subject to Bill C-46 is not overly broad as the legislation only
applies to records in which there is a reasonable expectation of privacy
This provision does not by itself deny access to documents to which the defence is
constitutionally entitled
Crown possession or control of such records cannot be equated with a total loss of any
reasonable expectation of privacy
 It is constitutionally permissible for the Crown to be subject to different treatment, to
different procedures, or even to end up with documents that the accused has not seen, as
long as the accused can make full answer and defence and the trial is fundamentally
Section 278.3(4) of the Criminal Code, which lists a series of assertions that cannot on their
own establish that a record is likely relevant, does not violate s. 7 of the Charter
 The purpose of this provision is to prevent speculative and unmeritorious requests for
production
In s. 278.5(1) of the Criminal Code Parliament supplemented the “likely relevant” standard for
production to the judge proposed in O’Connor with the further requirement that production be
“necessary in the interests of justice”
 Under the new provision a trial judge is required to consider the salutary and
deleterious effects of production to the court on the accused’s right to make full
answer and defence and on the rights to privacy and equality of the complainant
or witness
 The criterion that production to the court be necessary in the interests of justice invests
the trial judge with the discretion to consider the full range of rights and interests at
issue before ordering production, in a manner scrupulously respectful of the
requirements of the Charter
In determining whether production is necessary in the interests of justice, the judge need not
engage in a conclusive and in-depth analysis of each of the factors listed in s. 278.5(2)
 The provision merely requires that the judge take these factors into account
 Trial judges are only asked to take into account the factors listed in s. 278.5(2) and are
not required to rule conclusively on each factor
 Trial judges are also not required to determine whether factors relating to the privacy
and equality of the complainant or witness outweigh factors relating to the accused’s
right to full answer and defence
 The inclusion of societal interest factors in the analysis does not alter the constitutional
balance established in O’Connor
150
Lamer CJ (dissenting in part)
While agree with the majority’s finding that Bill C-46 complies with ss. 7 and 11(d) of the
Charter as it applies to the production of records in the possession of third parties, Lamer CJ
took a different view of the legislative regime’s approach to records in the hands of the Crown
 Bill C-46's treatment of records that form part of the case to meet tips the balance too
heavily in favour of privacy to the detriment of the accused’s right to make full answer
and defence
On the issue of equality, why was there no reference to the ten-part test for judging s. 15 set out
in Law v. Minister of Human Resoirces Development (1999)?
Preliminary Inquiry
Prior to the trial of certain indictable offences, a judge will inquire into the charge and determine
whether there is sufficient evidence to warrant placing the accused on trial
The procedure for preliminary inquiries is set out in Part XVIII of the Code
If the indictable offence is listed in s. 553 of the Code, the jurisdiction of the provincial court is
absolute and does not depend on the choice of the accused, so there will be no preliminary
inquiry
If the offence is listed under s. 469 of the Code, the offence must be tried at the Superior Court,
and a preliminary inquiry will likely be held
With respect to all other indictable offences, if the accused elects to be tried in a provincial court,
she will not have a preliminary inquiry
s. 535 – Criminal Code
Inquiry by justice
If an accused who is charged with an indictable offence is before a justice and a request has
been made for a preliminary inquiry under subsection 536(4) or 536.1(3), the justice shall, in
accordance with this Part, inquire into the charge and any other indictable offence, in respect of
the same transaction, founded on the facts that are disclosed by the evidence taken in
accordance with this Part.
Preferring the Indictment
The Criminal Code allows the Crown to jump over the preliminary inquiry and go straight to a
trial; this is known as a "preferred indictment".
151
The Code provides that following the preliminary inquiry, the Crown may prefer an indictment
against the person ordered to stand trial
 See ss. 566 and 574
 As per s. 574(1) the preferred indictment may be on
o Any charge on which that person was ordered to stand trial; or
o Any charge founded on the facts disclosed by the evidence taken on the
preliminary inquiry, in addition to or in substitution for any charge on which that
person was ordered to stand trial.
 The power of the Crown to prefer an indictment under s. 574 if broader that the power of
the Justice presiding at a preliminary inquiry
s. 566 – Criminal Code
Indictment
(1) The trial of an accused for an indictable offence, other than a trial before a provincial court
judge, shall be on an indictment in writing setting forth the offence with which he is charged.
Preferring indictment
(2) Where an accused elects under section 536 or re-elects under section 561 to be tried by a
judge without a jury, an indictment in Form 4 may be preferred.
What counts may be included and who may prefer indictment
(3) Section 574 and subsection 576(1) apply, with such modifications as the circumstances
require, to the preferring of an indictment pursuant to subsection (2).
s. 574 – Criminal Code
Prosecutor may prefer indictment
(1) Subject to subsection (3), the prosecutor may, whether the charges were included in one
information or not, prefer an indictment against any person who has been ordered to stand trial
in respect of
(a) any charge on which that person was ordered to stand trial; or
(b) any charge founded on the facts disclosed by the evidence taken on the preliminary
inquiry, in addition to or in substitution for any charge on which that person was ordered
to stand trial.
Preferring indictment when no preliminary inquiry requested
(1.1) If a person has not requested a preliminary inquiry under subsection 536(4) or 536.1(3)
into the charge, the prosecutor may, subject to subsection (3), prefer an indictment against a
person in respect of a charge set out in an information or informations, or any included charge,
at any time after the person has made an election, re-election or deemed election on the
information or informations.
152
Preferring single indictment
(1.2) If indictments may be preferred under both subsections (1) and (1.1), the prosecutor may
prefer a single indictment in respect of one or more charges referred to in subsection (1)
combined with one or more charges or included charges referred to in subsection (1.1).
Consent to inclusion of other charges
(2) An indictment preferred under any of subsections (1) to (1.2) may, if the accused consents,
include a charge that is not referred to in those subsections, and the offence charged may be
dealt with, tried and determined and punished in all respects as if it were an offence in respect
of which the accused had been ordered to stand trial. However, if the offence was committed
wholly in a province other than that in which the accused is before the court, subsection 478(3)
applies.
Private prosecutor requires consent
(3) In a prosecution conducted by a prosecutor other than the Attorney General and in which the
Attorney General does not intervene, an indictment may not be preferred under any of
subsections (1) to (1.2) before a court without the written order of a judge of that court.
In addition, the Crown may prefer an indictment where a preliminary inquiry has not been held
or where such has been held and the accused has been discharged
 This is sometimes referred to as direct indictment
s. 577 – Criminal Code
Direct indictments
Despite section 574, an indictment may be preferred even if the accused has not been given the
opportunity to request a preliminary inquiry, a preliminary inquiry has been commenced but not
concluded or a preliminary inquiry has been held and the accused has been discharged, if
(a) in the case of a prosecution conducted by the Attorney General or one in which the
Attorney General intervenes, the personal consent in writing of the Attorney General or
Deputy Attorney General is filed in court; or
(b) in any other case, a judge of the court so orders.
There is no absolute right for the accused to be given a hearing or to submit representations
before the preferement of a direct indictment and this does not violate s. 7 of the Charter
 However, the constitutionality of this was based on the fact that the accused is given
adequate disclosure prior to trial
Preliminary inquiry as discovery vehicle
Skogman v. R, [1984] 2 SCR 93
153
Estey J
The purpose of a preliminary hearing is to protect the accused from a needless and improper,
exposure to public trial where the Crown is not in possession of evidence to warrant the
continuation of the process
In addition, in the course of the preliminary hearing has become a forum where the
accused is afforded an opportunity to discover and to appreciate the case to be made
against her at trial where the requisite evidence is found to be
The inquiry has become mainly a judicial examination of the justification and need for pretrial detention of the accused as well as an examination of the need for a trial itself
Its primary purpose is to ascertain whether or not there is sufficient evidence to warrant the
accused being placed upon his trial
 This is not a determination of whether or not the accused is guilty or not guilty
 The judge’s function is to ascertain whether or not there is sufficient evidence to
induce the belief in the mind of a cautious person that the accused is probably
guilty
o Therefore, considerations of reasonable doubt have no application at this stage
of the proceedings.
From the point of view of defence counsel the preliminary hearing has another aspect
 It affords counsel an opportunity of ascertaining the nature and the strength of the
case against her client and it may be likened in that respect to an Examination for
Discovery
Test for committal: sufficiency of evidence
s. 548 – Criminal Code
Order to stand trial or discharge
(1) When all the evidence has been taken by the justice, he shall
(a) if in his opinion there is sufficient evidence to put the accused on trial for the
offence charged or any other indictable offence in respect of the same transaction, order
the accused to stand trial; or
(b) discharge the accused, if in his opinion on the whole of the evidence no sufficient
case is made out to put the accused on trial for the offence charged or any other
indictable offence in respect of the same transaction.
Endorsing charge
(2) Where the justice orders the accused to stand trial for an indictable offence, other than or in
addition to the one with which the accused was charged, the justice shall endorse on the
154
information the charges on which he orders the accused to stand trial.
Where accused ordered to stand trial
(2.1) A justice who orders that an accused is to stand trial has the power to fix the date for the
trial or the date on which the accused must appear in the trial court to have that date fixed.
Defect not to affect validity
(3) The validity of an order to stand trial is not affected by any defect apparent on the face of
the information in respect of which the preliminary inquiry is held or in respect of any charge
on which the accused is ordered to stand trial unless, in the opinion of the court before which an
objection to the information or charge is taken, the accused has been misled or prejudiced in his
defence by reason of that defect.
For many years, the test for committal at preliminary inquiry was held to be the same as that
declared by the SCC in United States v. Sheppard (1976)
 The judge was not to weigh the evidence by assessing credibility, but assess whether a
properly instructed jury would convict if it believed the evidence
R. v. Nelles (1982), 16 CCC (3d) 97 (Ont. Prov. Ct)
Vanek J
In arriving at a decision whether or not to commit the accused for trial, it is not the
function of a "justice" at a preliminary inquiry to WEIGH the evidence for the purpose of
determining whether it is credible or trustworthy
 These matters fall within the sphere of the jury
 The justice must assess whether it is "sufficient to warrant committing the accused
for trial
A justice at a preliminary inquiry is bound to weigh the evidence in the sense of analyzing
it and considering the inferences that may reasonably be drawn there from it for the
purpose of judging the sufficiency of the evidence for committal
 In this sense, perhaps one might substitute for “weighing” the word “assessing”
Comment
 Notice how the court here introduced the term ‘weighing’ the evidence in relation to an
assessment for committal;
R. v. Arcuri, [2001] 2 SCR 828
Facts
 The accused was charged with first degree murder
 At the preliminary inquiry, the Crown’s case was entirely circumstantial and the accused
called two witnesses whose testimony was arguably exculpatory
 The preliminary inquiry judge rejected the accused’s contention that he must weigh the
evidence and, after viewing the evidence as a whole, determined that the accused should
155
be committed to trial for second degree murder
 The accused’s certiorari application was dismissed and that decision was affirmed by the
CA
Issue: Did the trial judge, in determining whether the evidence was sufficient to commit the
accused, err in refusing to weigh the Crown’s evidence against that of the accused?
Holding: No; appeal dismissed
McLachlin CJ
The question to be asked by a preliminary inquiry judge under s. 548 of the Code is
whether there is any evidence upon which a reasonable jury properly instructed could
return a verdict of guilty
The task is essentially the same, in situations where the defence tenders exculpatory
evidence, whether it be direct or circumstantial
Where the Crown adduces direct evidence on all the elements of the offence, the case must
proceed to trial, regardless of the existence of defence evidence, as the only conclusion that
needs to be reached is whether the evidence is true
 However, where the Crown’s evidence consists of, or includes, circumstantial evidence,
the judge must engage in a limited weighing of the whole of the evidence (i.e. including
any defence evidence) to determine whether a reasonable jury properly instructed could
return a verdict of guilty
In performing the task of LIMITED WEIGHING, the preliminary inquiry judge does not
draw inferences from facts, nor does she assess credibility
 The judge’s task is to determine whether, if the Crown’s evidence is believed, it would
be reasonable for a properly instructed jury to infer guilt
 This task of limited weighing never requires consideration of the inherent
reliability of the evidence itself
 It should be regarded, instead, as an assessment of the reasonableness of the
inferences to be drawn from the circumstantial evidence
Notwithstanding certain confusing language in Mezzo and Monteleone, nothing in this Court’s
jurisprudence calls into question the continuing validity of the common law rule in
Shephard
Comments
 Notice here the use of the concept of ‘limited weighing’
United States v. Ferras, [2006] 2 SCR 77
Facts
 The US sought the extradition of the accused under the “record of the case” method
provided for in ss. 32(1)(a) and 33 of the Extradition Act
 The records of the case submitted at their committal hearings consist of unsworn
statements from law enforcement agents summarizing the evidence expected to be
presented at each trial
 The US certified that the evidence is available for trial and is sufficient to justify
156
prosecution under the law of the US
 The accused alleged that ss. 32(1)(a) and 33 infringe s. 7 of the Charter because they
allow for the possibility that a person might be extradited on inherently unreliable
evidence
 In both cases, the extradition judges rejected the constitutional objection and committed
the accused for extradition
 The Court of Appeal upheld the decisions.
Issue: Did tendering statements summarizing the evidence to be adduced at trial during the
preliminary inquiry violate s. 7 of the Charter?
Holding: No; appeal dismissed
McLachlin CJ
On the majority view in Shephard, committal may be ordered in the absence of certification that
the evidence is available for trial
 This raises particular concerns in an extradition context because the committal becomes
the final judicial determination that sends the subject out of the country
 If the extradition judge possesses neither the ability to declare unreliable evidence
inadmissible nor to weigh and consider the sufficiency of the evidence, committal for
extradition could occur in circumstances where committal for trial in Canada would not
be justified
Section 29(1) of the Extradition Act, as discussed, requires the extradition judge to be
satisfied that the evidence would justify committal for trial in Canada, had the offence
occurred here
Canadian courts in recent decades have adopted the practice of leaving a case or defence to the
jury where there is any evidence to support it, and have discouraged trial judges from weighing
the evidence and refusing to put a matter to the jury on the basis that the evidence is not
sufficiently reliable or persuasive
 This may explain the conclusion in Shephard that the extradition judge has no discretion
to refuse to extradite if there is any evidence, however scant or suspect, supporting each
of the elements of the offence alleged
 This narrow approach to judicial discretion should not be applied in extradition matters,
in my opinion
It is important to note the differences between extradition hearings and domestic preliminary
inquiries
 Both are pre-trial screening devices and both use the same test of sufficiency of
evidence for committal: whether evidence exists upon which a reasonable jury, properly
instructed, could return a verdict of guilty as in Shephard
 The new Act, however, does not maintain this close parallel in proceedings
o Section 24(2) of the Act grants the extradition judge the same powers as a
preliminary inquiry judge, but requires the judge to exercise those powers in a
manner appropriate to the extradition context
o A second difference comes from the different rules for admitting evidence
 Evidence adduced on extradition may lack the threshold guarantees of
157
reliability afforded by Canadian rules of evidence
o A third difference comes from the ability of extradition judges to grant Charter
remedies
To deny an extradition judge’s discretion to refuse committal for reasons of insufficient
evidence would violate a person’s right to a judicial hearing by an independent and
impartial magistrate, a right implicit in s. 7 of the Charter where liberty is at stake
 It would deprive the judge of the power to conduct an independent and impartial judicial
review of the facts in relation to the law, destroy the judicial nature of the hearing, and
turn the extradition judge into an administrative arm of the executive
 In so far as the majority view in the pre-Charter case of Shephard suggests a contrary
view, it should be modified to conform to the requirements of the Charter
However, s. 32(1)(a) and (b) and s. 33 of the 1999 Act do not violate the right of a person
sought under s. 7 of the Charter, because the requirements for committal of s. 29(1),
properly construed, grant the extradition judge discretion to refuse to extradite on
insufficient evidence such as where the reliability of the evidence certified is successfully
impeached or where there is no evidence, by certification or otherwise, that the evidence is
available for trial
In R. v. Hynes (2001), the SCC (by a 5-4 majority) confirmed the position that a justice
presiding a preliminary inquiry is not a court of competent jurisdiction to consider
whether evidence obtained in violation of the Charter should be excluded
 McLachlin CJ, held that this would change the screening function of the preliminary
inquiry and undermine the expeditious nature of the process
Given the attitude of some defence lawyers to “whack the complainant hard at the preliminary”
some have advocated the elimination of these hearings in cases involving vulnerable victims,
such as cases of sexual assault
In 2002 amendments to the Criminal Code imposed 3 major changes to preliminary
hearings
 Apart from murder, where preliminary inquiries are still automatic, defence counsel must
now request a preliminary (s. 536.4)
o The decision then rests on the competence of the accused’s legal counsel
 Defence counsel must now indicate which issues they want addressed and provide a list
of witnesses to be called
 Defence counsel may need to conduct preliminary inquiries on the basis of hearsay and
other inadmissible evidence subject to a discretion in the justice to order attendance of the
witness for cross-examination (ss. 540(7)-(9))
Constitutional Remedies
Any time after the election of mode of trial is made a pre-trial motion regarding constitutional
remedies can be made
158


Preliminary inquiries are not allowed to grant constitutional remedies
Alternatively, there may be a voir dire during the trial regarding to constitutional
remedies
s. 24 – Canadian Charter of Rights and Freedoms
Enforcement of guaranteed rights and freedoms
(1) Anyone whose rights or freedoms, as guaranteed by this Charter, have been infringed or
denied may apply to a court of competent jurisdiction to obtain such remedy as the court
considers appropriate and just in the circumstances.
Exclusion of evidence bringing administration of justice into disrepute
(2) Where, in proceedings under subsection (1), a court concludes that evidence was obtained in
a manner that infringed or denied any rights or freedoms guaranteed by this Charter, the
evidence shall be excluded if it is established that, having regard to all the circumstances, the
admission of it in the proceedings would bring the administration of justice into disrepute.
As for courts of competent jurisdiction
 The trial court is a court of competent jurisdiction
 The preliminary inquiry is not
 A Superior Court is always a court of competent jurisdiction
Section 24 is usually understood as a compromise between the American exclusionary rule
and the former Canadian position derived from the common law (see Collins)
 Before, the accused would be entitled to criminal and civil penalties against law
enforcement officials, but not the exclusion of evidence
 It appears that the legislature only picked one ground for exclusion of evidence: judicial
integrity
In R. v. Big M Drug Mart Ltd. (1985) the SCC held that any provincial court has the power to
declare laws of no force an effect under s. 52(1) of the Charter
 Apart for this remedy for all courts, those of competent jurisdiction have wide remedies
entrusted on them under s. 24
s. 52(1) Constitutional Act, 1982
The Constitution of Canada is the supreme law of Canada, and any law that is inconsistent with
the provisions of the Constitution is, to the extent of the inconsistency, of no force or effect.
The SCC has yet to pronounce on the range of “appropriate and just remedies”, though courts
have resorted over time to dismissals, stays, costs, return of seized goods or even reduction of
sentences
In R. v. Therens (1983) the Saskatchewan CA held that the exclusion of evidence was an
appropriate remedy under s. 24(1)
159



The SCC in 1985, however, held in this same case s. 24(2) is the sole basis for the
exclusion of evidence in a case of a Charter right infringement
Despite Therens, the SCC has since then recognized an uncertain discretion to exclude
evidence to ensure a fair trial under ss. 7, 11(d) and even s. 24 (1)
o See R. v. Harrer (1995) and R. v. White (1999)
In R. v. Buhay (2003) there is an obiter by Arbour J in recognition of a common law
discretion to exclude evidence obtained in circumstances that would lead to an unfair trial
or if the prejudicial effect outweighs the probative value
Exclusion of evidence
Prior to the Charter courts followed English common law in holding that illegality in
obtaining evidence generally has no bearing on its admissibility
 The leading case of this was A.G. Que. V. Begin (1955) where blood test that proved
intoxication where admissible despite having been obtained from the accused without
warning him that they could be used against him
 The key issue to determine was whether the evidence was relevant or not
o That is whether its probative value outweighed its prejudicial effect
 In 1984, the UK Parliament passed the Police and Criminal Evidence Act which
reconstructed the voluntary confession rule so that evidence admitted by oppression
would be inadmissible in court
 In the US there is an automatic exclusionary rule of illegally obtained evidence
o The fruit of the poisonous tree doctrine
o Much of this derives from constitutional rights protected by the 4th Amendment
and the evidence that civil and criminal penalties against police officers did not
offer enough protection of these rights and deterrence of illegal police conduct
R. v. Collins supra, [1987] 1 SCR 265
Facts
 [For the facts, see the summary of the case under the material on searches]
Issue: When should evidence be excluded under s. 24(2) of the Charter?
Lamer J
S. 24(2) has adopted an intermediate position with respect to the exclusion of evidence
obtained in violation of the Charter
 It rejected the American rule excluding all illegally obtained evidence and the common
law rule that all relevant evidence was admissible regardless of the means by which it
was obtained
The phrase "if it is established that" places the burden of persuasion on the applicant to show
that the evidence would bring the administration of justice into disrepute
 The standard of persuasion required can only be the civil standard of the balance of
probabilities
160
Whether the admission of the evidence would bring the administration of justice into
disrepute is the applicable test
Misconduct by the police in the investigatory process often has some effect on the repute of the
administration of justice, but s. 24(2) is not a remedy for police misconduct at this stage
 The drafters of the Charter decided to prevent having the administration of justice
brought into further disrepute by the admission of the evidence in the proceedings
 It will also be necessary to consider any disrepute that may result from the
exclusion of the evidence
Even though the inquiry under s. 24(2) will necessarily focus on the specific prosecution, it is
the long-term consequences of regular admission or exclusion of this type of evidence on the
repute of the administration of justice which must be considered
Since the concept of disrepute involves some element of community views, the test should be
put figuratively in terms of the REASONABLE PERSON: would the admission of the
evidence bring the administration of justice into disrepute in the eyes of the reasonable
person, dispassionate and fully apprised of the circumstances of the case
 A judge's discretion under this test is thus not untrammelled, for she should not render a
decision that would be unacceptable to the community, provided the community is not
being wrought with passion or otherwise under passing stress due to current events
S. 24(2) directs the judge to consider all the circumstances in determining whether the
admission of evidence would bring the administration of justice into disrepute
 The courts have considered a number of factors, which list is not exhaustive
1. Certain factors are relevant in determining the effect of the admission of
evidence on the fairness of the trial
 The trial is a key part of the administration of justice and its fairness is a
major source of the repute of the system guaranteed by s. 11(d) of the
Charter
 The factors relevant to this determination will include the nature of the
evidence obtained as a result of the violation and the nature of the right
violated and not so much the manner in which the right was violated
 An unfair trial may result with respect to cases where, after a violation of
the Charter, the accused is conscripted against herself through a
confession or other evidence emanating from her
2. A second group of factors relates to the seriousness of the Charter violation
and therefore to the disrepute that will result from judicial acceptance of
evidence obtained through that violation
 Relevant considerations include whether the violation it was deliberate,
whether there were alternative investigative techniques, or whether the
violation was motivated by necessity to prevent the loss or destruction of
the evidence
3. The third group relates to the effect of excluding the evidence: exclusion of
evidence essential to a charge because of a trivial breach of the Charter would
result in an acquittal and would bring the administration of justice into varying
161
degrees of disrepute directly proportionate to the seriousness of the charge
 The more serious the offence, however, the more damaging would be an
unfair trial to the system's repute
4. A final factor, the availability of other remedies, is not relevant
The threshold for exclusion under s. 24(2) is lower than that under the "community
shock test" enunciated in Rothman v. The Queen
 Under s. 24(2), there will have been a constitutional violation as opposed to the absence
of any unlawful behaviour as a result of the resort to tricks in Rothman
 The language of s. 24(2) further indicates a lower threshold
1. The French version of the text, which translates could bring the administration of
justice into disrepute, is less onerous than the more stringent English version,
"would bring the administration of justice into disrepute", and consequently is
preferable in that it better protects the right to a fair trial
The evidence should be excluded notwithstanding the fact that the trial would not be rendered
unfair by its admission or the fact that exclusion could bring the administration of justice into
disrepute by allowing a person convicted at trial of a relatively serious offence to evade that
conviction
 The administration of justice would be brought into even greater disrepute if the Court
did not exclude the evidence and dissociate itself from the conduct of the police which,
assuming it was based on mere suspicion, flagrantly and seriously violated the
individual's rights
 The matter might be clarified at a new trial where the officer could explain his reasons
for his actions
Lecture notes
 While Lamer J states that police actions can bring the administration of justice into
disrepute, the focus is to prevent bringing the administration of justice into further
disrepute by not excluding the evidence
o This highlights that the purpose of this exclusion is judicial integrity
 In terms of what would render a trial unfair, an important element is self-incrimination
o This could result from a forced confession (or conscriptive evidence) or forcing
the accused to provide something from her body which would not be otherwise
obtainable
o The admission of this kind of evidence would bring the administration of justice
into disrepute
 In terms of the second branch the court may consider whether the police was acting in
good faith or whether there were alternative means to obtaining the evidence without
infringing the Charter violation
In Hebert, Sopinka J discussed that when evidence is impugned because it violates the first set
of factors in Collins (trial fairness) it cannot be saved by resorting to the second set of
factors (seriousness of the offence) by showing that the police was not ill-intentioned
 This was upheld by the SCC in R. v. Elshaw (1991)
162
The leading decisions on the exclusion of conscriptive evidence and the consideration of trial
fairness is Stillman
R. v. Stillman supra, [1997] 1 SCR 607
Facts
 [For the facts, see the summary of the case under the material on searches]
Issue: When should conscriptive evidence be excluded under s. 24(2) of the Charter?
Cory J
A consideration of trial fairness is of fundamental importance; a conviction resulting from an
unfair trial is contrary to our concept of justice
 If after careful consideration it is determined that the admission of evidence
obtained in violation of a Charter right would render a trial unfair then the
evidence must be excluded without consideration of the other Collins factors
The primary aim and purpose of considering the trial fairness factor in the s. 24(2)
analysis is to prevent an accused person whose Charter rights have been infringed from
being forced or CONSCRIPTED to provide evidence in the form of confessions,
statements or bodily samples for the benefit of the state
When the trial fairness factor is being considered, it is necessary to classify the evidence as
conscriptive or non-conscriptive based upon the manner in which the evidence was obtained
 The crucial element which distinguishes non-conscriptive evidence from conscriptive
evidence is not whether the evidence may be characterized as “real” or not
If the accused was not compelled to participate in the creation or discovery of the evidence, the
evidence will be classified as non-conscriptive
 Its admission will not render the trial unfair and the court will proceed to consider
the seriousness of the breach and the effect of exclusion on the repute of the
administration of justice
If the evidence, obtained in a manner which violates the Charter, involved the accused being
compelled to incriminate herself either by a statement or by the use as evidence of her body or
of bodily substances, it will be classified as conscriptive evidence
 It is repugnant to fair-minded people to think that police can without consent or statutory
authority take or require an accused to provide parts of their body or bodily substances
in order to incriminate themselves
 The recognition of the right to bodily integrity and sanctity is embodied in s. 7 of the
Charter
A subset of conscriptive evidence is “derivative evidence”
 This is a term frequently used to describe what is essentially conscriptive “real”
evidence; it involves a Charter violation whereby the accused is conscripted against
herself (usually in the form of an inculpatory statement) which then leads to the
discovery of an item of real evidence
 If the evidence under consideration is classified as conscriptive, which in the case of
163
statements includes derivative evidence, then it will be necessary to take the second
step of the analysis and determine whether the admission of the evidence would
render the trial unfair
o As a general rule, this type of evidence will render the trial unfair, unless
it would have been discovered in the absence of the unlawful
conscription of the accused
 Either because an independent source of the evidence exists
 OR the discovery of the evidence was inevitable
o The Crown must bear the onus of establishing discoverability on a balance of
probabilities
o The Court would then proceed to consider the seriousness of the violation
and the effect of the exclusion
Trial fairness summary
1. Classify the evidence as conscriptive or non-conscriptive based upon the manner in
which the evidence was obtained
o If the evidence is non-conscriptive, its admission will not render the trial unfair
and the court will proceed to consider the seriousness of the breach and the effect
of exclusion on the repute of the administration of justice.
2. If the evidence is conscriptive and the Crown fails to demonstrate that the evidence
would have been discovered by alternative means, then its admission will render the trial
unfair
o The Court, as a general rule, will exclude the evidence without considering the
seriousness of the breach or the effect of exclusion
3. If the evidence is found to be conscriptive and the Crown demonstrates that it would
have been discovered by alternative means, then its admission will generally not render
the trial unfair
o However, the seriousness of the Charter breach and the effect of exclusion on
the repute of the administration of justice will have to be considered
The accused’s bodily samples and impressions existed as “real” evidence but the police, by
their words and actions, compelled the accused to provide evidence from his body
 The impugned evidence would not have been discovered had it not been for the
conscription of the accused in violation of his Charter rights and no independent source
existed by which the police could have obtained the evidence
 It follows that its admission would render the trial unfair and this finding is sufficient to
resolve the s. 24(2) issue as the evidence must be excluded
Cory J also found that the breach was also serious and would shock the consentience of the
community
On the issues of the tissue, the Court found that it should not be excluded
 The police did not force, or even request, a mucous sample from the accused, and
although the police acted surreptitiously in disregard for the accused’s explicit refusal to
provide them with bodily samples, the violation of the accused’s Charter rights with
164
respect to the tissue was not serious
 The seizure did not interfere with the accused’s bodily integrity, nor cause him any loss
of dignity
 The police could and would have obtained the discarded tissue, it was discoverable and
the administration of justice would not be brought into disrepute
L’Heureux-Dubé J (dissenting)
 The framework set out in Collins represents the proper approach to s. 24(2) and efforts
since then to explain, clarify and refine or distinguish Collins, have only served to
further muddy the waters
 The classification of evidence under the trial fairness aspect of the s. 24(2) analysis, in
terms of “non-conscriptive ‘real’ evidence” and “conscriptive evidence” is an
unfortunate development
McLachlin J (dissenting)
 Evidence which affects the fairness of the trial must not inevitably be excluded under s.
24(2)
o A proper consideration of “all the circumstances” demands a balancing of each
set of factors set out in Collins
 The principle against self-incrimination does not apply to real evidence, except that
which is derivative from compelled testimony, and there are different degrees of trial
fairness
o Depending on the degree of unfairness and countervailing circumstances, the
fairness of the manner in which the evidence was obtained may or may not result
in rejection of the evidence under s. 24(2)
Lecture notes
 Stillman broadens evidence emanating from the accused from merely the use of
statements, to the use of evidence emanating from the accused’s body or bodily
substances
o What matters if whether the evidence was obtained with or without the
participation of the accused
o Even that which was obtained with the participation of the accused can be
admissible if it could have been discoverable otherwise
 Derivative evidence is real evidence which location is obtained from conscriptive
evidence
 Does discoverability refer to evidence that “could” have been discoverable anyway or
“would” have been discoverable anyway (which is the term employed by the court)
 Criticisms of this decisions point to the fact that conscriptive evidence is arbitrarily
defined
o This case could also lead to a perversion of the discoverability doctrine
There is some debate as to whether the conscriptive categorization of Stillman should be
narrowly applied to statements where the accused incriminates herself, or situations where police
uses the body of the accused or forces the production of bodily samples
165
R. v. Feeney supra, [1997] 2 SCR 13
Facts
 [For the facts, see the summary of the case under the material on searches]
Issue: When should evidence be excluded under s. 24(2) of the Charter?
Sopinka J
Fairness of the trial
The bloody shirt is non-conscriptive, and thus its admission does not go to trial fairness
 The shirt existed in a form useable by the state independent of any actions by the police
and the accused was not compelled to incriminate himself by means of a statement, the
use of the body, or the production of bodily samples
The statements obtained by the police in the trailer in violation of the appellant’s s. 10(b) rights
are clearly conscriptive evidence, and self-incriminating
 The next question set out in Stillman is whether alternative legal means to obtain the
conscriptive evidence existed
o The statements were not “discoverable”, thus their admission would affect the
fairness of the trial
The shoes were observed by the police during the initial unconstitutional search of the trailer
which violated s. 8, yet they are clearly non-conscriptive evidence, given that they are not
compelled statements or bodily samples, and did not involve the use of the appellant’s body
 The admission of the shoes would thus not affect trial fairness
 The cigarettes were also observed by the police in the initial unconstitutional search of
the trailer, yet their admission would not affect trial fairness
The conscriptive statement at Williams Lake about the cash was a sufficient cause for obtaining
the cash, stating as it did the location of the cash
 However, the statement was not a necessary cause of the taking of the cash, so the
money was not conscriptive, derivative evidence
The fingerprints were also taken in violation of the Charter and were conscriptive evidence
It is important to note the distinction between the test for characterizing evidence as
conscriptive, derivative evidence and the test for determining whether conscriptive evidence is
discoverable
 Discoverability is concerned with whether a Charter breach was necessary to the
discovery and obtaining of conscriptive evidence
o If the conscriptive evidence would have been obtained even if the Charter had
not been breached, the evidence is discoverable and its admission, despite the
conscription of the accused, would not affect trial fairness
 The alternative means to obtain the evidence must comply with the
Charter
 The derivative evidence inquiry, on the other hand, is directed at determining
whether a piece of evidence should be viewed as having a conscriptive nature
166
because of its intimate relationship with other conscriptive evidence
o Evidence is derivative evidence if it would not have been obtained but for the
conscriptive evidence
o In analyzing this question, it is not relevant whether the means by which the
evidence would have been discovered in the absence of the conscription were
constitutional
 The unconstitutionality of the second search is a factor to be considered
under other branches of the Collins test
The evidence which does not affect trial fairness must be analyzed in light of the second
and third branches of the Collins test
Seriousness of the Violation
The violations were serious in the present case
 One of the indicia of seriousness is whether the violations were undertaken in good
faith
o One indication of bad faith is that the Charter violation was undertaken without
any lawful authority
o In the instant case, the police did not even have subjective belief in reasonable
and probable grounds for the accused’s arrest
Exigent circumstances did not exist in this case any more than they would exist in any
situation following a serious crime
 After any crime is committed, the possibility that evidence might be destroyed is
inevitably present
o To tend to admit evidence because of the mitigating effect of such allegedly
exigent circumstances would invite the admission of all evidence obtained soon
after the commission of a crime
Effect of Exclusion on the Repute of the Administration of Justice
The admission of the conscriptive evidence would impact on the fairness of the trial and harm
the repute of the administration of justice
 The other evidence, while not conscriptive, was obtained as the result of a very serious
intrusion of the accused’s privacy rights
 Moreover, the evidence was associated with serious violations of the accused’s s. 10(b)
rights, indicating a pattern of disregard for the Charter by the police
The serious disregard for the appellant’s Charter rights in the case at bar suggests that the
admission of the evidence would bring greater harm to the repute of the administration of
justice than its exclusion
 The shirt, shoes, cigarettes and money were inadmissible under s. 24(2), along with the
statements and the fingerprints
 Any price to society occasioned by the loss of a conviction in this case is fully justified
in a free and democratic society which is governed by the rule of law
167
L’Heureux-Dubé J (dissenting)
 The actions of the police did not breach the Charter
 If they had, the exigent circumstances of the case would make it so that the exclusion of
the evidence would bring the administration of justice into disrepute
Lamer CJ (dissenting) disagreed with the way in which Stillman was applied to the facts of the
case, but not the principles laid out in it
In R. v. Grant the Ontario CA questioned the automatic exclusionary rule of evidence because it
is conscriptive or derivative evidence
 Instead, they proposed looking into the other factors of the Collins test and these
outweighed the justification for the exclusion of the evidence due to trial unfairness
 We are waiting to hear from the SCC on this issue
In R. v. Buhay the court established that more attention was to be given to the last two factors of
the Collins test
Judicial stay as abuse of process
Prior to the Charter, the courts developed a common law power for judges to stay criminal
proceedings as an abuse of process, in order to control oppressive prosecutorial practices
 The rule was devised to deter private litigants from engaging in abuse of process
 The power was used sparingly, however
 In R. v. Rourke, the majority of the SCC seemed to reject any notion of this judicial
power
Rourke v. R., [1978] 1 SCR 1021
Facts
 An information was sworn against the accused alleging a kidnapping and robbery and a
warrant of arrest was issued
 At the opening of the trial, the County Court judge granted a stay of the proceedings on
the ground that the accused was prejudiced because of the delay in commencing the
prosecution, a person who would have been a key witness having died in the interval
 An application for mandamus was dismissed by a judge of the Supreme Court of British
Columbia on the basis that the trial judge had jurisdiction to do what he had done and,
having exercised his discretion bona fide, his decision could not be revised
 The Court of Appeal unanimously held that inordinate delay was not a fact which would
give discretion to the County Court judge to stay the proceedings, reversed the judgment
of the Supreme Court and granted the mandamus
Issue: Can judges stay a proceeding for inordinate prosecutorial delay?
Holding: No; appeal dismissed
Pigeon J
168
There is no rule in our criminal law that prosecutions must be instituted promptly and
ought not to be permitted to be proceeded with if a delay in instituting them may have
caused prejudice to the accused
 Nor is there any general discretionary power in courts of criminal jurisdiction to stay
proceedings regularly instituted because the prosecution is considered oppressive
The absence of any provision in the Criminal Code contemplating the staying of an
indictment by a trial judge or an appeal from such decision is a strong indication against
the existence of any power to grant such stay
 Section 7(3) of the Criminal Code which preserves common law defences does not
apply to a discretionary stay of proceedings
It would also be inconsistent with regular procedures of appeal to have a discretionary
jurisdiction that could be exercised by superior court judges in criminal matters where the only
possible appeal would be a direct appeal to the SCC under s. 41(1) of the Supreme Court Act
 Considerations of policy may not be of much importance in the application of explicit
statutory provisions because policy decisions are essentially for Parliament
Laskin CJC
Every court having criminal jurisdiction has the power to stay proceedings which are an abuse
of process or oppressive and vexatious
However, absent any contention that the delay in apprehending the accused had some
ulterior purpose, courts are in no position to tell the police that they did not proceed
expeditiously enough with their investigation, and then impose a sanction of a stay when
prosecution is initiated
 The time lapse between the commission of an offence and the laying of a charge cannot
be monitored by Courts by fitting investigations into a standard mould or moulds
 Subject to such controls as are prescribed by the Criminal Code, prosecutions initiated
a lengthy period after the alleged commission of an offence must be left to take
their course and to be dealt with by the Court on the evidence, which judges are
entitled to weigh for cogency as well as credibility
o The Court can call for an explanation of any improper delay in prosecution and
may be in a position, accordingly, to assess the weight of some of the evidence
In this case, there is no basis, despite the allegations of prejudice, upon which abuse of process
could be invoked
Lectures notes
 Pigeon J saw that a stay could not be appealed like a defence, and this informed his
decisions against recognizing the abuse of process
R. v. Jewitt, [1985] 2 SCR 128
Facts
 The accused was charged with unlawfully trafficking a narcotic
169



At trial, he admitted selling marijuana to an undercover police officer but only because
he was persuaded to do so by a fellow employee who was a police informer
The jury found there had been unlawful entrapment and the court therefore directed "a
stay of proceedings"
A majority of the Court of Appeal dismissed the Attorney General's appeal for want of
jurisdiction
Issues:
1. Is there a discretionary power at common law to stay proceedings in a criminal case for abuse
of process?
2. Is a judicially entered stay of proceedings a "judgment or verdict of acquittal of a trial court",
from which the Crown may appeal to the Court of Appeal under s. 605(1)(a) of the Criminal
Code?
Holding: 1.Yes; 2. Yes; appeal allowed
Dickson CJC
1. The discretionary power to stay proceedings for abuse of process
The inherent jurisdiction of a superior court to stay proceedings which are an abuse of its
process was recognized in Canada as early as 1886
 In recent years, however, uncertainty has clouded the question whether Canadian courts,
apart from powers given to the Attorney General under s. 508 of the Criminal Code,
have a discretion to stay proceedings for abuse of process
 This decision in Rourke was seen by some as a death blow to the doctrine of abuse
of process
The breadth of the decision in Rourke has been the subject of differing views in various
provincial appellate courts
 Abuse of process was most recently reviewed by the SCC in the case of Amato v. The
Queen
 Here, the decisions of Osborn and Rourke were interpreted as being limited to the ratio
of the judgments disposing of the issues arising on the facts of these cases
Adopting the conclusion of the Ontario CA in R. v. Young (1984), "there is a residual
discretion in a trial court judge to stay proceedings where compelling an accused to stand
trial would violate those fundamental principles of justice which underlie the community's
sense of fair play and decency and to prevent the abuse of a court's process through
oppressive or vexatious proceedings"
 A caveat, however, is that this power which can be exercised only in the CLEAREST
OF CASES
2. Whether a stay is a judgment or verdict of acquittal
The stay of proceedings was tantamount to a judgment or verdict of acquittal and subject
to appeal by the Crown pursuant to s. 605(1)(a) of the Criminal Code
 There is no logical reason why a decision to quash an indictment on a question of law
should be considered a judgment or verdict of acquittal whereas a decision to enter a
170
stay on a question of law should not
The decision to stay was not based on procedural considerations, but rather on questions
of law, and such decision was final
Lectures notes
 In justifying the SCC’s change in view Laskin CJC draws examples of cases where a
stay was granted
o Perhaps a better explanation would have been the Charter
R. v. Keyowski, [1988] 1 SCR 657
Facts
 The accused’s first two trials on a charge of criminal negligence causing death ended
with the juries failing to agree on a verdict
 A third trial was stayed by the trial judge on the grounds that it would constitute both an
abuse of process and a violation of s. 7 of the Charter
 The Crown's appeal was allowed by a majority of the Saskatchewan CA and a new trial
ordered
o The CA held that it was necessary for the accused to show prosecutorial
misconduct
Issue: When a series of trials have taken place is it necessary to show prosecutorial misconduct
in order to grant a stay for abuse of process?
Holding: No; the appeal was dismissed for other reasons
Wilson J
A stay of proceedings to remedy an abuse of process is available where the proceedings are
"oppressive or vexatious", but such power can be exercised only in the "clearest of cases"
To define "oppressive" as requiring misconduct or an improper motive would unduly
restrict the operation of the doctrine of abuse of process
Prosecutorial misconduct and improper motivation are two of many factors to be taken
into account when a court is called upon to consider whether or not in a particular case
the Crown's exercise of its discretion to re-lay the indictment amounts to an abuse of
process
Nevertheless the disagreement with the CA, the administration of justice would be best served
in this case by allowing the Crown to proceed with the new trial
 The appellant had not demonstrated this to be one of those "clearest of cases" which
would justify a stay
o The charge is a serious one, the proceedings have not occupied an undue amount
of time and the accused has not been held in custody
o A third trial may indeed, stretch the limits of the community's sense of fair play
but does not of itself exceed them.
[Since the parties did not argue why they agreed that the common law doctrine of abuse of
process was now subsumed in s. 7 of the Charter, this issue was left open]
171
Lectures notes
 Other factors that may lead to abusive process are
o Seriousness of the charge
o Time spent in custody
o Harm caused to the accused
R. v. O’Connor, [1995] 4 SCR 411
Facts
 A bishop was charged with a number of sexual offences
 Defence counsel obtained a pre-trial order requiring that the Crown disclose the
complainants' entire medical, counselling and school records
 The accused later applied for a judicial stay of proceedings based on non-disclosure of
several items
o The Crown asserted that the non-disclosure of some of the medical records was
due to inadvertence on their part
o The trial judge dismissed the application for a stay, finding that the failure to
disclose certain medical records had been an oversight
o He concluded that while the conduct of the Crown was "disturbing", he did not
believe that there was a "grand design" to conceal evidence, nor any "deliberate
plan to subvert justice"
 On the second day of the trial, counsel for the accused made another application for a
judicial stay of proceedings based largely on the fact that the Crown was still unable to
guarantee to the accused that full disclosure had been made
o The trial judge stayed proceedings on all four counts
 The CA allowed the Crown's appeal and directed a new trial
Issues:
1. Is there a need to maintain a distinction between the common law doctrine of abuse of
process and Charter requirements regarding abusive conduct?
2. Did the non disclosure by the Crown justify an order that the proceedings be stayed for abuse
of process?
Holding: 1. No; 2. No; appeal dismissed
L’Heureux-Dubé J
1. The distinction between the doctrine of abuse of process and Charter requirements
The modern resurgence of the common law doctrine of abuse of process began with the
judgment in R. v. Jewitt
 The general test for abuse of process adopted in that case has been repeatedly affirmed
The only instances in which there may be a need to maintain any type of distinction
between the common law doctrine of abuse of process and the Charter regime regarding
abusive conduct will be those cases in which the Charter, for some reason, does not apply
yet where the circumstances nevertheless point to an abuse of the court's process
172
Both individual rights to trial fairness and the general reputation of the criminal justice system
are fundamental concerns underlying both the common law doctrine of abuse of process and the
Charter
 While the Charter is certainly concerned with the rights of the individual, it is also
concerned with preserving the integrity of the judicial system
o Subsection 24(2) of the Charter gives express recognition to this dual role
 The principles of fundamental justice in s. 7 are largely inspired by values that are
fundamental to our common law, on which the doctrine of abuse of process is also
premised
It is clear that abuse of process under the Charter also contemplates important individual
interests
 One often cannot separate the public interests in the integrity of the system from the
private interests of the individual accused
Another justification for this convergence is that remedies less drastic than a stay of
proceedings are available under s. 24(1) in situations where the "clearest of cases" threshold is
not met but where it is proved, on a balance of probabilities, that s. 7 has been violated
 In this respect the Charter regime is more flexible than the common law doctrine of
abuse of process
 There is no reason why such balancing cannot be performed equally, if not more,
effectively under the Charter, both in terms of defining violations and in terms of
selecting the appropriate remedy to perceived violations
There is no one particular "right against abuse of process" within the Charter
 Depending on the circumstances, different Charter guarantees may be engaged
 Abuses may be best addressed by reference to ss. 7 and 11(b) of the Charter
 In addition, there is a residual category of conduct caught by s. 7 of the Charter
o This residual category does not relate to conduct affecting the fairness of the trial
or impairing other procedural rights enumerated in the Charter, but instead
addresses the diverse and sometimes unforeseeable circumstances in which a
prosecution is conducted in such a manner as to connote unfairness or
vexatiousness of such a degree that it contravenes fundamental notions of justice
2. Was the stay justified?
A stay of proceedings is only appropriate "in the clearest of cases", where the prejudice to
the accused's right to make full answer and defence cannot be remedied or where
irreparable prejudice would be caused to the integrity of the judicial system if the
prosecution were continued
Where life, liberty or security of the person is engaged in a judicial proceeding, and it is proved
on a balance of probabilities that the Crown's failure to make proper disclosure to the defence
has impaired the accused's ability to make full answer and defence, a violation of s. 7 will have
been made out
173
In such circumstances, the court must fashion a just and appropriate remedy, pursuant to s.
24(1)
 Although the remedy for such a violation will typically be a disclosure order and
adjournment, there may be some extreme cases where the prejudice to the accused is
irremediable, thus justifying a stay in these "clearest of cases"
[The minority felt that the Crown’s conduct was inappropriate and as such violated the
accused’s right to full answer and defence.
 For Cory J, however, the Crown’s actions did not justify a stay
 For Lamer CJ, the Crown violated fundamental principles of justice underlying the
community’s sense of decency]
Lectures notes
 L’Heureux-Dubé J broadens the remedies that are available for an abuse of process; a
stay is the most drastic remedy
o There are less drastic remedies available under s. 24(1) of the Charter
R. v. L.(W.K), [1991] 1 SCR 1091
Facts
 The accused was charged with 17 counts of sexual assault, gross indecency and assault
relating to his stepdaughter and two daughters
 Since there was a 30 year delay in reporting the abuse, the trial judge ordered a stay on
the basis that a present trial would be contrary to principles of fundamental justice
 The British Columbia CA reversed the stay
Issue: Did the delay in reporting past abuse justify a stay?
Holding: No; appeal dismissed
Stevenson J
Staying proceedings based on the mere passage of time would be the equivalent of
imposing a judicially created limitation period for a criminal offence
 In Canada, except in rare circumstances, there are no limitation periods in criminal law
The comments of Laskin C.J. in Rourke are equally applicable under the Charter
 An accused's rights are not infringed solely because a lengthy delay is apparent on the
face of the indictment
 Courts cannot assess the fairness of a particular trial without considering the
particular circumstances of the case
 A delay may even operate to the advantage of the accused, since Crown witnesses may
forget or disappear
The nature of sexual offences explain even further the delay that took place in reporting these
offences
 If proceedings were to be stayed based solely on the passage of time between the abuse
and the charge, victims would be required to report incidents before they were
psychologically prepared for the consequences of that reporting
174
Since O’Connor, the SCC has continued to vacillate on its approach to abuse of process and
in the application of the residual category for abuse of process
In R. v. Neil (2002), Binnie J, for the SCC, left open the question of whether the common law
doctrine of abuse of process can be invoked when there is no state action in question
 This may well be one of those situations alluded to by L’Heureux-Dubé J in O’Connor,
where the common law doctrine is not subsumed by the Charter, since the Charter
regime regarding abusive conduct does not apply
Pleas and Plea Bargaining
S. 606 of the Criminal Code allows the accused plead guilty or not guilty
 Such a plea will be accepted by the court if it is proved that the accused has made the
plea voluntarily and understood the nature of the charge
 Subsection 1.2 states that the failure of the court to verify the previous conditions does
not necessarily vacate the plea
s. 606 – Criminal Code
Pleas permitted
(1) An accused who is called on to plead may plead guilty or not guilty, or the special pleas
authorized by this Part and no others.
Conditions for accepting guilty plea
(1.1) A court may accept a plea of guilty only if it is satisfied that the accused
(a) is making the plea voluntarily; and
(b) understands
(i) that the plea is an admission of the essential elements of the offence,
(ii) the nature and consequences of the plea, and
(iii) that the court is not bound by any agreement made between the accused and
the prosecutor.
Validity of plea
(1.2) The failure of the court to fully inquire whether the conditions set out in subsection (1.1)
are met does not affect the validity of the plea.
Refusal to plead
(2) Where an accused refuses to plead or does not answer directly, the court shall order the clerk
of the court to enter a plea of not guilty.
Allowing time
175
(3) An accused is not entitled as of right to have his trial postponed but the court may, if it
considers that the accused should be allowed further time to plead, move to quash or prepare for
his defence or for any other reason, adjourn the trial to a later time in the session or sittings of
the court, or to the next of any subsequent session or sittings of the court, on such terms as the
court considers proper.
Included or other offence
(4) Notwithstanding any other provision of this Act, where an accused or defendant pleads not
guilty of the offence charged but guilty of any other offence arising out of the same transaction,
whether or not it is an included offence, the court may, with the consent of the prosecutor,
accept that plea of guilty and, if the plea is accepted, the court shall find the accused or
defendant not guilty of the offence charged and find him guilty of the offence in respect of
which the plea of guilty was accepted and enter those findings in the record of the court.
Video links
(5) For greater certainty, subsections 650(1.1) and (1.2) apply, with any modifications that the
circumstances require, to pleas under this section if the accused has agreed to use a means
referred to in those subsections.
R. v. Lessard (1976), 33 CRNS 16 (Ont. CA)
Facts
 The complainant (eighty-three years of age) is the step-grandmother of the respondent
who was charged of indecent assault
 The accused pleaded not guilty but did not testify in his own defence
 On the completion of the evidence, the trial judge found him guilty
 When the proceedings resumed for sentencing purposes, defence counsel moved to
reopen the case stating that his earlier decision not to call the accused was based on the
"mental state and condition of the accused" and that the accused was nervous and,
therefore, it was not in his best interest at the time to give evidence
 The trial judge granted the request as an unusual exercise of discretion, and after hearing
the testimony of the accused he acquitted him
Issue: Was the trial judge allowed to reopen the case after the finding that the accused was
guilty?
Holding: Yes; appeal dismissed
Martin JA
The word "conviction" is equivocal
 It is often used both to denote the adjudication of guilt and to denote the adjudication of
guilt together with the judgment of the Court upon the finding of guilt, that is, the
sentence
A judge exercising the functions of both judge and jury is not functus officio following a
finding of guilt until she has imposed sentence or otherwise finally disposed of the case
176
 It has been recognized for long that a trial judge may permit an accused who has pleaded
guilty to change his plea at any time before the imposition of sentence, notwithstanding
the acceptance of the plea by the Court, because up to that time the proceedings have
not been completed
Where the Court has dismissed the charge it is, of course, functus officio
 Since the proceeding has terminated the Court is not empowered to reopen the case and
vacate the acquittal
In the case of a trial by jury, the trial judge has no power to set aside the verdict of the
jury and direct a rehearing where the verdict has been arrived at on disputed facts on a plea of
not guilty
 The separation of the functions of the judge and the jury is incompatible with the
existence of such a power in the trial judge
 The immunity of the jury's verdict from interference by the trial judge is seen in the fact
that the trial judge, neither at common law nor under the Code, is empowered to set
aside the verdict of a jury in a criminal case on the ground that it is against the evidence
Since the trial Judge had a discretion to reopen the case and to permit the accused to give
evidence, the exercise of his discretion, unless he has failed to exercise it judicially, is not
subject to an appeal by the Crown which can be maintained only upon a "ground of appeal
that involves a question of law alone" under s. 605(1)(a) of the Code
Adgey v. R. (1973), 23 CRNS 298 (SCC)
Facts
 The accused pleaded guilty to several charges of false pretences, a charge of fraud, and a
charge of break, enter and theft
 After plea had been entered, the facts giving rise to each charge were related by a police
officer and the accused was afforded an opportunity to explain
 He was was convicted. No request for the change of plea was made during the hearing
 The appellant appealed to the CA which allowed the appeal as to one of the charges and
dismissed the appeal as to all other charges
Issue: Having heard the accused’s explanation, did the judge err in failing to strike the guilty
pleas and direct trial on the charges?
Holding: No; appeal dismissed
Dickson J
There are two stages in the proceedings in which a trial judge has a discretion so far as
accepting a plea of guilty is concerned
1. When the charge is read and the plea is entered
2. Second, following the hearing of evidence, if the judge chooses to hear evidence
A trial judge is not bound, as a matter of law, in all cases to conduct an inquiry after a
guilty plea has been entered
 If, however, evidence is heard, it may indicate that the accused never intended to admit
to a fact which is an essential ingredient of the offence or that she may have
177

misapprehended the effect of the guilty plea or never intended to plead guilty, in any of
which the judge may in her discretion direct that a plea of not guilty be entered or
permit the accused to change his plea to not guilty
This discretion if exercised judicially will not lightly be interfered with
An appellant can however later change her plea if she can satisfy the Appeal Court that there
are valid grounds for his being permitted to do so.
On reviewing the charges to which the appellant pleaded guilty neither the facts admitted nor
the statements made following the guilty pleas justify disturbing the exercise of discretion by
the trial judge
Laskin J (dissenting)
The accused did not have counsel until, on his third appearance in Court and on his trial at that
time, duty counsel stood with him
The fact that the trial judge did not inquire if duty counsel had had an opportunity to
consult with the accused and to ascertain the factual bases of the charges and whether the
accused was willing to be represented by him should there be a plea of guilty was sufficient to
require that the convictions be set aside and a new trial ordered
The duty of a trial judge respecting an inquiry into the facts on a plea of guilty is to satisfy
herself that the accused understands the nature of the charge and its consequences and is
unequivocal in his plea of guilty, but this must be complemented by the duty of the Crown to
adduce facts which, taken to be true, support the charge and conviction
 In the present case on the facts narrated by the Crown there was sufficient doubt on the
elements of the offence to warrant striking the plea
R. v. Moser (2002), 163 CCC (3d) 286 (Ont. SCJ)
Facts
 Application by the accused to set aside guilty pleas
 Moser pleaded guilty to charges of assault with a weapon and forcible confinement
 He admitted that he had accosted a 16- year-old girl and dragged her into his apartment
at knifepoint but disputed the fact that he had deliberately removed some of her clothing
 At the time of the plea, Moser was aware that the Crown intended to proceed with a
dangerous offender application
 In the assessment after the pleas, Moser denied using a knife in the assault
 The psychiatrists found that there was considerable risk that Moser would re-offend
violently or sexually
 Moser argued that he pleaded guilty because he did not want to put the girl through the
ordeal of giving evidence
Issue: Should the accused’s guilty pleas be set aside?
Holding: No; appeal dismissed
Hill J
178
An accused's plea of guilt is a fundamentally significant step in the criminal trial process.
 The plea relieves the Crown of the burden to prove guilt beyond a reasonable doubt, the
presumption of innocence, the right to silence, and the right to make full answer and
defence to the charge are at an end
Where the offender is represented by counsel in a guilty plea proceeding, the trial court
has a discretion, as a matter of law, to hold no further inquiry into the factual circumstances
alleged in support of the offences charged
 If the circumstances raise doubt as to the validity of the plea, the trial judge has a
discretion to undertake an inquiry, the extent of which will vary according to the
particular facts of each case
Where the accused is self-represented, common law notions of fairness, if not
constitutional concerns, mandate more extensive judicial involvement in the plea process
A guilty plea, to be considered valid, must have minimally sufficient characteristics in order to
provide an assurance that the forfeiture of a trial is fair
 To be valid, the plea must be unequivocal - the circumstances should not be such that
the plea was unintended or confusing, qualified, modified, or uncertain in terms of the
accused's acknowledgement of the essential legal elements of the crime charged
 A plea of guilty must be voluntary in the sense that the plea is a conscious volitional
decision of the accused
Plea negotiations in which the prosecution pursues a plea of guilt in exchange for forgoing legal
avenues open to it, or agrees not to pursue certain charges, do not render the subsequent plea
involuntary
 What is unacceptable is coercive or oppressive conduct of others or any circumstance
personal to the individual which unfairly deprives the accused of free choice in the
decision not to go to trial
Some circumstances calling into question the voluntariness of a guilty plea are
 Pressure from the court
 Pressure from defence counsel
 Incompetence of defence counsel
 Cognitive impairment or emotional disintegration of the accused
 Effect of illicit drugs or prescribed medications
A guilty plea's validity depends on the plea being informed
 It is essential that the accused understand the nature of the charges faced, the legal effect
of a guilty plea, and the consequences of such a plea
 The accused must generally know the jeopardy faced by way of possible punishment
 The prior experience of the accused in the criminal justice system is one factor weighing
toward the validity of the accused's plea as she has had the opportunity to participate in
the process
In the absence of any circumstances in the record, or a challenge to the competence and
179
professionalism of trial counsel, a trial judge is justified in drawing the inference that
counsel took the necessary steps to ensure the client understood the nature and
consequences of a guilty plea
We must have finality to proceedings unless the demands of justice dictate otherwise:
 Exceptional circumstances calling into question the validity of a guilty plea warrant a
judicious exercise of discretion to inquire into the legitimacy of the plea
o Where the circumstances are made known to the trial court, the presiding justice
is empowered with a discretion to conduct an inquiry into whether the plea
should be rejected or struck and the case proceed to trial
o Statements made in the course of the court's inquiry into the facts supporting the
allegations immediately following a plea of guilt, although not admitted by the
Crown, may justify the court in rejecting the guilty plea
 At the same time, the court must be vigilant that an experienced criminal not be allowed
to abandon a position when matters "did not play out as expected"
While the accused carries the burden of persuading the court the plea is invalid and ought
to be withdrawn, the jurisprudence is not clear as to the standard of persuasion
 Some authorities advocate a balance of probabilities standard and other for a heavier one
As for the present case
 Moser was attempting to avoid the potential impact of a dangerous offender finding on
his sentencing
 He knew the seriousness of the dangerous offender application when he pled
 The plea was unequivocal, voluntary, and informed
 As well, Moser had participated in prior guilty plea proceedings
 He retained experienced criminal counsel
 He admitted that his consumption of cocaine at the time of the assault affected his
recollection of the facts
Plea Bargaining
Plea bargaining is not something that is provided by the Code
 Before the 1990s courts were reluctant to enforce them
 Nevertheless, 80% of cases are typically resolved by a guilty plea
 It was finally agreed that they should be used in order to relieve institutional resources
 Certain promises are within the discretion of the prosecutor to offer for a guilty plea
Plea bargaining may be formal or involve a hurried hallway discussion
 The are usually entered into following a discussion between Crown attorneys and the
defence counsel but sometime, although less frequently, the judge is involved
Klein, Let’s Make a Deal
From the offender’s point of view, the kickback is generally the most beneficial kind of deal
180
The benefits obtained by law enforcement officials are sometime more apparent than real
 Some offenders may not be discourage from engaging in criminal activities
 Others may resort to crime to finance their trials if left out on bail awaiting trial
Information is the livelihood of any police department
 However, for many offender the mental and physical costs of becoming an informer
outweigh the costs
 Police often emerge as frequent and significant actors
o They are the gatekeepers to the criminal justice system
The need for bargaining often involves crime prevention, law enforcement and the
conservation of limited resources
Ericson, Making Crime: A Study of Detective Work
The bench in Canada has done little more than assert their disapproval of plea bargaining,
giving the impression that it is sporadic rather than established and widespread
In making their case to Crown attorneys, detectives rely upon several organizational elements
which allow them to effect considerable control, one being their superior knowledge of the
cases before them
 Crown attorney are likely to defer to detective in making deals for guilty pleas
Ferguson and Roberts, Plea Bargaining: Directions for Canadian Reform
The range of benefits offered in exchange for plea bargains include
 Reduction in charge
 Withdrawal of other charges
 Recommendation for a particular sentence
 The use of summary conviction
 Promise not to force trial by jury
 Promise not to charge friends or family of the accused
 Promise as to the place of imprisonment, type of treatment and time of parole
 Promise to arrange sentence before a lenient judge
 Promise not to oppose release on bail or while awaiting sentence
Ministry of Attorney General for Ontario Crown Policy Manual (2005)
In conducting resolution discussions, Crown counsel should:
 Attempt to balance the interests of the victim, protection of the public and the
rights of the accused in the framework of optimal use of limited resources
 A proposed sentence should reflect the public interest and the gravity of offence
Fundamental principles that are binding directives
 Crown counsel must not accept a guilty knowing that the accused is innocent
 Crown must not accept a guilty plea to a charge that can never be proven unless that fact
is fully disclosed to the defence
181


Crown counsel must not purport to bind the Attorney General’s right to appeal any
sentence
Unless for exceptional circumstances, Crown counsel must honours agreements reached
Kipnis, Criminal Justice and the Negotiated Plea
The appeal that plea bargaining has is rooted in our attitude towards bargaining in general
 Where both parties are satisfied with the terms of an agreement, it is improper to
interfere
 Judicial neutrality may be commendable were entitlements are being exchange, but in
the criminal justice system what matters is not whether people receive what they
have bargained for, but what they deserve
In its coercion of accused and the relaxation of conviction and sentencing standards, plea
bargaining falls short of the justice we expect of our legal system
While we need to alleviate economic injustices on the one hand and decriminalize some
victimless crime, we also need to demand respect of our legal institutions, which plea
bargaining does not accomplish
In general, courts have been reluctant to give any legal effect to a plea bargain
 However, on occasion, an attempt by the Crown to repudiate the bargain will be
considered an abuse of process
R. v. D.(E.) (1990), 73 OR (2d) 758
Facts
 The accused was charged with two counts of having sexual intercourse with a female
under 14 and two counts of having illicit sexual intercourse with the daughters of his
common law spouse
 After a period of counselling, the complainants decided that charges against the accused
should be pressed
 At the outset of the trial, the accused moved pursuant to s. 24 of the Charter for a stay of
proceedings
 The trial judge found that the accused's rights under s. 11(b) and (d) of the Charter had
not been violated so that there was no basis on which a remedy could be
 However, he went on to hold that to proceed against the accused in the circumstances
would constitute an abuse of process and he invoked his inherent jurisdiction to order a
stay of proceedings
Issue: Did the trial judge err in staying the proceedings?
Holding: Yes; appeal allowed
Arbour JA
The burden is on an accused to prove the abuse of process on a balance of probabilities
 The accused must show that allowing the state to proceed against him would violate the
community's sense of fair play and decency or that his trial would be an oppressive
proceeding
182
 Here, the trial judge held that it was offensive to the principles of fair play and decency
to allow the complainants to change their minds three years after the police had
informed the accused that charges would not be laid, especially since the accused had
heeded the police warning not to contact them
The police decision not to lay charges against the accused in 1984 could not be faulted,
considering the complainants' reluctance to proceed
 The conduct of the complainants, likewise, could not be faulted
Absent prejudice to the accused or other circumstances leading to unfairness, the complainant’s
willingness to lay charges at 22 offended neither common sense nor fair play
 To hold otherwise would require young victims of sexual abuse to speak up immediately
or never be heard
Several courts have found it to be an abuse of process for the Crown to renege on an
agreement with the accused
 Typically, those cases involved "deals" in which the accused had genuinely
compromised her position and made a real concession in anticipation of some
reward, such as the abandonment of prosecution, so that the accused was
prejudiced by the Crown reneging on the deal
In this case, even if the discussion between the accused and the police officer in which the
accused was warned to stay away from the family and told that no charges would be laid could
be characterized as an agreement, the accused made no real concession and did not turn over
evidence or in any way compromise his position
 He was not facing more serious charges now than he would have faced in 1984
 There was no evidence that his life was disrupted, that his reputation had to be restored
or that his working life was interfered with in the intervening period
 There was no evidence of trickery, improper motives or malice on the part of the police
or the complainants
 The potential prejudice to the accused in making full answer and defence could be
resolved at trial
Law Reform Commission of Canada. (1975). Working paper no. 15: Criminal procedure:
Control of the process
Plea bargaining is now an established practice in Canada
 Much of the controversy surrounding it results from disagreement as what the practice is
 Here it is defined as a guilty plea by the accused in return for the promise of some
benefit
The plea bargaining is made possible by the accused’s right to plead guilty, while relinquishing
her right to trial
Objections to plea bargaining are overwhelming
 It detracts from the pursuit of legitimate goals of the criminal justice system, and
destroys the appearance and reality of justice
183




As long as it exist, the parties will adopt tactics to maximize their bargaining power
Neither the public interest nor the interests of the parties can be properly served
It is generally conduct in secret
Justice should not be seen as something that can be purchased or negotiated at the
bargaining table
Plea bargaining is supported by some as a desirable method of achieving compassion and
flexibility
 It may a way to maintain a high proportion of guilty please
o However, it is possible that the causal connection between plea bargaining and
guilty pleas is exaggerated
 There are other ways to encourage guilty please
 The parties would be act more realistically if they didn’t need to prepare for negotiations
Judicial participation in the plea bargaining is rare
The commission thus recommends that plea bargaining be eliminated
Guest Speakers’ Presentation
Interests of the Crown in plea bargaining
 Protection of society and the public
o Is the accused likely to reoffend?
o Was there contribution of the victim to the wounds that she/he received?
 Within this, the interests of the victim and the interest of justice
 There is no case to win for the Crown, and one has to keep an open mind to the view of
the defence
 If the accused is not represented, the Crown is very aware of not abusing its power during
the bargaining process
When the Crown authorizes a file it has already taken into consideration the evidence against the
accused and the possibility of securing a conviction
 At the point of the preliminary inquiry, if the victim is absent, there is hardly any
evidence
o There is no person to identify the accused and document the crime
The judge should follow a joint-submission from the Crown and the defence, unless it is
unreasonable
 See R. v. GWC or Hollwick
Before the 1990s plea agreements were frowned upon and not highly recognized by the courts as
a legitimate practice
 Since then they have been recognized as a legitimate practice and part of the system of
criminal justice
 Without it the system would be impractical
184

The Karla Homolka case brought up many concerns related to plea bargaining
o Yet, when the Crown entered the plea agreement for manslaughter and a 12 year
sentence, there was almost no evidence against ether her or Paul Bernardo
It is often true that pleas are just decisions made by the Crown and the defence in closed
chambers, but their decisions are ultimately approved and made legitimate by a judge
 It is the judge who decided whether the plea is reasonable or not
 The judge ensures that the accused has made a free and voluntary decisions without fear
of threats
 Unfortunately, the media has given the plea bargaining process a very bad reputation
A guilty plea is a mitigating factor for sentencing purposes
 This is particularly the case when the accused is an informant of the criminal activities of
others
An important tool to use during the plea bargaining for the defence is to deal with an offence
through a summary conviction instead of an indictable offence
 Another is to receive concurrent sentences as opposed to consecutive ones
o This is appealing for the accused and recognizes the crime committed against
each victim
When entering the plea bargaining as a defence attorney, the first thing to consider is
whether the accused recognizes some level of culpability
 Otherwise there is not ground to negotiate
 It is essential to have the consent of the accused to negotiate
 The criminal past of the accused
S. 810 of the Code imposes conditions on the accused to be released in order to protect the
victim
The Trial Process: Preliminary Considerations
When the types of decisions that are discussed here are made by the trial judge, appeals to
these decisions are made as part of the appeal process of the ultimate verdict
Right to counsel at trial (Charter s. 7)
Before we discussed how there is no right to free counsel under s. 10(b)
 This deals with the period of arrest
The traditional common law right to make full answer and defence is often the tool used by
the trial judge to guarantee the right to counsel at trial
185
 Express provisions have long be in the Criminal Code (see ss. 650(3), 684 and 802)
Barrette v. R., [1977] 2 SCR 121
Facts
 Charged with assaulting a peace officer the accused was sentenced to imprisonment for
one year
 At his trial the accused filed an application for adjournment because his counsel was not
present
 The judge denied the application on the grounds that the case dated back to the previous
November and that counsel, who was occupied elsewhere, had not justified his absence
o The accused was therefore directed to proceed without the assistance of counsel
 The majority of the CA held that the accused, even though not represented by counsel,
was given the opportunity to make a full defence and received a fair trial, and refused to
order a new trial
Issue: Should the accused be directed to proceed without a counsel at trial when his lawyer is
unavailable?
Holding: No; appeal allowed and a new trial ordered
Pigeon J
Despite that the fault of counsel constituted prima facie contempt of court, there was
nothing which authorized the trial judge to presume the complicity of the accused or
without any evidence to lay the blame for the fault of counsel on him
The accused has the right “to make full …defence personally or by counsel” (s. 577(3))
 An adjournment necessary for the exercise of this right can only be refused for a
reason based on established facts
 In the present case, the accused cannot be held responsible for the fact that too many
cases are postponed because lawyers are not present
Although the decision on an adjournment necessary for the exercise of this right is in the
judge’s discretion, the judge must exercise this discretion judicially
 Her decision may thus be reviewed on appeal if it is based on reasons which are not
well founded in law
 This right of review is especially wide when the consequence of the exercise of
discretion was that someone was deprived of his rights, whether in criminal or in civil
proceedings
As to the English judgments where the Court refused to quash the conviction of accused
persons deprived of the services of counsel, it must not be overlooked that at that time in
England the quashing of a verdict by the CA meant the definitive’ acquittal of the accused
 Hence the tendency to uphold a conviction, despite an error of law, if there was no
miscarriage of justice
It cannot be said in the case at bar that the accused suffered no prejudice by being forced to
defend himself without enjoying the assistance of counsel, and without being able to summon
as a witness a person having knowledge of the incident which led up to the conviction
186

When the case of the accused is such that she cannot defend herself without
testifying, she is certainly in great need of assistance of counsel
While it is true that counsel for the prosecution treated the accused with consideration, it cannot
be concluded that he had a fair trial
De Grandpre J (dissenting)
As it must be determined whether a miscarriage of justice was perpetrated by the trial judge in
the exercise of his discretion, and his decision was upheld by the CA, this SCC must interfere
only if it is clear that the judgment a quo is based on an error of principle
The Court of Appeal of England has intervened in cases of this kind only when the fact that the
accused was not represented by counsel might have constituted a denial of justice and have
modified the result of the trial
The accused has not convinced this Court that the presence of his lawyer would have changed
the outcome, indeed to the contrary
 The right to the presence of counsel is a right which has limits, and the
administration of justice requires that society be protected as well
Deutsch v. Law Society of Upper Canada (1986), 47 CR (3d) 166
Issue: Do individuals possess a right under the Charter to be provided with a funded counsel to
defend them at trial against criminal charges for an indictable offence?
Holding: No; appeal dismissed
Craig J
The right to a funded counsel has not been entrenched by s. 7 and/or s. 11(d) of the
Charter
 With the exception of the language provisions, most of the rights guaranteed by the
Charter are expressed in negative terms in the sense that they require that the state
refrain from certain activities
 To impose a constitutionally entrenched positive duty on the government to expend
public funds on the defence of persons accused of crimes would require a specific
guarantee in express language
During the trial, the court was made aware of documentation that showed that there was a
proposal to amend s. 10 of the Charter to include the obligation to supply counsel for those
who could not afford it; this proposed amendment was defeated
 The Minutes the proceedings of the Special Joint Committee of the Senate and House of
Commons on the Constitution of Canada become persuasive authority for the
proposition that the Charter was not intended to en trench a right to funded counsel
Under the common law the accused has a right to a fair trial and the trial judge is bound to
ensure that an accused person receives a fair trial
 Where the accused faces possible imprisonment, pursuant to s. 7 of the Charter, the
187


accused has an entrenched right not to be deprived of his liberty except in accordance
with the principles of fundamental justice
Also pursuant to s. 11(d) he has an entrenched right to a "fair and public hearing ..."
The right to fundamental justice and a fair and public hearing includes the right to
a fair trial
There may be rare cases where legal aid is denied to an accused person facing trial, and
where the trial judge is satisfied that, because of the seriousness and complexity of the
case, the accused cannot receive a fair trial without counsel
 In such a case it seems to follow that there is an entrenched right to a funded counsel
under the Charter
 Nevertheless, this is not new; it is the same right enjoyed by an accused at common
law
It may still be a difficult question for a trial judge to decide whether a funded counsel shall be
appointed
 First, because an accused has an absolute right to proceed without counsel regardless of
the complexity of the case
 Second, where an accused has been denied legal aid without error in law or jurisdiction
on the part of the Legal Aid authorities, and/or where the accused refuses to comply
with reasonable requirements of those authorities, his appearance in court without
counsel may be construed as an exercise by him of his right to proceed without counsel
 It is unnecessary for the court to deal with administrative details, as these would be dealt
with on a case by case basis through the Department of the Attorney General of Ontario
Lecture Notes
 In arriving at the conclusion that right to free counsel is not constitutionally guaranteed,
the court makes reference to legislative history and the refusal of Parliament to include
this in the Charter despite human rights treaties
R. v. Peterman (2004), 19 CR (6th) 258
Facts
 The accused was charged with two counts of arson
 Legal Aid Ontario issues him a legal certificate, and a lawyer who was four hours away
and had acted for the accused before, agreed to act for the accused on the certificate
 The lawyer’s application for travel expenses to Legal Aid was denied given the
availability of local counsel
 The court granted an application brought against Legal Aid to pay for the lawyer’s travel
expenses, preparation time and junior counsel
o The trial judge found the application reasonable and required for fair
representation
o She also ordered costs against the Crown
Issue: Should the accused be entitled to receive from Legal Aid travel and other additional
expenses for her lawyer?
Holding: No; appeal allowed
Rosenberg JA
188
The state has a constitutional obligation to ensure that indigent accused receive a fair
trial, and in many cases that means ensuring that the accused is represented by counsel
 Under the Ontario legal aid certificate system, it is because defence lawyers are willing
to accept legal aid certificates that the state is able to fulfill its constitutional obligation
 The importance of the work that defence counsel perform for legal aid clients cannot be
underestimated
Nevertheless, the obligation for setting legal aid rates and policies relating to retention of
out-of-town counsel and of junior or co-counsel lies with Legal Aid Ontario, not the court
 A criminal trial court has no jurisdiction to review those policies
 A criminal trial court's jurisdiction rests solely on the obligation to ensure that an
accused person receives a fair trial
In some cases, the court will be satisfied that if an accused is not represented by counsel,
her right to a fair trial as guaranteed by ss. 7 and 11(d) of the Charter will be infringed
 If such an accused lacks the means to employ counsel privately, but has nevertheless
been refused legal aid, the court can make an order staying the proceedings until the
necessary funding for counsel is provided by the state
As per Rowbotham "there may be rare circumstances in which legal aid is denied but the trial
judge, after an examination of the means of the accused, is satisfied that the accused, because of
the length and complexity of the proceedings or for other reasons, cannot afford to retain
counsel to the extent necessary to ensure a fair trial"
 However, when a court makes a Rowbotham order, it is not conducting some kind of
judicial review of decisions made by legal aid authorities; rather, it is fulfilling its
independent obligation to ensure that the accused receives a fair trial
Summary of state obligations regarding a right to counsel
 The Charter guarantees to a fair trial and fundamental justice mean that the state must
provide funds so that an indigent accused can be represented by counsel where counsel
is required to ensure that the accused person has a fair trial
 Further, within reason, the court will protect an accused's right to choose his or her
counsel, as per common law principles
 Absent compelling reasons, such as a disqualifying conflict of interest or incompetence,
the courts will not interfere with an accused's choice of counsel
 Further, the courts will avoid actions that result in accused persons being improperly or
unfairly denied the opportunity to be represented by their counsel of choice
However, the right of an accused person to be free of unreasonable state or judicial
interference in her choice of counsel does not impose a POSITIVE obligation on the state
to provide funds for counsel of choice
There would appear to be two exceptions to this general proposition
1. In some unique situations it may be that an accused can establish that she can only
obtain a fair trial if represented by a particular counsel
189
o In those unusual circumstances, the court may be entitled to make an order to
ensure that the accused is represented by that counsel
2. In unusual circumstances, the court may find that the accused simply cannot find
competent counsel to represent him or her on conditions imposed by Legal Aid
o Those cases will be rare
In this case, there was never any dispute that the respondent required counsel to ensure that he
had a fair trial and that he could not fully fund it
 He was therefore granted a contributory legal aid certificate
 The application judge was, again, not entitled to review the reasonableness of the
decisions made by Legal Aid
o Her focus had to be on whether the respondent's right to a fair trial was imperiled
because of the conditions under which he was being defended
o There was no evidence to support a finding that the respondent's right to a fair
trial was at risk
o The respondent's case is not unique and it is not of an order of complexity that
requires special counsel
o This was also not a case where the accused could not otherwise obtain competent
counsel
 The application judge could have adjourned the case to permit the
respondent time to retain local counsel
 Alternatively, having regarded the circumstances, the application judge
could have required the accused’s lawyer to proceed with the case since
he was on the record and had undertaken to defend the respondent
Lecture Notes
 When the court finds that not having a counsel present will lead to an unfair trial, the
judge can issue a stay until the state can fund the counsel
 To prove that a lawyer must be funded by the state the accused must:
o First establish that he has been denied legal aid and cannot afford a lawyer
o That not having a counsel present will jeopardize the fair trial
 The court has to consider the seriousness of the charges
 The length and complexity of the case
 The ability of the accused to represent herself effectively at trial
Courts now frequently acknowledge that the right to make full answer and defence protected
under ss. 7 and 11(d) includes a right to effective assistance of counsel
 However, appeal courts have been reluctant to order a new trial on the basis of
incompetence of counsel and often hesitate to second-guess tactical decisions made at
trial
 When this matter reached the SCC in R. v. B.(G.D) the court stated that an
appellant was required to establish incompetence and that a miscarriage of justice
resulted
o Incompetence was assessed based on the Strickland test, which employs a
reasonable standard of professional assistance
o Miscarriage of justice can take many forms and include procedural unfairness
190
Where an accused in unrepresented, there are important obligations imposed on trial judges
 See R. v. Tran (2001) and R. v. Moghaddam (2006)
Adjournments
There are numerous provisions scattered across the Code dealing with judge’s adjournment
powers
 S. 516 permits the adjournment of a bail hearing, and s. 537 the adjournment of a
preliminary inquiry
 With respect to indictable offences, s. 571 (relating to non-jury trials) and s. 645 (for jury
trials) allow the judge to adjourn the trial from time to time
 S. 803 permits an adjournment on a matter of summary conviction
R. v. G. (J.C) (2004), 189 CCC (3d) 1
Facts
 The accused was charged with robbery and aggravated assault
 During trial the Crown sought an adjournment, because the complainant (who had been
subpoenaed by ordinary mail) was not present in court
o The Crown advised the judge that the complainant, who live with her parents,
was missing
 The trial judge refused to grant the adjournment and the accused was acquitted
Issue: Should an adjournment be granted when the complainant is unable to appear in court?
Holding: Yes; appeal allowed
Dalphond JA
It is undisputed that whether an adjournment or a postponement should be granted or not
is a discretionary matter for the trial judge
o Such judicial discretion can however be reviewed on appeal if it has not exercised
judicially
o The test for appellate review is whether the trial judge has given sufficient weight
to all relevant considerations
o Of course, if the judgment is based on reasons that are not well founded in law, a
court of appeal may also intervene
Amongst the elements to be considered by a judge when asked to grant an adjournment of
a criminal trial due to the absence of a witness are the following (R. v. Darville)
1. That the absent witness is a material witness in the case
2. That the party applying for an adjournment has been guilty of no laches or neglect in
omitting to endeavour to procure the attendance of this witness
3. That there is a reasonable expectation that the witness can be procured at the future time
to which it is sought to put off the trial
 It is also proper for a trial judge when asked for a postponement to consider other
relevant circumstances such as the gravity of the charges, the number of previous
postponements and the consequences of a postponement for the accused
191
A trial judge errs in law by refusing a request for an adjournment without having given
the party seeking it an opportunity to demonstrate that the conditions described above are
met
 The decision whether or not to grant the adjournment must be made in the light of the
realities of each case and shall be consistent with the interests of justice
In the case at bar, the first criterion mentioned in Darville was met; her presence was critical to
establish the case against the respondent
 As for the second criterion, whether the Crown negligent, if the trial judge was
unsatisfied with the Crown’s recital of the facts he should have said so and then ordered
an adjournment to provide the Crown the opportunity to bring before him proper
evidence of what really happened
o There ought to be no presumption that using a mailed subpoena constituted
neglect or that it did not
 There was no evidence made before the trial judge that if the complainant
had been personally served with a subpoena, she would not have run
away
o The Crown which had just found out about the disappearance of the
complainant, could not then be blamed by the trial judge for not knowing why
she was missing
o The third criterion is the most difficult, but the trial judge erred in law by
refusing the Crown's request for an adjournment without having given it an
opportunity to demonstrate that this condition was met
 All that being said, was it nevertheless in the interests of justice to order an acquittal in
this case?
o No: the charges were serious, there was no prior postponement and the defence
attorney did not mention any specific adverse consequence for the accused
Hilton JA (dissenting)
The trial judge did not abuse his discretion in disposing of the case in the manner he did
In this case, the trial judge focussed in the main on what he perceived to be the Crown's
negligence in not having taken adequate steps to ensure the presence of the victim
 No one can seriously challenge that the Crown acts in accordance with the law when it
sends a subpoena by ordinary mail
o Undoubtedly, that practice may work in many instances, but it can hardly be
doubted that using the ordinary mail to reach a 16-year old girl with a
"particular" lifestyle and then not following up to insure the subpoena was
received was conduct replete with risk
As for the gravity of the charges, this is a two-way street
 The Crown did not consider that the gravity of the charges against the accused
warranted any particular consideration when it came time to compel the attendance of its
only witness to the trial
192

The gravity of the charges also weighs on the accused, who had pleaded not guilty to
them, and whose liberty was at risk once he had entered a plea of not guilty
In addition, no party, be it the Crown or the accused, is entitled to a postponement simply
because one has not been previously requested
Trial within a reasonable time (Charter s. 11(b))
Motions to stay proceedings for unreasonable delay are generally made to the trial judge, at the
outset of trial
 The judge must make a decision on unreasonable delay in light of the way the case
has proceeded from the time of charge
s. 11(b) Canadian Charter of Rights and Freedoms
Proceedings in criminal and penal matters
Any person charged with an offence has the right
...
b) to be tried within a reasonable time;
The abuse of process is not created by the delay itself, but in some cases, the circumstances
surrounding the delay
 Unreasonable delay has a different test than abuse of process
Factors to consider when determining whether there has been an unreasonably long delay
 Length of delay
 Reasons for the delay
o Conduct of the Crown
o Systemic of institutional factors, like lack of resources
 The lack of institutional facilities, however, cannot be used to render the
meaning of 11(d) meaningless
 The burden is on the Crown to show that institutional factors are at play
 Morin seems to focus less on burdens, and in some questions they
are placed on the accused
o The conduct of the accused
 An accused should not be allowed to deliberately prolong a case with
dilatory applications
 If the delay acts in favour of the accused, even if this was not planned, it
must be taken into account
 Prejudice to the accused
 A waiver of behalf of the accused
o This is dealt in a similar way as the right to counsel
 The accused does not need to assert the right
193

Any waiver must be clear and unequivocal in relation to the rights being
waived
o While the waiver can be implicit as of Askov and Morin, the accused must
understand the consequences of the waiver
R. v. Askov, [1990] 2 SCR 1199
Facts
 The accused were charged with conspiracy to commit extortion
 All counsel agreed on a date early in July 1984 for the preliminary hearing, but it could
not be completed until September
 A trial was then set for the first available date, in October 1985
 The case could not be heard during that session, and was put over for trial to almost two
years after the preliminary hearing
 When the trial finally began, appellants moved to stay the proceedings on the ground
that the trial had been unreasonably delayed
 The trial judge found that the major part of the delay following appellants' committal
stemmed from institutional problems and granted the stay
 The CA found: (1) no misconduct on the part of the Crown; (2) no indication of any
objection by the appellants to any of the adjournments; and (3) no evidence of any
actual prejudice to the appellants. It accordingly set aside the stay and directed that the
trial proceed
Issue: Should a stay be granted because of judicial delay?
Holding: Yes; appeal allowed
Cory J
The right to be tried within a reasonable time, like other specific s. 11 guarantees, is
primarily concerned with an aspect of fundamental justice guaranteed by s. 7
 The primary aim of s. 11(b) is to protect the individual's rights and to protect
fundamental justice for the accused
 A community or societal interest, however, is implicit in the section in that it
ensures, first, that law breakers are brought to trial and dealt with according to the
law and, second, that those on trial are treated fairly and justly
A quick resolution of the charges also has important practical benefits, since memories fade
with time, and witnesses may move, become ill or die
 Victims, too, have a special interest in having criminal trials take place within a
reasonable time, and all members of the community are entitled to see that the justice
system works fairly, efficiently and with reasonable dispatch
The court should consider a number of factors in determining whether the delay in
bringing the accused to trial has been unreasonable
1. The length of the delay
2. The explanation for the delay
3. Waiver
4. Prejudice to the accused
194
The longer the delay, the more difficult it should be for a court to excuse it, and very lengthy
delays may be such that they cannot be justified for any reason
 Delays attributable to the Crown will weigh in favour of the accused
 Complex cases, however, will justify delays longer than those acceptable in simple cases
 Systemic or institutional delays will also weigh against the Crown
When considering delays occasioned by inadequate institutional resources, the question of how
long a delay is too long may be resolved by comparing the questioned jurisdiction to others in
the country
 The comparison of similar and thus comparable districts must always be made with the
better districts, not the worst

In all cases it will be incumbent upon the Crown to show that the institutional delay in
question is justifiable
Certain actions of the accused, on the other hand, will justify delays
 A waiver by the accused of his rights will justify delay, but the waiver must be
informed, unequivocal and freely given to be valid
Here, the delay of almost two years following the preliminary hearing was clearly excessive and
unreasonable
 The Crown did not show that the delay did not prejudice the appellants, and nothing in
the case was so complex or inherently difficult as to justify a lengthy delay
The height of allowing stays for delays came in the aftermath of Askov
 Justice Arbour for the Ontario CA noted in R. v. Bennet (1991) that in one six-month
period following Askov 34,000 charges had been stayed, dismissed or withdrawn
 Justice Cory stated in a conference in 1991 that while he knew Askov would have an
impact, he never imagined the extent of this decision
The purpose of the right to be tried within a reasonable time includes the right to security of the
person (see Mills)
 There is also an increasing recognition to other matters like a fair trial
In Morin some of the issues that were considered were
 Delays inherent to the case
 Actions of the accused
 Actions of the Crown
 Limits on institutional resources
o There is some obligation on the Crown to commit resources to avoid an
unreasonable delay, but one has to be careful when relying in unfair comparisons
across different jurisdictions
o The rapid and increasing population could be a reason to consider why a delay is
justified under some circumstances
195



While some case presumptions of unreasonable delay being due to prejudice are virtually
irrebuttable
o Morin sees no burden on the Crown at all
o A lack of prejudice can act to justify deviations of the 10-month rule
o What the court used to justify the lack of prejudice was the Crown’s proposal to the
accused to have the trial moved so that it would take place in two months, and her
refusal was taken as an inference that she suffered no prejudice
The case also brought up the issue that delays can in some cases act in favour of the accused
o Many accused individuals act so as to avoid having a speedy trial
The majority also claims that it was able to answer the question of this appeal without
resorting to burdens of proof
R. v. Morin, [1992] 2 SCR 771
Facts
 The accused was charged with impaired driving and with operating a motor vehicle while
having a blood alcohol level which exceeded the legal limit
 She was released from custody that same day on a promise to appear
 When she appeared in Provincial Court, her counsel explicitly requested "the earliest
possible trial date
 On her scheduled trial date the accused brought a motion to stay the proceedings pursuant
to s. 24(1) of the Charter, arguing that the 14½-month delay in bringing her to trial
infringed her right to be tried within a reasonable time under s. 11(b) of the Charter
 The motion was dismissed and the accused was convicted on the "over 80" charge
 A stay was entered with respect to the impaired driving charge for unrelated reasons
 On appeal, the summary conviction appeal court also stayed the "over 80" charge on the
basis that the accused had not been tried within a reasonable time
 The CA allowed the Crown's appeal and restored the conviction.
Issue: Should a stay be granted because of judicial delay?
Holding: No; appeal dismissed
Sopinka J
The primary purpose of s. 11(b) is the protection of the individual rights of accused
persons: (1) the right to security of the person, (2) the right to liberty, and (3) the right to
a fair trial
 The right to security of the person is protected by seeking to minimize the anxiety,
concern and stigma of exposure to criminal proceedings
 The right to liberty is protected by seeking to minimize exposure to the restrictions on
liberty which result from pre-trial incarceration and restrictive bail conditions
 The right to a fair trial is protected by attempting to ensure that proceedings take place
while evidence is available and fresh.
A secondary interest of society as a whole has also been recognized
 This interest is most obvious when it parallels that of the accused: society as a whole
has an interest in seeing that citizens who are accused of crimes are treated humanely
and fairly
196

There is, as well, a societal interest that is by its very nature adverse to the interests of
the accused: there is a collective interest in ensuring that those who transgress the law
are brought to trial and dealt with according to the law
The general approach to a determination of whether the s. 11(b) right has been denied is by a
judicial determination balancing the interests which the section is designed to protect
against factors which inevitably lead to delay
 The factors to be considered are
(1) Length of the delay
(2) Waiver of time periods
(3) Reasons for the delay, including
(a) Inherent time requirements of the case
(b) Actions of the accused
(c) Actions of the Crown
(d) Limits on institutional resources
(e) Other reasons for delay
(4) Prejudice to the accused
The period to be scrutinized is the time elapsed from the date of the charge to the end of the trial
 If by agreement or conduct the accused has waived any part of this time period, the
length of the period of delay will be reduced accordingly.
An inquiry into unreasonable delay is triggered by an application under s. 24(1) of the
Charter
 While the applicant has the legal burden of establishing a Charter violation, an
evidentiary burden of putting forth evidence or argument on particular factors will shift
depending on the circumstances of each case
An inquiry into unreasonable delay should only be undertaken if the period is of sufficient
length to raise an issue as to its reasonableness
 A case will only be decided by reference to the burden of proof if the court cannot
come to a determinate conclusion on the facts presented to it
.
As well as the complexity of a case, all cases are subject to certain intake requirements and
some cases must pass through a preliminary inquiry before reaching trial
The court will also need to consider whether the actions of either the accused or the Crown have
led to delay
In considering the explanation for delay, account must be taken of the limits of institutional
resources
 Institutional delay runs from the time the parties are ready for trial and continues until
the system can accommodate the proceedings

It is appropriate for this Court to suggest a guideline of between 8 and 10 months for
institutional delay in Provincial Courts
197


A guideline with respect to institutional delay after committal for trial in the range of 6
to 8 months was suggested in R. v. Askov, and is still apposite
The application of the guideline will be influenced by the presence or absence of
prejudice; the greater the prejudice, the shorter the acceptable period of institutional
delay
Prejudice may be inferred from the length of the delay
 The longer the delay, the more likely that such an inference will be drawn
In this case the delay of 14½ months is sufficient to raise the issue of reasonableness
 In the jurisdiction in which this case arose, a period in the order of 10 months would not
be unreasonable for systemic delay given the rapidly changing local conditions
 The accused led no evidence of prejudice and little or no prejudice is inferred from the
delay as the accused appeared to be content with the pace of litigation
 In view of the strain on institutional resources and the absence of any significant
prejudice to the accused, the delay in this case was not unreasonable
Is it fair to require accused individuals to prove prejudice?
 Would this be placing an unreasonable burden on individuals who are most sensitive
Since Morin, very few s. 11(b) challenges have succeeded
 Although the case seems to re-state Askov, there are at least four important differences
o The more serious the charge the less likely it will be stayed
o The Crown no longer has the burden of proving that delay was caused by the
accused, that institutional delay was justified, that there was no prejudice, or that
the accused had waived her s. 11 (b) right
o The comparative jurisdiction test is less important
o Whether the accused has been prejudices is crucial

Formal attacks on the information or indictment
In provincial courts, charges are always contained in informations (regardless whether they are
tried as indictable offences or summary convictions)
 If the trial is by indictment, the information is treated for all purposes as an indictment
(see ss. 2 and 554(4))
 In the case of superior courts, the information which began the process is replaced by an
indictment (s. 566)
s. 554 – Criminal Code
Trial by provincial court judge with consent
(1) Subject to subsection (2), if an accused is charged in an information with an indictable
offence other than an offence that is mentioned in section 469, and the offence is not one over
which a provincial court judge has absolute jurisdiction under section 553, a provincial court
198
judge may try the accused if the accused elects to be tried by a provincial court judge.
s. 566 – Criminal Code
Indictment
(1) The trial of an accused for an indictable offence, other than a trial before a provincial court
judge, shall be on an indictment in writing setting forth the offence with which he is charged.
Preferring indictment
(2) Where an accused elects under section 536 or re-elects under section 561 to be tried by a
judge without a jury, an indictment in Form 4 may be preferred.
What counts may be included and who may prefer indictment
(3) Section 574 and subsection 576(1) apply, with such modifications as the circumstances
require, to the preferring of an indictment pursuant to subsection (2).
The allegations provided in the information or indictment must be proved beyond a reasonable
doubt
 The document must give the accused adequate notice of what the burden of the case she
has to meet
In any trial there is only one information or indictment for each accused, though the document
may contain more than one accused and more than one count
 A formal attach may be against one or all the counts
A formal objection is sometimes introduced by general language such as a motion to quash for a
formal defect, that the charge is void for uncertainty or that the charge does not name an offence
known to law
 However, it is better to be more precise and distinguish between these types of objections
The three reasons to challenge the indictment are
 Insufficiency
 Duplicity
 Improper joinder
o Two offences which arose of different circumstances should not be joined
together as one could taint the trier of fact on the other
o Normally, prior offences will not be admissible at trial unless for similar fact
evidence
In the case of summary convictions, the proceedings of a successful formal objection may
act as a new defence, since there is a time limitation of six months
199
A) Grounds
1. Insufficiency
A charge in an indictment will be insufficient if it does not comply with the requirements se in
ss. 581 and 583 of the Code
s. 581 – Criminal Code
Substance of offence
(1) Each count in an indictment shall in general apply to a single transaction and shall contain
in substance a statement that the accused or defendant committed an offence therein specified.
Form of statement
(2) The statement referred to in subsection (1) may be
(a) in popular language without technical averments or allegations of matters that are
not essential to be proved;
(b) in the words of the enactment that describes the offence or declares the matters
charged to be an indictable offence; or
(c) in words that are sufficient to give to the accused notice of the offence with which he
is charged.
Details of circumstances
(3) A count shall contain sufficient detail of the circumstances of the alleged offence to give to
the accused reasonable information with respect to the act or omission to be proved against him
and to identify the transaction referred to, but otherwise the absence or insufficiency of details
does not vitiate the count.
Indictment for treason
(4) Where an accused is charged with an offence under section 47 or sections 49 to 53, every
overt act that is to be relied on shall be stated in the indictment.
Reference to section
(5) A count may refer to any section, subsection, paragraph or subparagraph of the enactment
that creates the offence charged, and for the purpose of determining whether a count is
sufficient, consideration shall be given to any such reference.
General provisions not restricted
(6) Nothing in this Part relating to matters that do not render a count insufficient shall be
deemed to restrict or limit the application of this section.
200
s. 583 – Criminal Code
Certain omissions not grounds for objection
No count in an indictment is insufficient by reason of the absence of details where, in the
opinion of the court, the count otherwise fulfils the requirements of section 581 and, without
restricting the generality of the foregoing, no count in an indictment is insufficient by reason
only that
(a) it does not name the person injured or intended or attempted to be injured;
(b) it does not name the person who owns or has a special property or interest in
property mentioned in the count;
(c) it charges an intent to defraud without naming or describing the person whom it was
intended to defraud;
(d) it does not set out any writing that is the subject of the charge;
(e) it does not set out the words used where words that are alleged to have been used are
the subject of the charge;
(f) it does not specify the means by which the alleged offence was committed;
(g) it does not name or describe with precision any person, place or thing; or
(h) it does not, where the consent of a person, official or authority is required before
proceedings may be instituted for an offence, state that the consent has been obtained.
The same provisions apply to summary conviction proceedings (s. 795)
s.795 – Criminal Code
Application of Parts XVI, XVIII, XX and XX.1
The provisions of Parts XVI and XVIII with respect to compelling the appearance of an accused
before a justice, and the provisions of Parts XX and XX.1, in so far as they are not inconsistent
with this Part, apply, with such modifications as the circumstances require, to proceedings
under this Part.
These provision strike a compromise between allowing enough notice to accused
individuals and avoiding undue technicalities
A motion to quash based on “a defect apparent on the face” of an indictment or information must
be brought before the plea or thereafter only be leave of the court (s. 601(1))
s. 601(1) – Criminal Code
Amending defective indictment or count
(1) An objection to an indictment or to a count in an indictment for a defect apparent on the face
thereof shall be taken by motion to quash the indictment or count before the accused has
pleaded, and thereafter only by leave of the court before which the proceedings take place, and
the court before which an objection is taken under this section may, if it considers it necessary,
order the indictment or count to be amended to cure the defect.
Brodie v. R., [1936] SCR 188
201


The charge must specify the time, the place and the matter
People should have sufficient notice so that they can prepare their case
R. v. McKenzie, [1972] 2 SCR 409
Facts
 The respondent was employed to drive one of his employer’s taxis upon terms which
entitled him to retain 45 per cent of the daily receipts and required him to account for
the balance, less the cost of gasoline and oil, to his employer
 In accounting for a trip in which he picked up five passengers bound for different
addresses, he only recorded one trip for one passenger
 He was charged that he “…did commit theft of the approximate sum of $16.50 the
property of Dominic Louis Christian contrary to the form of the statute in such case
made and provided.”
 He was convicted of theft by the trial judge, but acquitted by a majority judgment of the
Court of Appeal
o That Court expressed the view that this form of charge lacked any averment of
the essential ingredients of an offence
Issue: Was the charge void for insufficiency because it lacked the essential ingredients of the
offence?
Holding: No; appeal allowed
Ritchie J
The provisions of s. 492(2) (b) are to be read disjointly and envisage two alternative methods
in which an indictment may be phrased so that a charge may be laid either “in the words
of the enactment that describe the offence” or those of the enactment which “declares the
matters charged to be an indictable offence”
The charge in the present case complied with the Criminal Code in that it was in the words of
s. 280 (now 334) which declares “theft” to be an indictable offence
 The respondent’s failure to account to his employer was “theft” within the meaning of
s. 276(1) (now s. 330(1))
If there had been any doubt as to the conduct to which the charge related, further particulars
describing the means by which the offence was alleged to have been committed could have
been sought under s. 497(1) (f)
Lecture Notes
 The Brodie requirements already appeared too onerous to the courts
 Ritchie J
o In the Code of the time there were three definitions of offences
o The case could be distinguished from Brodie
o The indictment must not necessarily state which particular definition of theft he
was facing (in terms of the section in the Code) as it was sufficiently described
in the indictment
R. v. Côté, [1978] 1 SCR 8
Facts
202

The accused was charged with failing to provide a breath sample, the words "without
reasonable excuse" being omitted from the information
 The CA quashed his conviction on the bases that information was incomplete
Issue: Was the information for insufficiency because it lacked the terms “without reasonable
excuse”?
Holding: No; appeal allowed
De Grandpré J
The Crown submits that the words "without reasonable excuse" were brought to the
attention of the accused by the specific reference to the section of the Criminal Code
creating the offence
The golden rule is for the accused to be reasonably informed of the transaction alleged
against her, thus giving her the possibility of a full defence and a fair trial
When, as in the present case, the information recites all the facts and relates them to a
definite offence identified by the relevant section of the Code, it is impossible for the
accused to be misled
 To hold otherwise would be to revert to the extreme technicality of the old procedure
R. v. Wis Development Corporation Ltd. et al, [1984] 1 SCR 485
Facts
 The accused were charged with 32 summary conviction offences relating to the breaches
of the Aeronautics Act and the regulations issued thereunder
 Following the Crown’s refusal to provide particulars, respondents, prior to plea, made
an application to quash the information containing those counts
 A Provincial Court judge allowed the motion ruling that it did not satisfy the
requirements of s. 510(3) of the Code
 He also ruled that this was not a case where particulars should be ordered
 The Court of Queen’s Bench and the Court of Appeal upheld the quashing
Issue: Was the information void because it lacked sufficient details?
Holding: Yes; appeal dismissed
Lamer J
The court agreed with the accused arguments expressed in their factum:
 The operation of a “commercial air service” within the meaning of s. 9(1) of the
Aeronautics Act could therefore relate to a multitude of activities or usages of
aircraft in Canada, for example the use of an aircraft to haul passengers or freight, the
use of an aircraft as a demonstrator by a dealer or manufacturer, or indeed even the use
of the aircraft in a photo session for the purposes of advertisement of another product
such as liquor or cigarettes
 The statute under which the information at bar has been laid casts a broad net and
the prohibition is directed at many diverse and unrelated uses of aircraft
Lecture Notes
 It is important to distinguish a voidable information or indictment (which can still be
203

remedied) and one which is void, which cannot
Here the court found that the information was ab initio vitiated for want of sufficient
details
R. v. B. (G.), [1990] 2 SCR 30
Facts
 Each of the appellants, who are young offenders, was charged with sexually assaulting
the complainant, a fellow elementary school student who was seven years old at the time
of the alleged offence
 The complainant was sworn in by the trial judge and testified that the assault took place
sometime during the wintertime when she was in grade one
 The complainant's mother testified that her daughter experienced bed-wetting and
nightmares during the late months of 1985 and the early months of 1986
 The trial judge concluded that if the event did take place, its date had not been
established
o He noted that if the offence had taken place while the complainant was in grade
one, it would have had to occur a year earlier than alleged
o He found that the date is an essential element of the offence and refused to
amend the information as requested by the Crown
o Since one of the main ingredients of the offence had not been established beyond
a reasonable doubt, the appellants must be acquitted
 The Court of Appeal found that the trial judge erred in failing to find that the date had
been established with sufficient particularity and in failing to amend the information in
light of the evidence presented at trial
o It set aside the acquittals and ordered a new trial
Issue: Was the information void because of the date had not been established with sufficient
particularity?
Holding: No; appeal dismissed
Wilson J
An information or indictment must provide an accused with enough information to enable
her to defend the charge
While time must be specified, the exact time need not be identified or proved
 In this case the information provided was adequate, having regard to the nature of the
offence charged and the age of the victim
Under s. 529(4.1) of the Criminal Code, a variance between the indictment and the evidence is not
material with respect to the time of commission of the offence
 The common law had developed a similar rule: if the time specified in the information
conflicts with the evidence and time is not an essential element of the offence or crucial to
the defence, the variance is not material and the information need not be quashed
 Further, the Crown need not prove the alleged date unless time is an essential element
of the offence, as when an accused defends the charge by providing evidence of an
alibi for the date or time period alleged
204
If time was not an essential element in the case, then a conviction could result even though
the time of the offence is not proven
 Had the trial judge addressed this question, he would have been forced to conclude that
time was not an essential element of the offence or crucial to the defence
 The Court of Appeal was therefore correct in holding that the time of the offence did not
need to be proven beyond a reasonable doubt in the circumstances of this case.
2. Duplicity
Duplicity is really another aspect of sufficiency
The common law against duplicity prohibits alternative charges in a single count
 There is nothing duplicitous about alternative counts
 A duplicitous charge is confusing to the accused and prejudices a fair defence
In the case of indictments the relevant provisions are ss. 581(1) of the Code and the apparently
contradictory s. 590(1)
s. 581(1) – Criminal Code
Substance of offence
Each count in an indictment shall in general apply to a single transaction and shall contain in
substance a statement that the accused or defendant committed an offence therein specified.
s. 590(1) – Criminal Code
Offences may be charged in the alternative
A count is not objectionable by reason only that
(a) it charges in the alternative several different matters, acts or omissions that are stated
in the alternative in an enactment that describes as an indictable offence the matters, acts
or omissions charged in the count; or
(b) it is double or multifarious.
In the case of summary convictions the relevant provision in s. 789(1) (although s. 581(1) and
(6) still apply)
s. 789(1) – Criminal Code
Formalities of information
(1) In proceedings to which this Part applies, an information
(a) shall be in writing and under oath; and
(b) may charge more than one offence or relate to more than one matter of complaint,
but where more than one offence is charged or the information relates to more than one
matter of complaint, each offence or matter of complaint, as the case may be, shall be
set out in a separate count.
205
s. 581 – Criminal Code
Substance of offence
(1) Each count in an indictment shall in general apply to a single transaction and shall contain
in substance a statement that the accused or defendant committed an offence therein specified.
...
General provisions not restricted
(6) Nothing in this Part relating to matters that do not render a count insufficient shall be
deemed to restrict or limit the application of this section.
A motion to quash with respect to a defect that is apparent on the face of the charge must be
brought before the plea, and thereafter with leave of teh Court
R. v. Sault Ste. Marie, [1978] 2 SCR 1299
Facts
 The accused City was charged for polluting under s. 32(1) of The Ontario Water
Resources Commission Act
 In dismissing the charge against the City the trial judge found that the City had nothing
to do with the actual operations, that the company was an independent contractor and
that its employees were not employees of the City
 On appeal by trial de novo the judge found that the offence was one of strict liability and
he convicted
 The Divisional Court set aside the charge as duplicitous, but the CA rejected this claim
Issue: Was the information void for duplicity?
Holding: No; appeal dismissed
Dickson J
The primary test for duplicity should be the practical one based on the only valid
justification for the rule against duplicity, the requirement that the accused know the case
he has to meet and be not prejudiced in the preparation of his defence by ambiguity in the
charge
 In this case there was nothing ambiguous or uncertain in the charge, the City knew the
case it had to meet
 Section 32(1) of The Ontario Water Resources Commission Act is concerned with only
one matter, pollution and that is the gist of the charge and the evil against which the
offence is aimed
3. Improper joinders of Counts
One needs to be careful that two unrelated offences are not tried together, as the merits of one
charge may influence the trier of fact on the other
206

Nonetheless, it make sense to join charges to save judicial resources under some
circumstances
S. 590(3) gives the court a wide jurisdiction to order that the count in an indictment be divided
into two or more counts
s. 590(3) – Criminal Code
Order
The court may, where it is satisfied that the ends of justice require it, order that a count be
amended or divided into two or more counts, and thereupon a formal commencement may be
inserted before each of the counts into which it is divided.
The same wide discretion is given to the trial judge to grant a motion before or during trial to
separate the trial of an accused upon one of more counts (s. (591(3))
s. 591(3) – Criminal Code
Severance of accused and counts
The court may, where it is satisfied that the interests of justice so require, order
(a) that the accused or defendant be tried separately on one or more of the counts; and
(b) where there is more than one accused or defendant, that one or more of them be tried
separately on one or more of the counts.
R. v. Racco (No. 1) (1975), 29 CRNS 303 (Ont. Co. Ct)
Facts
 The accused was charged in Count 1 of the indictment for possession of counterfeit
money without lawful justification or excuse
o He had in his possession seven counterfeit United States $20 federal reserve
notes
 He was also charged in Count 2 that at the same time and the same place, without
lawful excuse, he had in his possession an explosive substance, one ground burst
projectile simulator, for other than a lawful purpose
 The accused asked for a severance
Issue: Should the counts of possession of counterfeit money and possession of an explosive
substance be severed?
Holding: No
Graburn J
The accused argued that notwithstanding any direction that this Court would give in
relation to considering the evidence pertinent to each count separately, a jury would be
unconsciously influenced by one count on considering the other
 Particularly, since the issue in relation to both counts is one of credibility, as the accused
argues that both the money and the explosive were planted by the police
207
Where the issue is substantially credibility and where there is a close nexus in time and
place in relation to the counts as exists in this case, it is in the interests of justice that both
of these counts be tried together
 The interests of justice require that this issue of a plant by the police be resolved by one
tribunal and not by two tribunals
To permit a severance here could lead to the following undesirable resul:.a jury on a trial of
Count 1 could have a reasonable doubt whether the counterfeit money has been planted and
acquit the accused., while a different jury might come to a different conclusion in relation to the
second count
 In the light of the identical nature of the defence to both offenses, such a result is not in
the interests of the administration of justice and is not a criterion on which a court
should grant a severance
As to the difficulty of the jury being influenced on one count by the evidence relating to
the other, the judge felt it was possible to adequately instruct the jury so as to negate this
danger
B) Saving devices
1. Particulars
In response to the attack for insufficiency, the court may demand that the Crown furnish
particular as per s. 587 of the Code
 Alternatively, particulars might come from informal interactions between the parties
 If the informal process is considered satisfactory by the defence, it is imperative that
these particulars are place on the court record for the protection of the accused
o This is because the Crown is bound by particulars in proving its charge
s. 587 – Criminal Code
What may be ordered
(1) A court may, where it is satisfied that it is necessary for a fair trial, order the prosecutor
to furnish particulars and, without restricting the generality of the foregoing, may order the
prosecutor to furnish particulars
(a) of what is relied on in support of a charge of perjury, the making of a false oath or a
false statement, fabricating evidence or counselling the commission of any of those
offences;
(b) of any false pretence or fraud that is alleged;
(c) of any alleged attempt or conspiracy by fraudulent means;
(d) setting out the passages in a book, pamphlet, newspaper or other printing or writing
that are relied on in support of a charge of selling or exhibiting an obscene book,
pamphlet, newspaper, printing or writing;
(e) further describing any writing or words that are the subject of a charge;
(f) further describing the means by which an offence is alleged to have been committed;
or
208
(g) further describing a person, place or thing referred to in an indictment.
Regard to evidence
(2) For the purpose of determining whether or not a particular is required, the court may give
consideration to any evidence that has been taken.
Particular
(3) Where a particular is delivered pursuant to this section,
(a) a copy shall be given without charge to the accused or his counsel;
(b) the particular shall be entered in the record; and
(c) the trial shall proceed in all respects as if the indictment had been amended to
conform with the particular.
Thatcher v. R. (1984), 42 CR (3d) 259
Facts
 The accused was charged with first degree murder
 At the preliminary inquiry, evidence was adduced showing that the accused was either
guilty of murder or of aiding and abetting a murder
 The accused applied pursuant to s. 516(1)(f) of the Code for an order for particulars
describing the means by which he was alleged to have committed the offence as charged
Issue: Should an order for particulars be granted to describe the means by which the accused
committed a murder?
Holding: No; appeal allowed for other reasons
Maher J
Section 516 empowers the court to order the Crown to furnish particulars where necessary for a
fair trial and where necessary to enable the accused to make a proper defence to the charge
against him
In this case particulars were not needed for those purposes but were sought to fetter the
prosecution's conduct of the case
If there was evidence upon which a properly instructed jury could find the accused guilty
of murder or that he abetted a murder, it had to be left to the jury to make either of such
findings
 The right to do so could not be restricted by an order for particulars
2. Amendment
A trial judge has a wide discretion to amend an indictment or information (ss. 601 and 795)
 See also ss. 590(2) and (3)
s. 601 – Criminal Code
209
Amending defective indictment or count
(1) An objection to an indictment or to a count in an indictment for a defect apparent on the face
thereof shall be taken by motion to quash the indictment or count before the accused has
pleaded, and thereafter only by leave of the court before which the proceedings take place, and
the court before which an objection is taken under this section may, if it considers it necessary,
order the indictment or count to be amended to cure the defect.
Amendment where variance
(2) Subject to this section, a court may, on the trial of an indictment, amend the indictment or a
count therein or a particular that is furnished under section 587, to make the indictment, count
or particular conform to the evidence, where there is a variance between the evidence and
(a) a count in the indictment as preferred; or
(b) a count in the indictment
(i) as amended, or
(ii) as it would have been if it had been amended in conformity with any
particular that has been furnished pursuant to section 587.
Amending indictment
(3) Subject to this section, a court shall, at any stage of the proceedings, amend the indictment
or a count therein as may be necessary where it appears
(a) that the indictment has been preferred under a particular Act of Parliament instead of
another Act of Parliament;
(b) that the indictment or a count thereof
(i) fails to state or states defectively anything that is requisite to constitute the
offence,
(ii) does not negative an exception that should be negatived,
(iii) is in any way defective in substance,
and the matters to be alleged in the proposed amendment are disclosed by the evidence taken on
the preliminary inquiry or on the trial; or
(c) that the indictment or a count thereof is in any way defective in form.
Matters to be considered by the court
(4) The court shall, in considering whether or not an amendment should be made to the
indictment or a count in it, consider
(a) the matters disclosed by the evidence taken on the preliminary inquiry;
(b) the evidence taken on the trial, if any;
(c) the circumstances of the case;
(d) whether the accused has been misled or prejudiced in his defence by any variance,
error or omission mentioned in subsection (2) or (3); and
(e) whether, having regard to the merits of the case, the proposed amendment can be
made without injustice being done.
210
Variance not material
(4.1) A variance between the indictment or a count therein and the evidence taken is not
material with respect to
(a) the time when the offence is alleged to have been committed, if it is proved that the
indictment was preferred within the prescribed period of limitation, if any; or
(b) the place where the subject-matter of the proceedings is alleged to have arisen, if it
is proved that it arose within the territorial jurisdiction of the court.
Adjournment if accused prejudiced
(5) Where, in the opinion of the court, the accused has been misled or prejudiced in his defence
by a variance, error or omission in an indictment or a count therein, the court may, if it is of the
opinion that the misleading or prejudice may be removed by an adjournment, adjourn the
proceedings to a specified day or sittings of the court and may make such an order with respect
to the payment of costs resulting from the necessity for amendment as it considers desirable.
Question of law
(6) The question whether an order to amend an indictment or a count thereof should be granted
or refused is a question of law.
Endorsing indictment
(7) An order to amend an indictment or a count therein shall be endorsed on the indictment as
part of the record and the proceedings shall continue as if the indictment or count had been
originally preferred as amended.
Mistakes not material
(8) A mistake in the heading of an indictment shall be corrected as soon as it is discovered but,
whether corrected or not, is not material.
Limitation
(9) The authority of a court to amend indictments does not authorize the court to add to the
overt acts stated in an indictment for high treason or treason or for an offence against any
provision in sections 49, 50, 51 and 53.
Definition of “court”
(10) In this section, "court" means a court, judge, justice or provincial court judge acting in
summary conviction proceedings or in proceedings on indictment.
Application
(11) This section applies to all proceedings, including preliminary inquiries, with such
211
modifications as the circumstances require.
R. v. Moore, [1988] 1 SCR 1097
Facts
 Accused was charged with eight counts of theft and possession of stolen property
 The information on two counts lacked the averment of an essential element of the offence
 The accused entered pleas of not guilty to the charges against him and elected to be tried
by a provincial court judge without a jury
 The trial judge and counsel agreed that it was not possible to strike the pleas and that,
since two of the counts simply failed to allege an offence rather than alleging an offence
improperly, it was not possible to amend them at all
 A new information containing the necessary words omitted from the earlier information
was subsequently sworn
 Respondent's pleas of autrefois acquit were refused by the second trial judge who
convicted on one of the counts
 The Court of Appeal allowed the respondent's appeal from conviction
Issue: Does quashing an information, after plea, for failure to allege a material averment
constitutes a verdict of acquittal for the purpose of pleading autrefois acquit to a new
information?
Holding: Yes; appeal dismissed
Lamer J
Section 529 (now 601) grants very wide powers to the trial judge, subject to certain limits,
to cure any defect in a charge by amending it
The judge can only quash if the required amendment cannot be made without injustice
being done and it is a reversible error of law if a judge does so without coming to that
conclusion
 If an amendment, as a matter of law, cannot be made without causing irreparable
prejudice, the quashing of the charge at the trial is tantamount to an acquittal
 Relaying an amended charge before another judge would be no less prejudicial to the
accused than the previous judge's amending the first one
The judge should not have quashed here for an amendment would not have prejudiced the
accused
 The Crown, however, cannot lay an amended charge once the accused has been acquitted,
albeit by error
The trial judge's decision is open to appeal
 The Court of Appeal, assuming error is found, will direct the trial judge to amend and
hear the case, or will amend the charge itself and return the matter for trial on the
amended charge
R. v. Tremblay, [1993] 2 SCR 932
212
Facts
 The appellants were charged with keeping a bawdy-house for the purpose of the practice
of indecent acts
 Nude dancers would perform in individual cubicles for their clients and would assume a
variety of suggestive positions
 At trial, the Crown's motion to amend the charge by deleting the words "the practice of
indecency", and its subsequent motion to include the words "practice of prostitution"
were denied because they would cause serious prejudice to the accused
 The motions were made late in the trial when virtually all the evidence had been called
 The Court of Appeal materially amended the charge and entered a conviction on the basis
of the amended charge
Issue: Should the Crown ne allowed to amend the charge substantially?
Holding: No; appeal allowed
Cory J
By virtue of s. 601 of the Code the courts now possess reasonably wide powers of
amendment
 Yet, it remains an important principle of criminal law that persons accused of a crime
must know the charge brought against them in order to present a full answer and defence
 A court cannot amend an information or indictment where to do so would cause
irreparable prejudice
 Moreover, a court cannot amend an information unless the evidence tendered is
capable of supporting such a charge
In the present case it is not necessary to consider whether evidence of prostitution was disclosed
during the trial
 The prejudice that granting the amendment would cause the accused is determinative of
the issue
 When the motion for the amendment was brought it was obvious that the appellants had
prepared their defence on the basis that the acts performed were not indecent
o In light of the wording of the original charge the appellants quite properly
prepared their entire defence on this issue
3. On appeal
Motions to quash indictments or information for defects on they face must be brought before an
accused has pleaded and thereafter by leave of the court (s. 601(1)
Appeal courts are very reluctant to hear such objections for the first time on appeal
 This is felt to be a matter of the trial judge and there is a fear that such technical
objections were deliberately left out for appeal in case there was an undesirable
outcome
The Trial Process: Jury Trials
213
R. v. Bryant (1984), 48 OR (2d) 732
Facts
 The accused was charged, inter alia, with forcible confinement, indecent assault and rape
and elected trial by judge and jury
 He was committed for trial, was released on bail, and failed to appear on the day fixed for
trial
 He was subsequently arrested and tried by a judge without a jury, pursuant to s. 526.1 of
the Criminal Code, which provides that a person who has elected trial by jury and who
fails to appear at his trial shall be tried by judge alone unless he can establish a legitimate
excuse for his failure to appear
 Here, the accused offered no excuse but applied for an order directing that he be tried by
a jury, relying on s. 11(f) of the Charter
 That application was dismissed
Issue: Should the accused be entitled to a trial by jury after having failed to appear at his trial?
Holding: Yes; appeal allowed
Blair JA
Section 526.1(1) on its face infringes the accused's right to trial by jury under s. 11(f) of the
Charter
The right to trial by jury is a right given to the accused which prevails unless he or she
voluntarily chooses not to utilize it by electing another mode of trial
 Any waiver of a jury trial must be voluntary and made with full understanding
 Section 526.1, which takes away an accused's right to trial by a jury where he does not
provide a legitimate excuse for failing to appear, by deeming him to have elected trial
before a judge alone, cannot be regarded as a waiver
In carrying out a s. 1 analysis, the court found that the Crown had not established that the denial
of a jury trial to absconding accused was necessary for the attainment of the purpose of s. 526.1,
which was to overcome the abuse of the judicial system and other administrative problems
arising from the Bail Reform Act
 Other sanctions were available to the Crown to punish absconding accused, and there had
been no evidence that the proper operation of the Bail Reform Act and the attainment of
its objectives was dependant upon s. 526.1
The section could not therefore be justified as a reasonable limit on the right to trial by jury
under s. 1 of the Charter, and was of no force or effect
s. 11(f) – Canadian Charter of Rights and Freedoms
Arrest or detention
Any person charged with an offence has the right
f) except in the case of an offence under military law tried before a military tribunal, to
214
the benefit of trial by jury where the maximum punishment for the offence is
imprisonment for five years or a more severe punishment;
Number and qualifications of jurors
The number is set to twelve (s. 631.(5)) for all provinces
The Code delegate to the provinces through s. 626(1) the power to determine the appropriate
qualifications for somebody to setve as a jury, except that s. 626(2) expresely precluded sex
ddiscrimination
s. 631(5) – Criminal Code
Drawing additional cards if necessary
If the number of persons who answer under subsection (3) or (3.1) is not sufficient to provide a
full jury and the number of alternate jurors ordered by the judge, the clerk of the court shall
proceed in accordance with subsections (3), (3.1) and (4) until twelve jurors and any
alternate jurors are sworn.
s. 626 – Criminal Code
Qualification of jurors
(1) A person who is qualified as a juror according to, and summoned as a juror in accordance
with, the laws of a province is qualified to serve as a juror in criminal proceedings in that
province.
No disqualification based on sex
Notwithstanding any law of a province referred to in subsection (1), no person may be
disqualified, exempted or excused from serving as a juror in criminal proceedings on the
grounds of his or her sex.
(2)
In R. v. Church of Sientology (1997) it was held that the exclusion of non-citizens from jury trials
did not violate ss. 7, 11(d), 11(f) or 15 of the Charter
Challenge to array
s. 629– Criminal Code
Challenging the jury panel
(1) The accused or the prosecutor may challenge the jury panel only on the ground of partiality,
fraud or wilful misconduct on the part of the sheriff or other officer by whom the panel was
returned.
215
In writing
(2) A challenge under subsection (1) shall be in writing and shall state that the person who
returned the panel was partial or fraudulent or that he wilfully misconducted himself, as the case
may be.
Form
(3) A challenge under this section may be in Form 40.
s. 630 – Criminal Code
Trying ground of challenge
Where a challenge is made under section 629, the judge shall determine whether the alleged
ground of challenge is true or not, and where he is satisfied that the alleged ground of challenge
is true, he shall direct a new panel to be returned.
Two essential themes in jury selection is ensuring impartiality and ensuring representativeness
(see R. v. Born With a Tooth)
 A jury can be a barrier to the application of oppressive laws, as long as it is
representativeness
In Canada there has been a strong rejection of the US system of jury selection and racial
representation, challenging the idea that jurors cannot be impartial based on their belonging to
individual groups
 Instead of dealing with whether a jury is representative or not, in Canada we tend to be
based on the assumption that if a jury is properly instructed, it will be impartial
R. v. Born With a Tooth (1993), 22 CR (4th) 232 (Alta. Q.B)
Facts
 The accused native Indian was charged with several criminal offences and elected trial
by jury
 The sheriff issued jury summonses to 252 people, 200 of whom were selected "at
random" from the municipal population
 The remainder were native Indians residing on reservations within the judicial district
o These were selected deliberately by the sheriff from utility customer lists and
other sources in order to ensure a number of individuals of native origin were on
the jury panel
 The Crown filed a written challenge to the array on the grounds of partiality and wilful
misconduct.
Issue: Should jury empanelling take into consideration racial characteristics of the jurors?
Holding: No; challenge upheld
O’Leary
The two fundamental elements of the criminal jury system are impartiality and
216
representativeness
Representativeness is guaranteed by ensuring that as far as practicable the population from
which jury panels are selected is representative of the whole community and by selecting jury
panels from that population on a random basis
 Artificially skewing the composition of jury panels to accommodate the demands of
any of the distinct segments of Canadian society would compromise the integrity of
the jury system
The effectiveness of the jury system is based on its widespread acceptance by the
community as a fair and just method of deciding issues of criminal responsibility, and
manipulation of the composition of juries would erode that trust
There was no justification for the assumption that some otherwise qualified members of
Canadian society were incapable of judging the conduct of other members of the same
community in a fair and impartial manner
Lecture Notes
 The court held that this kind of affirmative action assumes that Canadians are unable to
handle the trials of other community members in a affair and impartial manner and that
race issues cannot be appreciated by ordinary jurors
o This assumption is unjustified
o This is a small step away from the calculated inclusion of one group as jurors
and the exclusion of others
R. v. F.A. (1984)
 The person wanted the trial moved to another jurisdiction where he felt he would find a
jury panel that would be more understanding to his racial circumstances
 The complainant did not want this because she could face retribution or criticism from
community members from having brought the case forth in the first place
 The court found that counsel could address cultural idiosyncrasies by alluding to expert
evidence
R. v. Born With a Tooth (1993), 22 CR (4th) 232 (Alta. Q.B)
Facts
 The accused native Indian was charged with several criminal offences and elected trial
by jury
 The sheriff issued jury summonses to 252 people, 200 of whom were selected "at
random" from the municipal population
 The remainder were native Indians residing on reservations within the judicial district
o These were selected deliberately by the sheriff from utility customer lists and
other sources in order to ensure a number of individuals of native origin were on
the jury panel
 The Crown filed a written challenge to the array on the grounds of partiality and wilful
misconduct.
Issue: Should jury empanelling take into consideration racial characteristics of the jurors?
217
Holding: No; challenge upheld
O’Leary
The two fundamental elements of the criminal jury system are impartiality and
representativeness
Representativeness is guaranteed by ensuring that as far as practicable the population from
which jury panels are selected is representative of the whole community and by selecting jury
panels from that population on a random basis
 Artificially skewing the composition of jury panels to accommodate the demands of
any of the distinct segments of Canadian society would compromise the integrity of
the jury system
The effectiveness of the jury system is based on its widespread acceptance by the
community as a fair and just method of deciding issues of criminal responsibility, and
manipulation of the composition of juries would erode that trust
There was no justification for the assumption that some otherwise qualified members of
Canadian society were incapable of judging the conduct of other members of the same
community in a fair and impartial manner
Lecture Notes
 The court held that this kind of affirmative action assumes that Canadians are unable to
handle the trials of other community members in a affair and impartial manner and that
race issues cannot be appreciated by ordinary jurors
o This assumption is unjustified
o This is a small step away from the calculated inclusion of one group as jurors
and the exclusion of others
Challenge to poll
Peremptory challenges are challenges were the parties do not need to provide reasons for
the challenge of a juror
 The number of peremptory challenges depend on the offence
 The risk is that they serve to exclude people with certain demographics from the case and
thus deal with the representativeness of the jury
s. 632 – Criminal Code
Excusing jurors
The judge may, at any time before the commencement of a trial, order that any juror be excused
from jury service, whether or not the juror has been called pursuant to subsection 631(3) or
(3.1) or any challenge has been made in relation to the juror, for reasons of
(a) personal interest in the matter to be tried;
(b) relationship with the judge presiding over the jury selection process, the judge before
whom the accused is to be tried, the prosecutor, the accused, the counsel for the accused
218
or a prospective witness; or
(c) personal hardship or any other reasonable cause that, in the opinion of the judge,
warrants that the juror be excused.
s. 633 – Criminal Code
Stand by
The judge may direct a juror who has been called pursuant to subsection 631(3) or (3.1) to stand
by for reasons of personal hardship or any other reasonable cause.
s. 634 – Criminal Code
Peremptory challenges
A juror may be challenged peremptorily whether or not the juror has been challenged for cause
pursuant to section 638.
Maximum number
(2) Subject to subsections (2.1) to (4), the prosecutor and the accused are each entitled to
(a) twenty peremptory challenges, where the accused is charged with high treason or
first degree murder;
(b) twelve peremptory challenges, where the accused is charged with an offence, other
than an offence mentioned in paragraph (a), for which the accused may be sentenced to
imprisonment for a term exceeding five years; or
(c) four peremptory challenges, where the accused is charged with an offence that is not
referred to in paragraph (a) or (b).
If alternate jurors
(2.1) If the judge makes an order for alternate jurors, the total number of peremptory challenges
that the prosecutor and the accused are each entitled to is increased by one for each alternate
juror.
Supplemental peremptory challenges
(2.2) For the purposes of replacing jurors under subsection 644(1.1), the prosecutor and the
accused are each entitled to one peremptory challenge for each juror to be replaced.
Where there are multiple counts
(3) Where two or more counts in an indictment are to be tried together, the prosecutor and the
accused are each entitled only to the number of peremptory challenges provided in respect of
the count for which the greatest number of peremptory challenges is available.
Where there are joint trials
219
(4) Where two or more accused are to be tried together,
(a) each accused is entitled to the number of peremptory challenges to which the
accused would be entitled if tried alone; and
(b) the prosecutor is entitled to the total number of peremptory challenges available to
all the accused.
s. 635 – Criminal Code
Order of challenges
(1) The accused shall be called on before the prosecutor is called on to declare whether the
accused challenges the first juror, for cause or peremptorily, and thereafter the prosecutor and
the accused shall be called on alternately, in respect of each of the remaining jurors, to first
make such a declaration.
Where there are joint trials
(2) Subsection (1) applies where two or more accused are to be tried together, but all of the
accused shall exercise the challenges of the defence in turn, in the order in which their names
appear in the indictment or in any other order agreed on by them,
(a) in respect of the first juror, before the prosecutor; and
(b) in respect of each of the remaining jurors, either before or after the prosecutor, in
accordance with subsection (1).
Challenges for cause
 Prosecutors and defence do not have information about individuals that make part of the
jury panel other than the name or occupation and they may have been asked by the judge
if there are any obvious sources of conflict
 This makes it difficult to raise a claim of impartiality
 The most common challenge for cause is the lack of indifference between the Queen and
the accused
s. 638 – Criminal Code
Challenge for cause
(1) A prosecutor or an accused is entitled to any number of challenges on the ground that
(a) the name of a juror does not appear on the panel, but no misnomer or misdescription
is a ground of challenge where it appears to the court that the description given on the
panel sufficiently designates the person referred to;
(b) a juror is not indifferent between the Queen and the accused;
(c) a juror has been convicted of an offence for which he was sentenced to death or to a
term of imprisonment exceeding twelve months;
(d) a juror is an alien;
(e) a juror, even with the aid of technical, personal, interpretative or other support
services provided to the juror under section 627, is physically unable to perform
properly the duties of a juror; or
(f) a juror does not speak the official language of Canada that is the language of the
220
accused or the official language of Canada in which the accused can best give testimony
or both official languages of Canada, where the accused is required by reason of an
order under section 530 to be tried before a judge and jury who speak the official
language of Canada that is the language of the accused or the official language of
Canada in which the accused can best give testimony or who speak both official
languages of Canada, as the case may be.
The process of challenges
 First, potential jurors are called based on a lottery system
 Then they are asked questions by both counsel, though judges place limits on allowing
fishing expedition to find the perfect jurors
 When a challenge is raised, the trial judge first looks at whether the challenge has basis,
and if it does, then two trier of facts who have been already selected will decide whether
the juror can be impartial
 In practice, challenges are difficult to bring forth; they are most common when:
o There has been so much pre-trial publicity that there is reason to believe that
jurors to be biased
o When there is widespread racism in the community
R. v. Crosby (1979), 49 CCC (2d) 255
Osler J
In the absence of any notorious episode in a community indicating that prejudice is
rampant, an application to challenge for cause simply on the ground that man is
prejudiced and that black and white may frequently be prejudiced against each other
should not be allowed
 Otherwise, such challenges might become everyday occurrences
R. v. Sherratt, [1991] 1 SCR 509
L’Heureux-Dubé J
The jury must perform its duties impartially and represent the larger community as far as
is possible and appropriate in the circumstances
The fundamental right to a fair and proper trial is denied where the accused is not allowed to
challenge any number of jurors for cause when the grounds of challenge are properly specified
 Counsel is entitled to determine whether any potential juror is, by reason of the pretrial
publicity and the notoriety of the appellant, sufficiently impartial
 The trial judge cannot, in the exercise of his discretion in the area of admitting grounds of
challenge for cause and settling the questions, effectively curtail the statutory right to
challenge for cause
That trial judges have a wide discretion in these matters and that jurors will usually
behave in accordance with their oaths cannot supercede the right of every accused person
to a fair trial, which necessarily includes the empanelling of an impartial jury.
221
An accused does not have the right to a favourable jury and the selection procedure cannot
be used to thwart the representativeness that is essential to the proper functioning of a jury
Peremptory challenges, however, are justified on a number of grounds even though they, along
with the Crown's right to stand aside, can be used to alter somewhat the degree to which the jury
represents the community
Challenges for cause are properly used to rid the jury of prospective members who are not
indifferent or who otherwise fall within s. 567 of the Criminal Code, but they stray into
illegitimacy if used merely, without more, to over-or under-represent a certain class in society or
as a "fishing expedition" in order to obtain personal information about the juror
Information obtained on an ultimately unsuccessful challenge for cause may, however, lead the
challenger to exercise the right to challenge peremptorily or to stand aside the particular juror.
The issue raised in a challenge for cause is tried by a "mini-jury" of two jurors or two
prospective jurors if no jurors have been sworn
 Section 567 of the Criminal Code places little, if any, burden on the challenger

On the other hand, a reasonable degree of control must be retained by the trial judge
and, thus, some burden placed upon the challenger to ensure that the selection of the jury
occurs in a manner that is in accordance with the principles here articulated and also to
ensure that sufficient information is imparted to the trial
 Thus, while there must be an "air of reality" to the application, it need not be an
"extreme" case
A valuable distinction might be drawn in pre-trial publicity cases between mere publication
of the facts of a case and situations where the media misrepresents the evidence, dredges up and
widely publicizes discreditable incidents from an accused's past or engages in speculation as to
the accused's guilt or innocence
 It may well be that the pre-trial publicity or other ground of alleged partiality will, in
itself, provide sufficient reasons for a challenge for cause
 The threshold question is not whether the ground of alleged partiality will create such
partiality in a juror, but rather whether it could create that partiality which would
prevent a juror from being indifferent as to the result
 In the end, there must exist a realistic potential for the existence of partiality, on a
ground sufficiently articulated in the application, before the challenger should be
allowed to proceed
Lecture Notes
 The jury system is a way in which the public increases it knowledge of criminal justice
o Means by which the public legitimacy of the justice system is increased
R. v. Williams, [1998] 1 SCR 1128
McLachlin J
The prosecution and the defence are entitled to challenge potential jurors for cause on the ground
of partiality
222
Candidates for jury duty are presumed to be indifferent or impartial and this presumption
must be displaced before they can be challenged and questioned
 Usually the party seeking the challenge calls evidence substantiating the basis of the
concern
 Alternatively, where the basis of the concern is widely known and accepted, the law of
evidence may permit a judge to take judicial notice of it
The judge has a wide discretion in controlling the challenge process and should permit
challenges if there is a realistic possibility that the jury pool may contain people whose
racial prejudice might incline them to favour the Crown rather than the accused in deciding
the matters that fall to them in the course of the trial.
Judicial directions to act impartially cannot always be assumed to be effective in countering
racial prejudice
 Where doubts are raised, the better policy is to err on the side of caution and permit
prejudice to be examined
 The expectation that jurors usually behave in accordance with their oaths does not
obviate the need to permit challenges for cause where it is established that the community
suffers from widespread prejudice against people of the accused’s race sufficient to create
a realistic potential for partiality
The potential for partiality is irrefutable where the prejudice can be linked to specific aspects of
the trial, like a widespread belief that people of the accused’s race are more likely to commit the
crime charged.
The trial judge has the discretion to determine whether widespread racial prejudice in the
community, absent specific “links” to the trial, is sufficient to give an “air of reality” to the
challenge in the particular circumstances of each case
Section 638(2) of the Criminal Code requires two inquiries and entails two different
decisions
1. The first stage is the inquiry before the judge to determine whether challenges for cause
should be permitted
o The test at this stage is whether there is a realistic potential or possibility for
partiality
2. If the judge permits challenges for cause, a second inquiry occurs on the challenge itself
o At this stage, the question to be determined by the triers is whether the candidate
in question will be able to act impartially
Section s. 638(1)(b) is intended to prevent persons who may not be able to act impartially
from sitting as jurors
o This object cannot be achieved if the evidentiary threshold for challenges for cause is set
too high
The appropriate evidentiary standard on applications to challenge for cause based on
223
racial prejudice is a “realistic potential for partiality” (the rule in R. v. Sherratt)
o Prejudice less than widespread might in some circumstances meet this test.
A judge’s discretion to allow challenge for cause must be exercised in accordance with the
Canadian Charter
o Section s. 638(1)(b) should be read in light of the fundamental rights to a fair trial by an
impartial jury and to equality before and under the law
o The rule in Sherratt suffices to maintain these rights without adopting the United States
model or a variant on it
o It protects the accused’s right to a fair trial by an impartial jury and the privacy
interests of prospective jurors while avoiding lengthening trials or increasing their
cost
Lecture Notes
 The question of how challenges for cause can be made, was clarified in this case
 First, there is an inquiry before a judge to see if challenges are allowed at all
 Two statements were made by the Court with regards to when challenges of cause will be
allowed
o First, when there is a realistic potential of partiality in the jury pool due to
racial bias
 This test can be passed through judicial notice
o Later on the SCC (in Fine) decided that these threshold test is not met just
because the case deals with sexual assault
The Trial Process – Further Considerations
The adversarial system
 The best way to discover the historic truth of a case and get its full picture, is allowing
both parties to battle out
 This is not so much about factual and scientific truth
Jerome Frank, Courts on Trial: Myth and Reality in American Justice
 We shouldn’t blame the lawyers for misconstruing the truth
 People should have more appropriate representation and they should have better access to
that representations
R. v. Murray (2000), 144 CCC (3d) 289
Facts
 The accused lawyer was charged with attempting to obstruct justice by concealing
videotaped evidence of crimes committed by client charged with murder
 Accused was instructed by client to retain tapes but not reveal their contents, and claimed
his intention was to use them at trial to discredit witness for prosecution
Issue: Was the accused guilty of obstructions of justice?
Holding: Yes; appeal dismissed
Gravely J
224
Applying the "tendency test," the actus reus of the offence of attempting to obstruct justice is the
doing of an act which has a tendency to pervert or obstruct the course of justice.
 Wilfully" doing the act for the purpose of obstructing the course of justice constitutes the
mens rea.
On the face of the evidence, the accused's act in concealing the critical tapes had the tendency to
obstruct the course of justice at several stages of the proceedings
While the accused's act in concealing the tapes was prima facie caught by the tendency test,
he could not be said to have attempted to obstruct justice if he had legal justification for his
conduct.
While the accused's discussions with his client about the tapes were covered by solicitorclient privilege, the physical objects themselves were not
 Solicitor-client privilege protects communications between solicitor and client
o These tapes were not "communications," but dramatic evidence of crime which
pre-existed the solicitor-client relationship
Nor was concealing the tapes permissible on the basis that they had some exculpatory value,
since they were overwhelmingly inculpatory
 Not only does solicitor-client privilege not protect physical evidence, but there is also a
suggested obligation on defence counsel to turn over incriminating physical evidence to
the prosecution
By putting the tapes beyond the reach of the police and the Crown, the accused clearly intended
to impede the prosecution
Despite the inferences that could be drawn against the accused's credibility based on his actions,
a defence strategy to use the tapes at trial was reasonably feasible
 That tended to support the accused's evidence that he did not intend to permanently
suppress them
The conduct in which he engaged was not specifically criminalized by s. 139(2)
 The only official guide on this issue is contained in the Law Society's Rules of
Professional Conduct, which states that a lawyer must not knowingly attempt to influence
the course of justice by suppressing evidence that "ought to be disclosed," but provides
no guidance as to the nature of such evidence
Assuming that the accused did intend to use the tapes in the defence, he may well have believed
that he had no legal duty to disclose the tapes until resolution discussions or trial.
 Where credibility is a significant issue, the accused must be acquitted if his evidence is
believed; or if his evidence is not believed, but a reasonable doubt remains as to his guilt
after considering his evidence in the context of the evidence as a whole
225
Res Judicata
Two principles
 A person should not be tried for the same events
 There should not be multiple punishments
Res judicata also serves to
 Save judicial resources
 Maintain the legitimacy of the judicial system
 Spare the accused from multiple trials and from being found guilty for one charge based
on the outcome of others
s. 607 – Criminal Code
Special pleas
(1) An accused may plead the special pleas of
(a) autrefois acquit;
(b) autrefois convict; and
(c) pardon.
In case of libel
(
2) An accused who is charged with defamatory libel may plead in accordance with sections 611
and 612.
Disposal
(3) The pleas of autrefois acquit, autrefois convict and pardon shall be disposed of by the judge
without a jury before the accused is called on to plead further.
Pleading over
(4) When the pleas referred to in subsection (3) are disposed of against the accused, he may
plead guilty or not guilty.
Statement sufficient
(5) Where an accused pleads autrefois acquit or autrefois convict, it is sufficient if he
(a) states that he has been lawfully acquitted, convicted or discharged under
subsection 730(1), as the case may be, of the offence charged in the count to which
the plea relates; and
(b) indicates the time and place of the acquittal, conviction or discharge under
subsection 730(1).
1. Double jeopardy
226
The rule against double-jeopardy is set to protect the person for being convicted twice for the
same event
s. 11(h) – Canadian Charter of Rights and Freedoms
Arrest or detention
Any person charged with an offence has the right
h) if finally acquitted of the offence, not to be tried for it again and, if finally found guilty
and punished for the offence, not to be tried or punished for it again
R. v. Riddle, [1980] 1 SCR 380
Facts
 Accused was charged with common assault
 He pleaded not guilty and the matter was adjourned for trial
 At the trial the Crown applied for an adjournment as the complainant was not present.
The application was refused. As the Crown called no evidence, the charge was
dismissed and the accused was discharged.
 A week later, the complainant swore a new information in terms identical to the first
information
 When the matter came on before another provincial court judge, the accused entered a
plea of autrefois acquit and the charge was dismissed
 The Alberta CA dismissed the Crown’s appeal
Issues:
1. Can the accused raise the plea of autrefois acquit in a summary conviction court?
2. If so, is the plea available when in an earlier proceeding the charge is dismissed following
non-appearance of the informant and refusal of an adjournment?
Holding: 1. Yes, 2. Yes; appeal dismissed
Dickson J
The proper procedure in summary conviction matters is not to raise the special plea of
autrefois acquit, but simply to enter a general plea of not guilty embracing the concept of
res judicata
The submission that the special pleas authorized in Part XVII (ss. 534-537) of the Criminal
Code, headed “Procedure by Indictment”, are not available in respect of summary conviction
offences, provision for which is contained in Part XXIV, failed
 Something akin to the plea of autrefois acquit was available at common law to a
defendant accused of an offence punishable by summary conviction
Section 743 (now 808) is intended to supplement, and not to supplant, common law rights
In principle, there is no reason why a different situation ought to prevail where the defendant
has not obtained the certified copy as per s. 743
227
 Nor, in principle, is it easy to distinguish between the situation where the Crown
leads evidence which fails to make out a case for the defendant to answer and the
situation where, as here, no evidence is led
 So long as the case has proceeded to a verdict and a dismissal, that should be
sufficient
Lecture Notes
 The principle that the person must not be put in jeopardy twice for the same
offence is a principle of fundamental justice
 This rule can be pleaded in summary convictions
 The case also set out when an accused was to be considered previously in jeopardy
o When there was an acquittal or conviction, this is obviously the case
o The more difficult question is when short of an acquittal or a conviction a person
is in jeopardy?
 Jeopardy begins when there has been a plea and the judge has the
capacity of entering a conviction
 A trial that is stayed and not recommenced, is considered not to have
happened at all
 So the person is not in jeopardy
 If the information is flushed before there being a plea, then there is no
jeopardy either
When are matters considered identical?
 Some of these matters are answered in ss. 609 and of the Code
 It is not whether they just make part of the same transaction, but whether the accused
could have been convicted of the first charge based on the evidence that has been
tendered for the second
s. 609 – Criminal Code
What determines identity
(1) Where an issue on a plea of autrefois acquit or autrefois convict to a count is tried and it
appears
(a) that the matter on which the accused was given in charge on the former trial is the
same in whole or in part as that on which it is proposed to give him in charge, and
(b) that on the former trial, if all proper amendments had been made that might then
have been made, he might have been convicted of all the offences of which he may be
convicted on the count to which the plea of autrefois acquit or autrefois convict is
pleaded,
the judge SHALL give judgment discharging the accused in respect of that count.
Allowance of special plea in part
(2) The following provisions apply where an issue on a plea of autrefois acquit or autrefois
convict is tried:
(a) where it appears that the accused might on the former trial have been convicted of an
offence of which he may be convicted on the count in issue, the judge shall direct that the
228
accused shall not be found guilty of any offence of which he might have been convicted on the
former trial; and
(b) where it appears that the accused may be convicted on the count in issue of an offence of
which he could not have been convicted on the former trial, the accused shall plead guilty or not
guilty with respect to that offence.
R. v. Wigglesworth, [1988] 1 SCR 541
Facts
 Appellant police officer committed a common assault, as defined in the Criminal Code,
which was also a "major service offence" under the Royal Canadian Mounted Police Act
 The major service offence was dealt with first
 The trial judge quashed the information for the charge of common assault under s. 24(1)
of the Charter on the ground that the accused was being tried twice for the same
misconduct contrary to s. 11 of the Charter
 The Saskatchewan Court of Queen's Bench allowed an appeal from that judgment
holding that the common assault charge and the major service offence constituted
separate offences
 An appeal to the Court of Appeal was dismissed
Issue: Does a "major service offence" under the Act precluded subsequent proceedings under
the Criminal Code for the same misconduct on the ground that such proceedings would violate
the accused's right, under s. 11(h) of the Charter?
Holding: No; appeal dismissed
Wilson J
A matter could fall within s. 11 either because by its very nature it is a criminal
proceeding or because a conviction in respect of the offence may lead to a true penal
consequence
 In cases where the two tests conflict the "by nature" test must give way to the "true
penal consequence" test.
If a particular matter is of a public nature, intended to promote public order and welfare
within a public sphere of activity, then that matter falls within s. 11
 This is to be distinguished from private, domestic or disciplinary matters which are
regulatory, protective or corrective and which are primarily intended to maintain
discipline, professional integrity and professional standards or to regulate conduct
within a limited private sphere of activity
The proceedings before the Royal Canadian Mounted Police Service Court are accordingly
neither criminal nor quasi-criminal proceedings
 However, an officer charged and convicted under the Code of Discipline faces a
true penal consequence since conviction can result in imprisonment for one year
Nevertheless, appellant does not have the benefit of s. 11(h) because he was not being tried
and punished for the same offence
229

The "offences" were quite different; one was an internal disciplinary matter where the
accused was found guilty of a major service offence, and the other was the criminal
offence of assault where the accused must account to society at large for his
conduct
Lecture Notes
 RCMP officer convicted of assault under RCMP proceedings
 He later appears in provincial court for the same assault
 One question was whether the RCMP committee could be have been considered to have
given a penal sanction
 Was the RCMP proceeding a criminal proceeding?
o The first question is whether the proceeding was penal by nature
 The court finds that the proceeding was not by nature penal, but instead
regulatory
o The second question was whether the proceeding have penal consequences,
however?
 If the consequences are such that they amount to being penal, then it
could offend s.11(h)
 S. 11 was not considered infringed in this case, however, because the accused had
committed two separate offences
o Assault AND an assault that is contrary to his duties
o It is thus acceptable to be punished for both
2. Multiple punishment
The policy forbidding multiple punishment has at its aim that a wrongdoer receives an
adequate penalty, but not more
R. v. Kinnear (2005), 30 CR (6th) 1 (Ont. CA)
Facts
 Accused was charged with theft, threatening death, escaping lawful custody, using
imitation firearm while committing indictable offence of threatening death, using
imitation firearm while committing indictable offence of escape custody
 Trial judge entered convictions on all counts
 Trial judge, on his own initiative, raised question of constitutionality of s. 85(4) of the
Criminal Code which provides that any sentence imposed upon conviction on charge of
using imitation firearm while committing indictable offence shall be served
consecutively to any other sentence imposed at same time
 The judge ruled that s. 85(4) contravened prohibition against cruel and unusual
punishment in s. 12 of the Charter
 There was also the issue that some of the convictions should be set aside pursuant to the
Kienapple rule against multiple convictions
Issue: Were the multiple convictions?
Holding: Yes; appeal allowed
Doherty JA
230
The court applied the Kienapple rule to stay two of three charges relating to use of firearm
 Section 85(4) does not abrogate Kienapple rule
 The rule in Kienapple applies unless Parliament has clearly indicated otherwise
In R. v. Kienapple the court recognized and arguably expanded the common law rule
prohibiting more than one conviction for the same criminal wrong
 The principle foreclosed more than one conviction for offences arising out of the
same delict
 The relevant inquiry so far as res judicata is concerned is whether the SAME
CAUSE OR MATTER (rather than the same offence) is comprehended by two or
more offences.
The principle that emerged from Kienapple provides that where the same transaction gives
rise to two or more offences with substantially the same elements and an accused is found
guilty of more than one of those offences, that accused should be convicted of only the
MOST SERIOUS of the offences
 The other charges should be stayed
Laskin J linked the Kienapple rule to the court's power to protect against abuses of its
process
 He described the rule as designed to "protect an individual from an undue exercise by
the Crown of its power to prosecute and punish".
Lecture Notes
 The accused was caught shoplifting socks, when stopped by the police he pulled a gun
to force the guard to let them go
 The gun was found to not be real
 He was charged with two counts of using an imitation firearm while carrying out an
indictable offence
 He was also charged with using an imitation gun while running from a police officer
o These law two charges could have been considered redundant
 The court uses a factual and legal nexus so as to assess the violation of the the
Kienapple principle (i.e. two offences with substantively the same elements)
o In this case both offences did in fact arise from one single act
o Did these constitute a single criminal wrong or delict?
o The crucial distinction from Kienapple is one between different wrongs, and the
same wrong being committed in different ways
 The court finds that some charge here did deal with the same wrong committed in
different ways
o Being charged with possessing a firearm while committing an indictable offence
and possessing a firearm while committing a specific offence (which is
indictable) amounts to the same wrong
231